Download as pdf or txt
Download as pdf or txt
You are on page 1of 132

Subject Code & Name: MA 8151 / ENGINEERING MATHEMATICS I Class Notes 2018 - 2019

DEPARTMENT OF MATHEMATICS
UNIT I - DIFFERENTIAL CALCULUS

REPRESENTATION OF FUNCTION:

Machine diagram for a function f

A function f is a rule that assigns to each element x in a set D exactly one element, called f (x), in a set E.
The set D is called the domain of the function.
The number f (x) is the value of f at x and is read “ f of x.”
The range of f is the set of all possible values of f (x) as x varies throughout the domain.
A symbol that represents an arbitrary number in the domain of a function f is called an independent
variable.
A symbol that represents a number in the range of f is called a dependent variable.

There are four possible ways to represent a function:


 verbally (by a description in words)
 numerically (by a table of values)
 visually (by a graph)
 algebraically (by an explicit formula)
Problem 1: Find the domain of each function.
1
(a) f ( x)  x  2 (b) g ( x)  2
x x
Solution:
(a) Because the square root of a negative number is not defined (as a real number), the domain of f consists
of all values of x such that x + 2 ≥ 0.
This is equivalent to x ≥ -2, so the domain of f(x) is the interval [-2, ∞).
1 1
(b) Since g ( x)  2 
x  x x( x  1)
When x = 0 or x = 1 g(x) is not defined. Therefore the domain of g(x) is the interval (-∞, 0)(0, 1)(1, ∞)

LIMIT OF A FUNCTION:

A limit is the idea of looking at what happens to a function as you approach particular values of x.
A limit is strictly the behavior of a function “near” a point.
Left-hand and right-hand limits are the idea of looking at what happens to a function as you approach a
particular value of x, from a particular direction.
The left hand limit is the limit of f(x) as x approaches the value of a from the left is written lim f ( x)
x a

The right hand limit is the limit of f(x) as x approaches the value of a from the right is written lim f ( x)
x a

Page No. 1
Subject Code & Name: MA 8151 / ENGINEERING MATHEMATICS I Class Notes 2018 - 2019

Example 1 : Consider the graph of f(x) in the given figure below.

The following observations can be made


when x = -2, we notice there is a “break” in the function.
However, if we approach x = -2 “from the left” (Figure a) we can see that the function values are getting
closer and closer to 1.
On the other hand, if we approach x = -2 “from the right” (Figure b) we can see that the function values are
getting closer and closer to 3.
Figure a Figure b

When x = 1, we notice there is a hole in the function.


If we approach f(x) from the left or from the right (Figure c), we can see that the function values are getting
closer and closer to 2.
Figure c

lim f ( x)  1 lim f ( x)  3 lim f ( x)  2 lim f ( x)  2


Therefore, we have x 2 x 2 x 1 x 1

Page No. 2
Subject Code & Name: MA 8151 / ENGINEERING MATHEMATICS I Class Notes 2018 - 2019

Problem 2: Using the given graph of g(x), find the following left- and right-hand limits.

lim g ( x)
x 0

lim g ( x)
x 0

lim g ( x)
x 1

lim g ( x)
x 1

Solution:
The graph of g(x) implies as x approaches 0 from the left, we can see that the function values are getting
closer and closer to -1. (Fig a) So, lim g ( x)  1
x  0

As x approaches 0 from the right, we can see that the function values are getting closer and closer to -1.
(Fig b)So, lim g ( x)  1
x 0

As x approaches 1 from the left, we can see that the function values are getting closer and closer to -2.(Fig
c) So, lim g ( x)  2
x 1

As x approaches 1 from the right. You can see that the function values are getting closer and closer to -2.
(Fig d) So, lim g ( x)  2
x 1

Fig a Fig b Fig c Fig d

By considering both the left- and right-hand limits of a function as you approach a particular value of x,
you can determine whether or not the limit of the function at that point exists.

DEFINITION : For a given function y  f ( x) , if lim f ( x)  lim f ( x)  l , finite answer, we say that
x a xa

limit exists at x  a and we write that lim f ( x)  l .


xa

Otherwise limit does not exist.

Page No. 3
Subject Code & Name: MA 8151 / ENGINEERING MATHEMATICS I Class Notes 2018 - 2019

Example 2: Using the graph of f(x) below, find the following limits.

lim f ( x)
x 1

lim f ( x)
x 2

lim f ( x)  2 lim f ( x)  2
Already in example 1, we have got x1 and x1 .
Therefore, by definition, since lim f ( x)  lim f ( x)  2 , then lim f ( x)  2 .
x 1 x 1 x 1

[Note : It is important to notice that this limit exists even though f(1) does not exist.]
lim f ( x)  1 lim f ( x)  3
Again by example 1, x2 and x2 .
Therefore, by definition, since lim f ( x)  lim f ( x) , then lim f ( x) does not exist.
x 2 x 2 x 2

1
Problem 3: Find lim , if it exists
x 1 x  1

1
Solution: Consider f(x)=
x 1
X .9 0.99 0.999 0.9999 1.0 1.0001 1.001 1.01 1.1
f(x) -10 -100 -1000 -10000 - 10000 1000 100 10
1 1
By the observation of the values lim   and lim   . Thus limit does not exist.
x 1 x  1 x 1 x  1

ex 1
Problem 4: Use conjecture method to determine the limit lim .
x 0 x
ex 1
Solution: Consider f(x)= x
X -1 -0.1 -0.001 -0.0001 0.0 0.0001 0.001 0.1 1
f(x) 0.632 0.956 0.999 1.000 - 1.000 1.001 1.052 1.718
From the table it appears that as x gets closer to 0, the function f(x) gets closer to 1.
ex 1
lim 1
Therefore we conclude x0 x
lim x  0
Problem 5: Show that x 0
 x, x  0
Solution: x  
  x, x  0
Since x  x , for x  0 ,  lim x  lim x  0
x 0 x 0

For x  0 , x   x, lim x  lim   x   0


x0 x0

 lim x  lim x  lim x  0


x 0 x 0 x 0

Page No. 4
Subject Code & Name: MA 8151 / ENGINEERING MATHEMATICS I Class Notes 2018 - 2019

x
Problem 6: Prove that lim does not exist
x0 x
 x, x  0
Solution: x  
  x, x  0
x x
For x  0 , x  x,  lim  lim
1
x 0
x x x 0

x x
For x  0 , x   x,  lim  lim  1
x 0 x x 0 x

x x
 lim  lim  limit does not exist.
x 0 x x 0 x

 1  x; x  1

Problem 7: Sketch the graph of the function f  x    x 2 ;  1  x  1 and use it to determine the value
 2  x; x1

of “a” for which lim f ( x ) exists? [Jan 2018]
xa

From the graph lim f ( x) exists for all ‘a’ except when a = -1,
xa

 the left and right limits are different at a = -1


1
Problem 8: Find lim 2
x 0 x

Solution:
x - - - - - -0.01 -0.001 0 1 0.5 0.2 0.1 0.05 0.01 0.001
1 0.5 0.2 0.1 0.05
f(x)= 1 4 25 100 400 10,000 1,000,000 - 1 4 25 100 400 10,000 1,000,000
2
1/x
As x becomes close to 0, x2 also becomes close to 0, and 1yx2 becomes very large. Thus the values of f
(x) can be made arbitrarily large by taking x close enough to 0. Thus the values of f (x) do not approach a
1
number, so lim 2 does not exist.
x 0 x

Page No. 5
Subject Code & Name: MA 8151 / ENGINEERING MATHEMATICS I Class Notes 2018 - 2019

x4  1
Problem 9: Evaluate lim , if it exists.
x 1 x3  1
Solution:

lim 3
x4  1
 lim
 
x2  1 x2  1 
x 1 x  1

x 1 x  1

 x2  x  1 
 x  1 x  1  x 2  1
 lim
x 1
 x  1  x 2  x  1
 x  1  x 2  1  1  11  1 4
 lim  
x 1
 x 2  x  1  1  1  1 3
t2  9  3
lim
Problem 10: Evaluate t 0 t2
t2  9  3 t2  9  3 t2  9  3
Solution: lim  lim
t 0 t2 t 0 t2 t2  9  3
t2  9  9 1 1 1
 lim  lim 
t 0 t2 t2  9  3 t 0
t2  9  3 6
x2  4
Problem 10: lim  lim( x  2)  4
x 2 x  2 x 2

x 8
3
Problem11: lim  lim( x 2  2 x  4)  12
x 2 x  2 x 2

x2  5x  6
Problem12: lim  lim( x  3)  1
x 2 x2 x 2

x  4x  3
2
x2 1
Problem13: lim  lim  lim  0
x  x 2
3 x  x 3 x  x

x  4x  3
2
x 2
Problem 14: lim  lim 2  1
x  x 2
2 x  x

Page No. 6
Subject Code & Name: MA 8151 / ENGINEERING MATHEMATICS I Class Notes 2018 - 2019

x4  4 x  3 x4 x
Problem15: lim  lim  lim   ,or Does Not Exist.
x  x 2
3 x  x 3 x  1

The Squeeze theorem (or) The Sandwich Theorem (or) The Pinching Theorem:
If f(x)≤ g(x) ≤ h(x) when x is near a (except possibly at a) and lim f ( x)  lim h( x)  L then lim g ( x)  L
x a x a xa

1
Problem16 : Show that lim x 2 sin    0
x 0
 x
1
Solution: Let g  x   x 2 sin  
 x
1
Since lim sin   does not exist
x 0
x
 we cannot use lim x 2 sin  1   lim x 2 . lim sin  1 
x 0
 x  x 0 x 0
 x
1 1
Since 1  sin    1   x 2  x 2 sin    x 2
x x

Since lim x 2  0 and lim  x 2  0
x 0 x 0

1
 lim x 2 sin    0
x 0
x

CONTINUNITY OF A FUNCTION:
Definition: A function f is continuous from the right at a point a if lim f ( x )  f ( a ) and f is continuous
xa

from the left at a if lim f ( x )  f ( a )


xa

Definition: The function f(x) is continuous at a point a if lim f ( x )  f ( a ) .


xa

(i.e) 1. f(a) is defined


lim f ( x )
2. xa exists
lim f ( x )  f ( a )
3. x a
Otherwise the function is discontinuous.
Geometrically, a function is continuous on an interval if its graph is connected – that means, it has
no breaks (no holes, no jumps).
Note:
The following types of functions are continuous at every number in their domains:
 polynomials
 rational functions
 root functions
 trigonometric functions
 inverse trigonometric functions
 exponential functions
 logarithmic functions

Page No. 7
Subject Code & Name: MA 8151 / ENGINEERING MATHEMATICS I Class Notes 2018 - 2019

Intermediate value theorem:


Suppose that f is continuous on the closed interval  a, b and let N be any number between f  a  and
f  b  , where f  a   f  b  , then there exists a number c in  a, b  such that f  c   N

 x 2  1, for x  0

Problem 17: Let the function f(x) be defined for all values of x by f ( x)   2
 x  1, for x  0

Draw the graph of the function f(x) and test the continuity from the graph.

Solution.

The function f(x) is discontinuous at x = 0

Because lim f ( x)  1 and lim f ( x)  1 and f (0)  1 . In this case lim f ( x) does not exist.
x 0 x 0 x 0

1
Problem 18 : Determine the value of x at which the function f ( x)  is continuous.
2 x
Solution.
The domain of the function is ( , 2) . The function is not defined at x = 2. Thus the function is
continuous for all values of x in the interval ( , 2) .
The graph of f(x).

 x  2, x  0

Problem 19: Find lim f ( x) , also check the continuity for f ( x)  0 x0
x 0
2 x  2, x  0

Solution:

lim f ( x)  lim 2 x  2  2
x  0 x 0

Page No. 8
Subject Code & Name: MA 8151 / ENGINEERING MATHEMATICS I Class Notes 2018 - 2019

lim f ( x)  lim x  2  2
x  0 x 0 ,
limit exists and we have lim f ( x )  2 .
x 0

But f (0)  0 . The function is not continuous.


Thus f(x) has removable discontinuity at x = 0.
Problem 19: For what value of the constant “c” is the function “f” continuous on (-∞, ∞),
 cx 2  2 x , x2
f  x   3
 x  cx , x2
Solution:
cx 2  2 x & x3  cx are polynomials in x  They are continous everywhere in (-∞, ∞).
The only point to be checked is x = 2.
lim f ( x)  lim x3  cx  23  c(2)  8  2c
x 2 x 2

lim f ( x)  lim cx2  2 x  c22  2(2)  4c  4


x 2 x 2

If “f ” is continuous at x = 2 , then lim f ( x)  lim f ( x)  f (2)


x 2 x 2

2
 8  2c  4c  4  6c  4  c 
3
 2  x; x2
 2
Problem 20: Find the values of ‘a’ and ‘b’ such that the function f  x    ax  bx  3; 2 x 3
 2 x  a  b; x3

is continuous everywhere.
Solution:
If “f ” is continuous everywhere in (-∞, ∞), then
lim f ( x)  lim f ( x)  f (2) -------------------------(1)
x 2 x 2

and lim f ( x)  lim f ( x)  f (3) --------------------- (2)


x 3 x 3

lim f ( x)  lim ax2  bx  3  a22  b(2)  3  4a  2b  3


x 2 x 2

lim f ( x)  lim 2  x  2  2  4
x  2 x 2

By (1), 4a  2b  3  4  4a  2b  1------------------(3)

lim f ( x)  lim 2 x  a  b  2(3)  a  b  6  a  b


x 3 x 3

lim f ( x)  lim ax2  bx  3  a(3)2  b(3)  3  9a  3b  3


x 3 x 3

By (2), 6  a  b  9a  3b  3 10a  4b  3 ------------------(4)

2 X (3)  8a - 4b = 2

(4)  10a – 4b = 3

Subtracting, -2a = -1 a = ½

Page No. 9
Subject Code & Name: MA 8151 / ENGINEERING MATHEMATICS I Class Notes 2018 - 2019

(4)  10a – 4b = 3  10(1/2) – 4b = 3 5-4b=3-4b = -2b = ½ .


Problem 21: For the following f(x), find the points at which f(x) is discontinuous. Also, find the points at
 1  x; x1

which f(x) is continuous from right and from the left. Justify your answer. f  x    1 x ; 1 x  3

 x  3; x3
Solution:
lim f ( x)  lim1/ x  1
x 1 x 1

lim f ( x)  lim1  x  2
x 1 x 1

lim f ( x)  lim f ( x) At x = 1, the function is discontinuous


x 1 x 1

 1  x; x 1

lim f ( x)  lim x  3  0 f  x    1 x ; 1 x  3
x 3 x 3

 x  3; x3
lim f ( x)  lim1 x  1
3
x 3 x 3

lim f ( x)  lim f ( x) At x = 3, the function is discontinuous


x 3 x 3

lim f ( x)  0  f (3) At x = 3, the function is continuous from right.


x 3

lim f ( x)  2  f (1) At x = 1, the function is continuous from left.


x 1

DERIVATIVE OF A FUNCTION
The derivative of a function represents the rate of change of a variable with respect to another
variable. For example, the velocity of a body is defined as the rate of change of the location of the body
with respect to time. The location is the dependent variable while time is the independent variable. Now if
we measure the rate of change of velocity with respect to time, we get the acceleration of the body. In this
case, the velocity is the dependent variable while time is the independent variable.
f ( a  h)  f ( a )
The derivative of a function f ( x) at x  a is defined as f (a )  lim
h 0 h
Note:
1. If f is differentiable at a, then f is continuous at a.
2. The converse is not always true. (i.e) A function can be continuous at a, but not differentiable at a.
Problem 22: Find the domain at which the function f(x) = |x| is continuous and differentiable.

Solution:

 x, x  0
x 
  x, x  0

For x  0 , x  x,  lim x  lim x  0


x 0 x 0

For x  0 , x   x, lim x  lim   x   0


x0 x0

 lim x  lim x  lim x  0


x0 x0 x0

Page No. 10
Subject Code & Name: MA 8151 / ENGINEERING MATHEMATICS I Class Notes 2018 - 2019

At x  0 , f(x) is continuous.
Therefore the domain of the continuity of f(x) is (-∞, ∞)
Consider x  0 Since x  x,
f ( x  h)  f ( x ) | xh|| x| xhx h
f ( x)  lim  lim  lim  lim  1
h 0 h h 0 h h 0 h h 0 h

f(x) is differentiable for x  0


Consider x  0 Since x   x,
f ( x  h)  f ( x ) | xh|| x| ( x  h)  [( x)] h
f ( x)  lim  lim  lim  lim  1
h 0 h h 0 h h 0 h h 0 h

f(x) is differentiable for x  0


Consider x  0 Since x  0,
f (0  h)  f (0) |0h||0| |h|
f (0)  lim  lim  lim
h 0 h h  0 h h  0 h
|h| h
Now lim  lim  1
h 0 h h 0 h

|h| h
and lim  lim  1
h 0 h h 0 h

|h| |h| |h|


lim  lim  lim does not exist.  f (0) does not exist.
h 0 h h 0 h h 0 h

f(x) is differentiable for x  0 and x  0 but not differentiable at x  0 .


Therefore the domain of the differentiability of f(x) is (-∞, 0)(0, ∞)
Problem 23: Find f (3) if f ( x)  4 x 2 .
Solution:
f (3  h)  f (3)
f (3)  lim
h 0 h
4(3  h)2  4(3)2
 lim
h 0 h
4(9  h  6h)  36
2
 lim
h 0 h
36  4h 2  24h  36 h(4h  24)
 lim  lim  lim(4h  24) =24
h 0 h h 0 h h 0

 
Problem 24: Find f    if f ( x)  sin(2 x)
4
          
f   h  f   sin  2   h    sin  2   
   4   lim   4    4 
f     lim  
4
Solution:
 4  h 0 h h 0 h
   
sin   2h   sin  
 lim 2  2
h 0 h

Page No. 11
Subject Code & Name: MA 8151 / ENGINEERING MATHEMATICS I Class Notes 2018 - 2019

     
sin   cos(2h)  cos   sin(2h)  sin  
 lim  
2 2  2   lim cos (2h)  0  1
h 0 h h 0 h
cos (2h)  1
 lim =0
h 0 h
Problem 24: Find the derivative of f ( x)  10 x6 .
d d
Solution: f ( x)  10 x6 f ( x)  (10 x 6 )  10 x 6  10(6 x5 )  60 x5
dx dx
Derivatives of logarithmic and exponential functions
For the function of the type f ( x)  a x , where a is a constant, then f '( x)  a x ln a .
Derivative of f ( x)  e x is f '( x)  e x
1
For the logarithmic function f ( x)  ln x , f '( x )  .
x
1
Derivative of f ( x)  log a x is f '( x)  .
x ln a
dy
Problem 25: Find . a. f ( x)  3x 20  10 x b. f ( x)  e x  e5 x
dx
Solution.
df df
a.  60 x19  10 x ln10, b.  e x  5e5 x
dx dx
dy
Problem 26: Find . a. f ( x)  3ln x  10 b. f ( x)  log x  3
dx
df 3 df 1
Solution. a.  , b. 
dx x dx x ln10

Problem 27: Find the derivative of f ( x)  3x3  8 .


Solution:
f ( x)  3 x 3  8
d d d d
f ( x)  (3x 3  8)  (3 x 3 )  (8)  3 ( x 3 )  0
dx dx dx dx
 3(3x 2 )  9x 2
Problem 28: Differentiate the functions (addition/subtraction).

a. f ( x)  x  2 x  1 b. f ( x)  3x  5 x c. f ( x)  3  x
2 20 3
x
1 1
Solution. a. f '( x)  2 x  2 b. f '( x)  60 x19  5 c. f '( x)  
4/3

x 2 x
d d
Product Rule: If f ( x)  u ( x)v( x) , then f ( x)  u ( x) v( x)  v( x) u ( x) .
dx dx
Problem 29: Find the derivative of f ( x)  (2 x 2  6)(3x3  8)
Solution : Using the product rule,
Page No. 12
Subject Code & Name: MA 8151 / ENGINEERING MATHEMATICS I Class Notes 2018 - 2019

f ( x)  (2 x 2  6)(3x 3  8)
Let u( x)  2 x 2  6 and v( x)  3x3  8
Taking the derivative of u ( x) ,
du d d d d
 (2 x 2  6)  (2 x 2 )  (6)  2 ( x 2 )  0
dx dx dx dx dx  2(2 x)  4 x
Taking the derivative of v ( x ) ,
dv d d d d
 (3 x3  8)  (3x3 )  (8)  3 ( x 3 )  0  3(3 x 2 )  9 x 2
dx dx dx dx dx
Using the formula for the product rule
d d
f ( x)  u ( x) v( x)  v( x) u ( x)  ( 2 x 2  6 )( 9 x 2 )  ( 3x3  8 )( 4 x )
dx dx
 18 x 4  54 x 2  12 x 4  32 x  30 x 4  54 x 2  32 x

Problem 30: Differentiate the functions a. f ( x)  ( x 2  2 x  1) x10 b. f ( x)  (3x 20  2)(5x)

Solution.

a. f '( x)  (2 x  2) x  ( x  2 x  1)10 x
10 2 9

b. f '( x)  (60 x )5x  (3x  2)5


19 20

Quotient Rule:

d d
u ( x) v( x) u ( x)  u ( x) v( x)
If f ( x)  , then f ( x)  dx dx
v( x) (v( x)) 2

(2 x 2  6)
Problem 31: Find the derivative of f ( x)  .
(3x3  8)
(2 x 2  6)
Solution: Using the Quotient Rule, f ( x) 
(3x3  8)
Let u( x)  2 x 2  6 and v( x)  3 x 3  8
Taking the derivative of u ( x) ,

du d d d d
 (2 x 2  6)  (2 x 2 )  (6)  2 ( x 2 )  0  2(2 x)  4 x
dx dx dx dx dx

Taking the derivative of v ( x ) ,

dv d d d d
 (3 x3  8)  (3x3 )  (8)  3 ( x 3 )  0  3(3 x 2 )  9 x 2
dx dx dx dx dx

Page No. 13
Subject Code & Name: MA 8151 / ENGINEERING MATHEMATICS I Class Notes 2018 - 2019

d d
v( x) u ( x)  u ( x) v( x)
Using the formula for the quotient rule, f ( x)  dx dx
(v( x)) 2

(3x3  8)(4 x)  (2 x2  6)(9 x 2 ) 12 x 4  32 x  18 x 4  54 x 2 6 x 4  54 x 2  32 x


f ( x)   
(3x3  8)2 9 x6  48 x3  64 9 x 6  48 x3  64

3x 2  5
Problem 32: Differentiate the function f ( x) 
x
1
x (6 x)  (3x 2  5)
Solution: f '( x)  2 x
x
The chain rule:
dy dy du
The chain rule is  
dx du dx
dy
Problem 33: Find by chain rule.
dx
a. y  u10 and u  1  x5 b. f ( x)  (3x 20  2)21 c. f ( x)  x 2  1
Solution.
dy dy du
a.    10u 9  (5 x 4 )  50 x 4 (1  x5 )9
dx du dx
dy
b.  21(3x 20  2) 20 (60 x19 )  1260 x19 (3 x 20  2) 20
dx

dy 2x
c. 
dx 2 x 2  1

Problem 34: Find f ( x ) if f ( x )  ln( 3x  2 x )


2

dy dy du d (ln u ) du
Solution: By chain rule.     u  3x 2  2 x
dx du dx du dx

dy d (ln u ) du 1 d (3x 2  2 x) 1
     2  (6 x  2)
dx du dx u dx 3x  2 x

Problem 35: If f ( x)  sin(cos(tan x)), find f ( x)

d  sin(cos(tan x)) 
Solution: f ( x) 
dx

d (cos(tan x))
f ( x)  cos(cos(tan x))
dx
d (tan x)
 cos(cos(tan x))[ sin(tan x)]
dx

Page No. 14
Subject Code & Name: MA 8151 / ENGINEERING MATHEMATICS I Class Notes 2018 - 2019

  cos(cos(tan x))sin(tan x)sec2 x

Problem 36: Differentiate y  esec3

Solution:

dy d e

sec3

 esec3
d  sec 3 
d d d
d  3 
 esec3 sec 3 tan 3
d
sec3
e sec 3 tan 3 (3)  3esec3 sec 3 tan 3

Implicit differentiation:
Functions like 3 y 2  5xy  9 xy5  2  0 , where x can not be written as independent variable with respect to
another variable y. In this case direct differentiation is either difficult or impossible. To find derivative we
will use implicit differentiation using chain rule. Following examples will illustrate the situation.
dy
Problem 37: 1. Find or y ' .
dx
a. x2  y 2  2 xy b. 3 y 2  5xy  9 x  2  0 b. y 2  log x  3
Solution:
a. x2  y 2  2 xy
d 2 d
(x  y 2 )  (2 xy)
dx dx

d 2 d d
(x )  ( y 2 )  (2 xy)
dx dx dx

dy dy
2x  2 y  2x  2y
dx dx

dy dy
2y  2x  2 y  2x
dx dx
dy
(2 y  2 x)  2 y  2x
dx

dy 2 y  2 x

dx 2 y  2 x
dy
1
dx
5y  9
b. 6 yy ' 5 y  5 xy ' 9  0  y ' 
6 y  5x

1 1
c. 2 yy '   y' 
x ln10 2 xy ln10
Problem 38: If x 2  xy  y 2  5 , find the value of y and y .
Page No. 15
Subject Code & Name: MA 8151 / ENGINEERING MATHEMATICS I Class Notes 2018 - 2019

Solution: x 2  xy  y 2  5
d 2 d
( x  xy  y 2 )  (5)
dx dx

d 2 d d
( x )  ( xy)  ( y 2 )  0
dx dx dx
dy dy
2x  x  y  2 y 0
dx dx
dy
( x  2 y )  2 x  y
dx
dy y  2 x

dx 2 y  x
y  2x
y 
2y  x
(2 y  x) y   y  2 x
d d
(( 2 y  x) y )  ( y  2 x)
dx dx
d d d d
(2 y  x) ( y)  y (2 y  x)  ( y )  (2 x)
dx dx dx dx
y(2 y  x)  y(2 y  1)  y  2
2 y  2  2 y2
y 
2y  x
After substitution of y ,
2
y  2x  y  2x 
2  2  2 
2y  x  2 y  x  6( y 2  xy  x 2 )
y  
2y  x (2 y  x) 3

if sin  x  y   y 2 cos x
dy
Problem 39 : Find
dx
Solution: Given sin  x  y   y 2 cos x
Diff. w.r.to x, we get,

   
cos  x  y  1  y '  y 2   sin x    cos x  2 yy '

cos  x  y   cos  x  y  y'   y 2 sin x  2 yy' cos x

cos  x  y   y 2 sin x   2 y cos x  cos  x  y   y '


cos  x  y   y 2 sin x
 y' 
2 y cos x  cos  x  y 
Problem 40 : If x4 + y4 = 16, find the value of y and y .
Solution: x 4  y 4  16
4 x3 x3
4 x  4 y y  0  y  
3 3
 3
4 y3 y
Page No. 16
Subject Code & Name: MA 8151 / ENGINEERING MATHEMATICS I Class Notes 2018 - 2019

Differentiating with respect to x


 3 2   3 y 2 x 2  y  xy 
 y (3x )  x3 (3 y 2 y) 
y     
   
2
y 3
 y6 
 
 2 2   x3     2  y ( y 3 )  x( x 3 )  
 3 y x  y  x 
 y3     3x  
x3        y3 
y   3  y      
y y6  y4 
   
   
  y 4  x 4    16 
   3 x 2  3 4      3 x 2  7   y 4  x 4  16
  y    y 
48 x 2
 7
y
Equations of a tangent line

Problem 41: Find the slope of the tangent line to the curve 3xy  2 x 2  7 at (1, 3). Also compute the
second derivative.
4 x  3 y 4(1)  3(3) 5
Solution. 3xy ' 3 y  4 x  0  y '   
3x 3(1) 3
For the second derivative we consider again 3xy ' 3 y  4 x  0

4  6 y ' 6 y  4 x 14
Taking derivative we find 3xy " 3 y ' 3 y ' 4  0  y "   
3x 3x 2 3

Problem 42: Find the tangent line to the equation x3  y 3  6 xy at the point (3, 3) and at what point the
tangent line is horizontal in the first quadrant.
Solution:
Differentiating both sides of x3 + y3 = 6xy with respect to x, regarding y as a function of x, and using the
Chain Rule on the term y3 and the Product Rule on the term 6xy, we get 3x2 +3 y2 y= 6xy+6y

x2 + y2 y= 2xy+2y

y2 y - 2xy = 2y - x2

y (y2 - 2x) = 2y - x2

2 y  x2
y 
y2  2x

2 y  x 2 2(3)  32
At (3, 3), y  2   1
y  2 x 32  2(3)

Slope ofthe tangent is -1

Page No. 17
Subject Code & Name: MA 8151 / ENGINEERING MATHEMATICS I Class Notes 2018 - 2019

Equation of the tangent is given by y  y1  m  x  x1 


 y  3  (1)  x  3  y  3   x  3  x  y  6
The tangent line is horizontal if y  0
2 y  x2 x2
  0  2 y  x 2
 0  2 y  x 2
 y 
y2  2x 2 provided y 2  2 x  0
x2
y in x 3  y 3  6 xy
2
3
 x2   x2 
 x     6x  
3

 2   2
x6
x 
3
 3x3
8
8 x3  x 6
  3x3
8
 8 x  x 6  24 x3
3

 8 x3  24 x3   x 6
 16 x3   x 6
 x 6  16 x3  0  x3 ( x3  16)  0  x  0, (16) 3  x  0, (24 ) 3  x  0, (2) 3
1 1 4

2   2   2
4 2 8
3 3
2
x 8 1
y   23
5
3

2 2 2
4 5
Thus the tangent line is horizontal at (2 3 , 2 3 )
Problem 43: If xsiny + ysinx = 1, find the value of y .
Solution: xsiny  ysinx  1
x(cos y ) y  (sin y )(1)  y (cosx)+y(sinx)=0
y[ x(cos y )  sin x]   sin y  y (cosx)
 sin y  ycosx 
y    
 x cos y  sin x 
 sin y  ycosx 
y    
 x cos y  sin x 
ex  e
Problem 44: Find an equation of the tangent line to the curve y  at the point  1, 2 
1  x2  
Solution:
ex
Given y 
1  x2

dy 1  x  dxd  e   e
2 x x d
dx

1  x2   
1  x 2 e x  e x  2 x  e x  1  x 2
   
1  x     
2 2 2
dx 2
1  x2 1  x2

Page No. 18
Subject Code & Name: MA 8151 / ENGINEERING MATHEMATICS I Class Notes 2018 - 2019

 e dy
The slope of the tangent line at 1,  is 0
 2 dx x 1

 e e
This means that the tangent line at 1,  is horizontal and its equation is y 
 2 2
Problem 45: At what point on the curve y  e x is the tangent line parallel to the line y  2 x ?
Solution:
Given y  e x , we have y  e x . Let the x-coordinate of the point to be a. The the slope of the tangent line
at that point is e a . This tangent line will be parallel to the line y  2 x if it has the same slope, that is 2.
Equating slopes, we get ea  2  a  log 2
 the required point is  a, e a    log 2, 2  .

MAXIMUM AND MINIMUM VALUES OF f:

The maximum and minimum values of f are called extreme values of f .

Definition Let c be a number in the domain D of a function f. Then f (c) is the

absolute maximum or global maximum value of f on D if f (c) ≥ f (x) for all x in D.

absolute minimum or global minimum value of f on D if f (c) ≤ f (x) for all x in D.

Definition f (c) is the

local maximum value of f if f (c) ≥ f (x) when x is near c.

local minimum value of f if f (c) ≤ f (x) when x is near c.

Definition A critical number of a function f is a number c in the domain of f such that either f (c) =0 − 0
or f (c) does not exist.

Extreme Value Theorem: If f is continuous on the closed interval [a,b], then f attains an absolute
maximum value f(c) and a absolute minimum value f(d) at some numbers c and d in [a,b].

Fermat’s Theorem: If f has a local maximum or minimum at c, and if f’(c) exists, then f’(c) = 0. Or
rephrased: If f has a local maximum or minimum at c, then c is a critical number of f.

Closed Interval Method:


To find the absolute maximum and minimum values of a continuous function f on a closed interval [a,b]:
1. Find the values of f at the critical numbers of f in (a,b) (the open interval)
2. Find the values of f at the endpoints of the interval, f(a) and f(b)
3. The largest value from steps 1 and 2 is the absolute maximum value; the smallest of theses
values is the absolute minimum value.
Increasing/Decreasing Test (I/D test):

(a) If f (x)> 0 on an interval, then f is increasing on that interval.

(b) If f (x)< 0 on an interval, then f is decreasing on that interval.

Page No. 19
Subject Code & Name: MA 8151 / ENGINEERING MATHEMATICS I Class Notes 2018 - 2019

Definition
If the graph of f lies above all of its tangents on an interval I, then it is called concave upward on I.
If the graph of f lies below all of its tangents on I, it is called concave downward on I.

Concavity Test:
Suppose that a function f is differentiable on an open interval containing c and that f ( c )
exists. Then
1. If f ( c )  0 , then the graph of f is concave upward at the point P ( c , f ( c ) )
2. If f ( c )  0 , then the graph of f is concave downward at the point P ( c , f ( c ) ) .

Definition: Inflection Point (Point of Inflection or Flex Point):


A point P on a curve y = f (x) is called an inflection point if f is continuous there and the curve changes
from concave upward to concave downward or from concave downward to concave upward at P.
(i.e) A point P on a curve y = f (x) is called an inflection point if f is continuous there and if the
concavity of the curve changes there (from upward to downward, or from downward to upward).
Note:
1. The possible points for inflection points are points where either f ″ = 0 or f ″ is undefined.
2. However, just like that not every critical point is a local max / min, not every such point is an inflection
point either.
3. They are just the pool of points you need to check in order to find the inflection point(s) of a curve.
4. It often happens that a graph has different concavity on the two sides of a vertical asymptote.
5. However, because a curve is not continuous at a vertical asymptote, it can never have an inflection point
there, even if f is defined there.

The First Derivative Test


Suppose that c is a critical number of a continuous function f.
(a) If f changes from positive to negative at c, then f has a local maximum at c.
(b) If f  changes from negative to positive at c, then f has a local minimum at c.
(c) If f  is positive to the left and right of c, or negative to the left and right of c, then f has no local
maximum or minimum at c.

The Second Derivative Test


Suppose f  is continuous near c. If f is differentiable on an open interval containing c, then
1. If f ( c )  0 and f ( c )  0 , then f has a local maximum occurs at x  c
2. If f ( c )  0 and f ( c )  0 , then f has a local minimum occurs at x  c
3. If f ( c )  0 and f ( c )  0 , then the test fails and First Derivative Test have to be used to classify
whether a local maximum, local minimum, or neither occurs at x  c .
Problem 46: Find the absolute maximum and minimum values for f  t   t 4  t 2 in the interval [-1,2].
Solution:
f t   t 4  t 2

Page No. 20
Subject Code & Name: MA 8151 / ENGINEERING MATHEMATICS I Class Notes 2018 - 2019

1
f   t   4  t 2 (1)  t   (4  t 2 ) 2 (2t )
1 1

2
 12 t2 4  t2  t2 4  2t 2
 4  t 2  t 2 (4  t 2 )  4  t2   
4  t2 4  t2 4  t2
To find the critical points
4  2t 2
put f ( t )  0   0  4  2t  0  t  2  t   2
2 2

4  t2
and also f ( t ) does not exist at 4  t 2  0  t 2  4  t  2
t  2,  2, 2,  2 are the critical points.
But t  2,  2 does not belong to [-1, 2]
Applying closed interval method,
f  1  (1) 4  (1)2   3 --------- SMALLEST VALUE

f  2  (2) 4  (2)2  0

f  2  ( 2) 4  ( 2)2  2
--------- LARGEST VALUE

Absolute maximum of f  t   t 4  t 2 on [-1, 2] is at t = 2 and it is f  2  2


Absolute minimum of f  t   t 4  t 2 on [-1, 2] is at t = -1 and it is f  1   3
Problem 47: Find the local maximum and minimum values of the function f  x   x  2sin x, 0  x  2
Solution:
f   x   1  2cos x
To find the critical points
put f ( x )  0 1 + 2cos x = 0  cos x = -1/2
x = 2/3 and x = 4/3
Interval f   x   1  2cos x f  x   x  2sin x Conclusion
(Using first derivative test)
0  x  2 3 Positive Increasing on f ( x ) changes from positive to
(0, 2 3) negative at 2/3
2 3  x  4 3 Negative Decreasing on  Local maximum at x = 2/3
(2 3, 4 3)
f ( x ) changes from negative to
positive at 4/3
4 3  x  2 Positive Increasing on
 Local minimum at x = 4/3
(4 3, 2 )

 2  2 2 2 3 2
 f  x   max  f    2sin  2   3  3.83
 3  3 3 3 2 3

Page No. 21
Subject Code & Name: MA 8151 / ENGINEERING MATHEMATICS I Class Notes 2018 - 2019

 4  4 4 4  3  4
 f  x   min  f    2sin   2      3  2.46
 3  3 3 3  2  3

Problem 48: Find the local maximum and minimum values of the function f  x   x  4 x using both
the first and second derivative tests.
Solution:
f  x   x  4 x Domain of the given function is (0, ∞)

f  x 
dx
 x  x   x (1/2)1  x (1/4)1  x  (1/2)  x  (3/4)
d 1/2 1/4 1
2
1
4
1
2
1
4
1 1 1  1  1  1 
 (1/2)  (3/4)  (1/2) 1  (3/4) (1/2)   (1/2) 1  (1/4) 
2x 4x 2x  2x  2x  2x 
1  2 x(1/4)  1  2 x(1/4)  1 2 4 x  1
   
2 x(1/2)  2 x (1/4)  4 x (3/4) 4 4 x3
To find the critical points
2 4 x 1
4
1
put f ( x )  0
1 1
 0  2 4 x 1  0  4 x   x  
4 4 x3 2  2  16
and also the denominator = 0 when x = 0.
1
Therefore the critical points are x  &x 0
16
Interval 2 4 x 1 f  x  x  4 x Conclusion
f  x  (Using first derivative test)
4 4 x3
0  x  1 16 Negative Decreasing on (0,1 16) f ( x ) changes from negative
to positive at 1/16
1 16  x   Increasing on (1 16, )
Positive
 Local minimum at x = 1/16
Second Derivative test:
d d 1 1 
f ( x)   f ( x)    x  (1/2)  x  (3/4) 
dx dx  2 4 
1 1  (1/2) 1 1 3  (3/4) 1 1  (3/2) 3  (7/4)
 x  x  x  x
22 44 4 16
 (3/2)  (7/4) (3/2) (7/4)
 1  1 1  31 1  16  3  16 
f               
 16  4  16  16  16  4 1  16  1 
1 3 1 3 3
 16    16     4   2  0
(1/2) 3 (1/4) 7 7

4  16   4 16
 Local minimum at x = 1/16
1  (3/2) 3
f   0    0   0
 (7/4)
 0  x = 0 is an inflection point
4 16
1 1 4 1 1 1 1
 f  x   min  f       
 16  16 16 4 2 4

Page No. 22
Subject Code & Name: MA 8151 / ENGINEERING MATHEMATICS I Class Notes 2018 - 2019

Problem 49: Discuss the curve y  x 4  4 x 3 with respect to concavity, points of inflection, and local
maxima and minima. Use this information to sketch the curve
Solution:
Domain of the function y  x 4  4 x3 is (-∞, ∞)
y  4 x3  12 x 2
y  12 x2  24 x
To find critical points, y  0  4 x3  12 x 2  0  4 x 2 ( x  3)  0
x= 0, 3 are the critical points
To find inflection points, y  0  12 x 2  24 x  0  12 x( x  2)  0
x= 0, 2 are the inflection points
By second derivative test,
 yx0  12(0)2  24(0)  0
x = 0 is neither local maximum or local minimum as x = 0 is an inflection point.
 yx3  12(3)2  24(3)  36  0 x = 3 is local minimum
 ymin   yx3  34  4(33 )  81 108  27
Interval y  12 x 2  24 x  12 x( x  2) y  x 4  4 x3

  x  0 Positive Concave upward on (, 0)

0 x2 Negative Concave downward on (0, 2)

3 x  Positive
Concave upward on  2,  

Graph

Page No. 23
Subject Code & Name: MA 8151 / ENGINEERING MATHEMATICS I Class Notes 2018 - 2019

Problem 50: Given f  x   x 3  12 x  2 , (i) Find the intervals of increase or decrease, (ii) Find the local
maximum and minimum values, (iii) Find the intervals of concavity and the inflection points,
Solution:
Domain of the function f  x   x3  12 x  2 is (-∞, ∞)
f   x   3x2 12
f   x   6 x
To find critical points, f   x   0  3x2  12  0  x2  4  x  2
x= 2, -2 are the critical points
To find inflection points, f ( x)  0  6 x  0  x  0
x= 0 is the inflection points
Interval f   x   3x 2  12 f  x   x3  12 x  2 Conclusion
(Using first derivative test)
 3( x  2)( x  2)
  x  2 Positive Increasing on (,  2) f ( x ) changes from
2  x  2 Negative Decreasing on ( 2, 2) positive to negative at -2
 Local maximum at x = -2
f ( x ) changes from
negative to positive at 2
2 x Positive Increasing on (2, )
 Local minimum at x = 2
 f  x   max  f  2   (2)3  12(2)  2  8  24  2  18

 f  x   min  f  2   (2)3  12(2)  2  8  24  2  14


Interval f   x   6 x f  x   x3  12 x  2
  x  0 Negative Concave downward on (, 0)
0 x Positive
Concave upward on
 0,  
Problem 51: For the function f  x   2 x 3  3 x 2  12 x find the maximum and minimum of f  x  . Also
find the intervals on which f  x  is increasing and decreasing.
Solution:
Domain of the function f  x   2 x3  3x2 12x is (-∞, ∞)
f   x   6 x2  6 x 12
f   x   12x  6
To find critical points, f   x   0  6x2  6x 12  0  x2  x  2  ( x  2)( x  1)  x  1, 2
x= -1, 2 are the critical points
To find inflection points, f ( x)  0  12 x  6  0  x  12
x= ½ is the inflection point

Page No. 24
Subject Code & Name: MA 8151 / ENGINEERING MATHEMATICS I Class Notes 2018 - 2019

Interval f   x   6 x 2  6 x  12 f  x   2 x3  3x2 12x Conclusion


(Using first derivative test)
 6( x 2  x  2)
 6( x  2)( x  1)
  x  1 Positive Increasing on f ( x ) changes positive to
( ,  1) negative at -1
1  x  2 Negative Decreasing on  Local maximum at x = -1
(1, 2)
f ( x ) changes from from
negative to positive at 2
2 x Positive Increasing on
 Local minimum at x = 2
(2, )

 f  x   max  f  1  2(1)3  3(1) 2  12(1)  2  3  12  7

 f  x   min  f  2   2(2)3  3(2) 2  12(2)  16  12  24  20


x2  x  1
Problem 52: Find the maximum and minimum values of f  x   2
x  x 1
Solution:
x2  x  1
f  x 
x2  x  1

f  x 
x 2
 x  1  2 x  1   x 2  x  1  2 x  1

x  x  1
2 2


 2x 3
 2 x 2  2 x  x 2  x  1   2 x3  2 x 2  2 x  x 2  x  1

x  x  1
2 2


  x  1   x  1  2 x  2  2(1  x )
2 2 2 2

 x  x  1  x  x  1  x  x  1
2 2 2 2 2 2

 x  x  1  2  0  2 x    2 1  x  2  x  x  1  2 x  1
2 2 2 2

f   x  
 x  x  1 2 4

 x  x  1  4 x   x  x  1  4 1  x   2 x  1
2 2 2

f   x  
 x  x  1 2 4

 4 x  4 x  4 x  4  2 x  2 x  1  x  
3 2 3 2

 
 x  x  1 2 3

 4 x3  4 x 2  4 x  8 x  8 x3  4  4 x 2   4 x3  12 x  4 
 
   x2  x  1
3 3
x 2
 x  1

Page No. 25
Subject Code & Name: MA 8151 / ENGINEERING MATHEMATICS I Class Notes 2018 - 2019

2(1  x 2 )
To find the critical points f   x   0   0  2(1  x 2 )  0  x  1
x  x  1
2 2

And also f   x  is not defined when the denominator is 0 (i.e)  x 2  x  1  0  x 2  x  1  0


2

b  b2  4ac (1)  (1)2  4(1)(1) 1  3 1  i 3


By applying the formula x    
2a 2(1) 2 2
Which are complex numbers.
Therfore x = 1 are the critical points.
Applying the second derivative test,
 4(1)3  12(1)  4
f (1)   4  0 x = 1 is the local maximum point
 (1)2  (1)  1
3

 4(1)3  12(1)  4   4  12  4 12
f (1)     0 x = -1 is the local mminimum point
 (1)2  (1)  1 1  1  1
3 3
27

(1)2  (1)  1
f max  x   f (1)  3
(1)2  (1)  1
(1)2  (1)  1 1
f min  x   f (1)  
(1)2  (1)  1 3
Problem 53: Find the minimum and maximum value of f ( x)  x 2  2 x  5 in the interval [0,5].
Solution:
f ( x)  x 2  2 x  5 is continuous in [0, 5], we can use closed interval method
f ( x)  2 x  2
f ( x )  0 at x  1 .
f ( x ) exists everywhere in[0,5].
So the critical points are x  0, x  1, x  5 .
f (0)  (0)2  2(0)  5  5
f (1)  (1)2  2(1)  5  6 --------------- SMALLEST VALUE
f (5)  (5)2  2(5)  5  10 --------------- LARGEST VALUE
Hence, the minimum value of f ( x) occurs at x  1 , and the maximum value occurs at x  5 .

Problem 54: Find the absolute maximum and minimum values of the function
f ( x)  x3  3x 2  1,  12  x  4
Solution : f ( x)  x3  3x 2  1 is continuous in  12  x  4 , we can use closed interval method
f ( x)  x 3  3 x 2  1
f ( x)  3x 2  6 x
f ( x )  0  f ( x)  3x 2  6 x  0  3x( x  2)  0  x  0, 2
So the critical points are x  0, x  2
f (0)  03  3(0)2  1  1
Page No. 26
Subject Code & Name: MA 8151 / ENGINEERING MATHEMATICS I Class Notes 2018 - 2019

f (2)  23  3(2)2  1  8  12  1  3 ------------------- SMALLEST VALUE

f ( 12 )  ( 12 )3  3( 12 ) 2  1   81  3( 14 )  1  1 6 8
8  1
8

f (4)  43  3(4)2  1  64  48  1  17 ------------------ LARGEST VALUE


Hence, the minimum value of f ( x) occurs at x  2 , and the maximum value occurs at x  4 .
APPLICATIONS OF DERIVATIVES
Problem 55. Find the dimensions of a circular cylinder whose volume is 9 m3 but which uses the least
amount of material.
Solution: The total surface area, A of the cylinder is
A = top surface + side surface + bottom surface
  r 2  2 rh   r 2
 2 r 2  2 rh
The volume, V of the cylinder is V   r 2 h since V  9 m3 .

9
We can write 9   r 2 h  h 
 r2
This gives the surface area just in terms of r as
 9  18
A  2 r 2  2 r  2   2 r 2   2 r 2  18r 1
r  r
To find the minimum, take the first derivative of A with respect to r as
dA 18
 4 r  18(1) r 2  4 r  2
dr r
dA
Solving for 0,
dr
18
4 r  2  0  4 r 3  18  0
r
18
r3 
4
1
 18  3
r    1.12725m
 4 
9 9
Since h  2 ,  h  = 2.2545m
r  (1.12725)2
d2A 36
2
 4  18(2)r 3  4  3
dr r
 d 2 A 36
 2  4   44.5025
 dr  r 1.12725 1.12725
d2A
This value  0 for r  1.12725 m .
dr 2
As per the second derivative test, r  1.12725 m corresponds to a minimum.

Page No. 27
Subject Code & Name: MA 8151 / ENGINEERING MATHEMATICS I Class Notes 2018 - 2019

UNIT II - FUNCTIONS OF SEVERAL VARIABLES


PARTIAL DERIVATIVES
Let z  f ( x, y ) be a function, then
z z
(i) First order partial derivatives : ,
x y
2 z 2 z 2 z
(ii) Second order partial derivatives , :,
x 2 y 2 xy
3 z 3 z 3 z 3 z
(iii) Third order partial derivatives : 3, , ,
x y 3 x 2y xy 2
u u u
1. If u  ( x  y )( y  z )( z  x) , show that   0
x y z
Solution:
Given u  ( x  y )( y  z )( z  x) , then
u
 ( y  z )  ( x  y )(1)  ( z  x)(1)   ( y  z )( z  x)  ( y  z )( x  y )    (1)
x
u
 ( z  x)  ( x  y )(1)  ( y  z )(1)   ( x  y )( z  x)  ( y  z )( z  x)    (2)
x
u
 ( x  y )  ( y  z )(1)  ( z  x)(1)   ( x  y )( y  z )  ( x  y )( z  x)    (3)
z
u u u
Adding (1),(2) and (3) we get   0
x y z
2. If u  x y , then find (i) (i ) u xy (ii ) u xyx
Solution
Given u  x y        (1) then
(i) Differentiating (1) w.r.t ‘y’ , we get
u y  x y log x
Again differentiating w.r.t ‘x’ we get
uxy  yx y 1 log x  x y 1  x y 1 1  y log x 
(ii) Differentiating (1) w.r.t ‘ x’, we get
u x  yx y 1
Again differentiating w.r.t ‘y’ we get
u yx  yx y 1 log x  x y 1
Again differentiating w.r.t ‘x’ we get
 y
u xyx  x y 1    (1  y log x)( y  1) x y  2  yx y  2  (1  y log e x )( y  1) x y  2
x
 y x x2  y 2
3. If z  x 2 tan 1    y 2 tan 1   , prove that zxy  2
x  y x  y2
Solution
 y x
Given z  x 2 tan 1    y 2 tan 1  
x  y

Page No. 28
Subject Code & Name: MA 8151 / ENGINEERING MATHEMATICS I Class Notes 2018 - 2019

 y 1  y  1 1
z x  2 x tan 1    x 2  2  y
2
 
x  y2  x   x2  y
1  2  1   2 
x  y 

Differentiating w.r.t ‘x’ we get


 y  x y
2
y3
 2 x tan 1    2 
x x  y x2  y 2
2

 y
 2 x tan 1    y
x
Again differentiating w.r.t ‘y’ we get
1 1 2 x2 x2  y 2
z yx  z xy  2 x    1   1 
 y2   x  x2  y 2 x2  y 2
1  2 
x 
4. If u  log( x3  y 3  z 3  3xyz ) , then prove that
2
u u u 3     9
(i)    (ii)      
x y z x  y  z  x y z   x  y  z
2

Solution
Given u  log( x3  y 3  z 3  3xyz )
u 3( x 2  zy) u 3( y 2  zx) u 3( z 2  xy)
Then  3 ;  3 ; 
x x  y 3  z 3  3xyz y x  y 3  z 3  3xyz z x3  y 3  z 3  3xyz

u u u 3( x 2  y 2  z 2  yz  zx  xy )
  
x y z x3  y 3  z 3  3xyz
3( x 2  y 2  z 2  yz  zx  xy ) 3
 
( x  y  z )( x  y  z  yz  zx  xy ) ( x  y  z )
2 2 2

     3
   u               (1)
 x y z  x yz
    
(ii) Operating     on both sides of (1), we get
 x y z 
2
          3 
    u      
 x y z   x y z  x  y  z 
  3    3   3 
      
x  x  y  z  y  x  y  z  z  x  y  z 
3 3 3
  
 x  y  z x  y  z x  y  z
2 2 2

9

 x  y  z
2

Page No. 29
Subject Code & Name: MA 8151 / ENGINEERING MATHEMATICS I Class Notes 2018 - 2019

5. If x = rcos θ, y = rsin θ, prove that


 2 r  2 r 1  r   r  
2 2
 2θ  2θ
(i)         (ii)   0 (x  0, y  0)
x 2 y 2 r  x   y   x 2 y 2
Solution:
x = rcos θ, y = rsin θ.
 x2 + y2 = r2 and tan θ = y/x
Differentiating r2 = x2 + y2 partially w.r.t x, we get
r r x
2r.  2x i.e.,  (1)
x x r
Differentiating r2 = x2 + y2 partially w.r.t y, we get
r r y
2r.  2y i.e.,  (2)
y y r

1  r   r   1  x 2 y2 
2 2

        2  2
r  x   y   r  r r 

1 1
 . 2 (x 2  y 2 )
r r
1
 (3)
r
Differentiating (1) partially w.r.t x, we get
2r   1  r 1
 x 2   1.
x  r  x
2
r
x x 1
  2 . 
 r  r r
Similarly from (2), we get,
2r   1  r 1
 y 2   1.
y  r  y
2
r
 y y 1
  2 . 
 r  r r
 2r  2r 1 2
  2   3 (x 2  y 2 ) 
x 2
y r r
1 2 1
   (4)
r r r
 2 r  2 r 1  r   r  
2 2

From (3) and (4), we get,        


x 2 y 2 r  x   y  
Euler’s Theorem
f f
If f(x, y) is a homogenous function of degree n in x and y, then x  y  nf
x y
2 f 2 f 2  f
2
If f(x, y) is a homogenous function of degree n in x and y, then x 2  2 xy  y  n (n  1) f .
x 2 xy y 2

Page No. 30
Subject Code & Name: MA 8151 / ENGINEERING MATHEMATICS I Class Notes 2018 - 2019

u u u  x 3  y3  z3 
6. Show that x.  y.  z.  2 tan u where u  sin -1  
x y z  ax  by  cz 
 x 3  y3  z3 
Solution: We have, sin u   
 ax  by  cz 
x 3  y3  z3
Let f(x, y, z) = (1)
ax  by  cz
t 3 x 3  t 3 y3  t 3z3
f(tx, ty, tz) = = t2 f(x, y, z)
atx  bty  ctz
 f(x, y, z) is a homogeneous function of degree 2.
 By Euler’s theorem,
f f f
x.  y.  z.  2.f (2)
x y z
From (1), we have, f = sin u
f u f u f u
  cos u.  cos u. and  cos u.
x x y y z z
Substituting these in (2), we get,
u u u
x.cos u.  y.cos u.  z.cos u.  2.sin u
x y z
u u u
x.  y.  z..  2.tan u
x y z
 x3  y 3 
1
If u  tan   , prove that x uxx  2xyuxy  y u yy  2sin u cos3u
7. 2 2

 x y 
Solution:
 x3  y 3  x3  y 3
Given that u  tan 1   , implies tan u  f ( x, y )  , a homogenous function of degree
 x y  x y
f f
2. Therefore, by Euler’s theorem x  y  nf  2 f
x y
(tan u ) (tan u )
 x y  2 tan u
x y
 u u 
 sec2 u  x  y   2 tan u
 x y 
 u u  sin u
 x y 2 cos 2 u  sin 2u                  (1)
 x y  cos u
Differentiating (1) partially with respect to x and multiply with x, we get,
 2 u u  2u u
 x 2  y  2cos 2u
x x xy x
u 2
u u
2
u
 x2 2  x  xy  2 x cos 2u
x x xy x

Page No. 31
Subject Code & Name: MA 8151 / ENGINEERING MATHEMATICS I Class Notes 2018 - 2019

2u 2u u
 x2  xy   2cos 2u  1 x                    (2)
x 2
xy x
Differentiating (1) partially with respect to y and multiply with y, we get,
 2 u u  2u u
 y 2  x  2cos 2u
y y xy y
2u u  2u u
 y 2
y  xy  2 y cos 2u
y 2
y xy y
2u 2u u
 y 2
 xy   2 cos 2u  1 y              (3)
y 2
xy y
Adding (2) and (3), we get
 u u 
x 2 uxx  2 xyuxy  y 2 u yy   2cos 2u  1  x y 
 x y 
  2 cos 2u  1 sin 2u  2sin u  4 cos3 u  3cos u   2sin u cos 3u
Change of Variables
u u u
8. If u  f ( x  y, y  z, z  x), then show that   0
x y z
Solution :
Given u  f ( x  y, y  z, z  x)  f (r , s, t ) where r  x  y; s  y  z ; t  z  x
r r r s s s t t t
 1,  1,  0 ;  0,  1,  1 ;  1,  0,  1
x y z x y z x y z
u u r u s u t u u u u u
    (1)  (0)  (1)  
x r x s x t x r s t r t
u u r u s u t u u u u u
    (1)  (1)  (0)   
y r y s y t y r s t r s
u u r u s u t u u u u u
    (0)  (1)  (1)   
z r z s z t z r s t s t
u u u u u u u u u
Now         0
x y z r t r s s t
x y z u u u
9. If u  f  , ,  , then show that x  y  z 0
 y z x x y z
x y z x y z
Solution : Given u  f  , ,   f (r , s, t ) where r  ; s  ; t 
 y z x y z x
r 1 r x r s s 1 s y t z t t 1
 ,   ,  0 ;  0,  ,   ;   ,  0, 
x y y y z
2 x y z z z 2 x x y
2 z x

u u r u s u t u  1  u u  z  u  1  u  z 
       (0)    2        2 
x r x s x t x r  y  s t  x  r  y  t  x 
u u r u s u t u  x  u  1  u u  x  u  1 
      2      (0)    2    
y r y s y t y r  y  s  z  t r  y  s  z 

Page No. 32
Subject Code & Name: MA 8151 / ENGINEERING MATHEMATICS I Class Notes 2018 - 2019

u u r u s u t u u  y  u  1  u  y  u  1 
    (0)              
z r z s z t z r s  z 2  t  z  s  z 2  t  x 
u u u u  x  u  z  u  x  u  y  u  y  u  z 
Now x y z               0
x y z r  y  t  x  r  y  s  z  s  z  t  x 
 x y yz zx 2 u u u
10. If u  f  , ,  , then show that x  y2  z2 0
 xy yz xz  x y z
 x y yz zx x y yz zx
Solution : Given u  f  , ,   f (r , s, t ) where r  ;s ;t 
 xy yz xz  xy yz xz
r 1 r 1 r s s 1 s 1 t 1 t t 1
 ,   ,  0 ;  0,  ,  ;   ,  0, 
x x y2
y z
2 x y y z
2
z 2 x x y
2 z z z

u u r u s u t u  1  u u  1  u  1  u  1 
     2   (0)    2       
x r x s x t x r  x  s t  x  r  x2  t  x2 

u u r u s u t u  1  u  1  u u  1  u  1 
      2    2   (0)     
y r y s y t y r  y  s  y  t r  y 2  s  y 2 

u u r u s u t u u  1
 u  1  u  1  u  1 
    (0)      2       
z r z s z t z r s  z 2
 t  z  s  z 2  t  z 2 
u u 2 u u u u u u u
Now x 2  y2 z       0
x y z r t r s s t
2
 u   u   u  1  u 
2 2 2

11. If u = f  x, y  and x  rcos , y  rsin , prove that          2  


 x   y   r  r  θ 
Solution:
x x
x  r cos    cos θ ;  rsin θ
r θ
y y
y  r sin    sin θ  rcos θ
r θ
u u x u y u u
We have  .  .  .cos θ  .sin θ - - - - - -(1)
r x r y r x y
u u x u y u u
Also we have  .  .  .(-rsin θ)  .rcos θ
θ x θ y θ x y
1 u u u
   .sin θ  .cos θ - - - - - - - - - - - - - - (2)
r θ x y
Squaring and adding (1) and (2), we get,
2
 u  1  u   u   u 
2 2 2

 θ 
 
2

   2      cos θ  sin θ    sin θ  cos θ
2 2 2
 
 r r    x  
y
2
 u   u 
2

     
 x   y 
2
 u   u   u  1  u 
2 2 2

          2  
 x   y   r  r  θ 
Page No. 33
Subject Code & Name: MA 8151 / ENGINEERING MATHEMATICS I Class Notes 2018 - 2019

12. If z = f  x, y  and x  u 2  v2 , y  2uv, prove that 4(u 2  v 2 )  z xx  z yy   ( zuu  zvv )


Solution:
x x y y
x  u 2  v2 
 2u ,  2v and y  2uv   2v ,  2u
u v u v
z z x z y z z
   (2u )  (2v)
u x u y u x y
2 z  2 z x  2 z y z  2 z x  2 z y
 (2u )  (2u ) 2  (2v)  (2v)
u 2 x 2 u yx u x xy u y 2 v
2 z 2 z z  2 z 2 z
 4u 2  (2uv)  2  (2uv)  (4v 2 )
x 2 yx x xy y 2
2 z 2 z z 2 z
 4u 2  (4uv)  2  (4v 2 )
x 2 yx x y 2
z z x z y z z z z
   (2v)  (2u )  2 (v)  (2u)
u x v y v x y x y
2 z  2 z x  2 z y z  2 z x  2 z y
 (2v)  (2v)  2  (2u )  (2u )
v 2 x 2 v yx v x xy v y 2 v
2 z 2 z z  2 z 2 z
 4v 2  (2uv)  2  (2uv)  (4v 2 )
x 2 yx x xy y 2
2 2 z
2 z z 2
2  z
 4v  (4uv)  2  (4v )
x 2 yx x y 2
2 z 2 z 2 z 2 z z 2 z 2 z 2 z z 2 z
  4u 2  (4uv)  2  (4v 2 )  4v 2  (4uv)  2  (4v 2 )
u 2 v 2 x 2 yx x y 2 x 2 yx x y 2


2 z
x 2  4u 2  4v 2   2 z
y 2  4u2  4v2 
 2 z 2 z 

 4 u v  2
 2
 
  4 u 2  v 2 ( z xx  z yy )
 x 2 y 2 
 


( zuu  zvv )  4 u 2  v 2 ( z xx  z yy ) 
If z = f  x, y  and x  eu sin v, y  eu cos v,
13.
prove that zxx  z yy  ( x2  y 2 )( zuu  zvv )
Solution:
x x y y
x  eu sin v 
 eu sin v ,  eu cos v and y  eu cos v   eu cos v ,  eu sin v
u v u v
z z x z y z u z
   e sin v  eu cos v
u x u y u x y

Page No. 34
Subject Code & Name: MA 8151 / ENGINEERING MATHEMATICS I Class Notes 2018 - 2019

2 z  2 z x  2 z u y z
 (eu sin v)  (e sin v)  eu sin v
u 2 x 2 u yx u x
2 z u x  2 z y z u
 (e cos v)  (eu cos v)  e cos v
xy u y 2 v y
2 z 2 z  2 z 2u  z z  2 z
 e2u sin 2 u 2 (e sin v cos v)  eu  sin v  cos v   (e2u cos 2 v) ..(1)
u 2 x 2 yx  x y  y 2
z z x z y z u z
   e cos v  eu sin v
v x v y v x y

2 z  2 z x  2 z u y z
 (eu cos v)  (e cos v)  (eu sin v)
v 2 x 2 v yx v x
2 z x  2 z y z
 (eu sin v)  (eu sin v)  (eu cos v)
xy v y v y
2

2 z 2 z  2 z 2u  z z  2 z
 e2u cos 2 u 2 (e sin v cos v)  eu   sin v  cos v   (e2u sin 2 v) ..(2) Adding
v 2 x 2 yx  x y  y 2
(i) and (ii)
2 z 2 z   2 z 2u  2 z 2u  2u
   e  e   e ( z xx  z yy )
u 2 v 2  x 2 y 2 

( zuu  zvv )  e2u ( z xx  z yy )

( zuu  zvv )  ( x 2  y 2 )( z xx  z yy )
JACOBIAN
If u  u ( x, y ) and v  v( x, y ) are two functions of two independent variable x and y. Then the Jacobian of
u u
 u, v  (u, v)  (u, v) x y
u & v is denoted by J   or and is defined by 
 x, y   ( x, y )  ( x, y ) v v
x y

Note: if u, v and w are functions of three independent variables of x, y and z. Then their Jacobian is
u u u
x y z
 u , v, w   (u , v, w) v v v
J  
 x, y, z   ( x, y, z ) x y z
w w w
x y z
(u, v) ( x, y )
Property 1. If u and v are functions of x and y, then X 1
( x, y ) (u, v)
Property 2. (Chain Rule or Jacobian of Composite Function)
If u and v are functions of r and s, where r and s are functions of x and y, then
Page No. 35
Subject Code & Name: MA 8151 / ENGINEERING MATHEMATICS I Class Notes 2018 - 2019

(u, v) (u, v) (r , s)


 X
( x, y ) (r , s) (u, v)
 (u, v, w)
Property 3. If u, v, w are functionally dependent of a function x, y and z, then  0.
 ( x, y , z )
 ( x, y )
14. If x  r cos  , y  r sin  , then find
 (r , )
Solution:
Given x  r cos  , y  r sin 
x x y y
Then  cos  ,   r sin  ,  sin  ,  r cos 
r  r 
x x
 ( x, y ) r  cos  r sin 
Now    r  sin 2   cos 2   r (1)  r
 (r , ) y v sin  r cos 
r 
u ( x, y)
15. If x  uv, y  , find
v  ( u, v )
u
Solution: Given x  uv , y 
v
x x y 1 y u
Then  v , u ,  ,  2
u v u v v v
x x
v u
 ( x, y ) u v 2u
  1 u  
 (u, v) y y v
v v2
u v
r
16. If x  r cos  and y  r sin  , then find .
x
Solution: Given x  r cos  , y  r sin  , then r 2  x 2  y 2  r  x 2  y 2
r 1 x x
Now  (2 x)  
x 2 x  y2
2
x y
2 2 r
 ( x, y , z )
17. If x  r cos  , y  r sin  , z  z ,then find
 (r , , z )
Solution: Given x  r cos  , y  r sin 
x x x y y y z z z
Then  cos  ,   r sin  ,  0,  sin  ,  r cos  ,  0,  0,  0,  1
r  z r  z r  z
x x x
r  z cos  r sin  0
 ( x, y, z ) y y y
Now   sin  r cos  0  cos  (r cos  )  r sin  (sin  )  r
 (r ,  , z ) r  z
0 0 1
z z z
r  z

Page No. 36
Subject Code & Name: MA 8151 / ENGINEERING MATHEMATICS I Class Notes 2018 - 2019

 ( x, y , z )
18. If x  r sin  cos  , y  r sin  sin  , z  r cos  , then find
 (r , , z )
Solution : Given x  r sin  cos  , y  r sin  sin  , z  r cos 
x x x
 sin  cos  ,  r cos  cos  ,  r sin  sin  ,
r  z
y y y
 sin  sin  ,  r cos  sin  ,  r sin  cos  ,
r  z
z z z
 cos  ,  r sin  ,  0
r  z
sin  cos  r cos  cos  r sin  sin 
 ( x, y , z )
 sin  sin  r cos  sin  r sin  cos 
 (r ,  , z )
cos  r sin  0
 sin  cos  (0  r 2 sin 2  cos  )  r cos  cos  (0  (r sin  cos  ) cos  )
 r sin  sin  (r sin 2  sin   r cos 2  sin  )
 r 2 sin 3  cos 2   r 2 sin  cos 2  cos 2   r 2 sin 3  sin 2   r 2 sin  cos 2  sin 2 
 r 2 sin 3  (cos 2   sin 2  )  r 2 sin  cos 2  (cos 2   sin 2  )
 r 2 sin  (sin 2   cos 2  )  r 2 sin 
( x, y, z)
19. If u  x  y  z, uv  y  z, uvw  z, show that  u2v
(u, v, w )
Solution:
Given
u  x  y  z  (1)
uv  y  z  (2)
uvw  z  (3)
Using (2) in (1), we get, x  u  y  z   u  uv  u(1  v)
Using (3) in (2) we get, y  uv  z= uv  uvw  uv(1  w)
x x x
From (4)  1 - v,  - u, 0
u v w
y y y
From (5)  v.(1 - w),  u.(1 - w),  - uv
u v w
z z z
From (3)  vw,  uw,  uv
u v w
1- v -u 0
( x, y, z)
 v(1 - w) u(1 - w)  uv
(u, v, w )
vw wu uv
 1  v  u 2v 1  w   u 2vw  u uv 2 1  w   uv 2 w 
 1  v  u 2v  u 2v2 =u 2 v
x2 x3 xx xx
20. Find the Jacobian of y1 , y 2 , y 3 with respect to x1 , x2 , x3 if y1  , y2  1 3 , y3  1 2
x1 x2 x3

Page No. 37
Subject Code & Name: MA 8151 / ENGINEERING MATHEMATICS I Class Notes 2018 - 2019

Solution:
x2 x3 xx xx
Given y1  , y2  1 3 , y3  1 2
x1 x2 x3
y1 x x y x y x y x y x x y x
  2 23 , 1  3 , 1  2 ; 2  3 , 2   1 23 , 2  1 and
x1 x1 x2 x1 x3 x1 x1 x2 x2 x2 x3 x2
y2 x2 y2 x1 y2 xx
 ,  ,   1 22
x1 x3 x2 x3 x3 x3
y1 y1 y1 xx x3 x2
 2 23
x1 x2 x3 x1 x1 x1
 ( y1 , y2 , y3 ) y2 y2 y2 x3 x1 x3 x1
   2
 ( x1 , x2 , x3 ) x1 x2 x3 x2 x2 x2
y3 y3 y3 x2 x1 x1 x2
 2
x1 x2 x3 x3 x3 x3
1 1 1
Taking from Row 1 , from Row 2 and from Row 3,we get
x1 x2 x3
x2 x3
 x3 x2
x1
1 x1 x3
 x3  x1
x1 x2 x3 x2
x1 x2
x2 x1 
x3

1 x 2 x 3 x1x 3 x1x 2
 x 2 x3 x1x 3 x1x 2 (multiply R1 by x1 , R 2 by x 2 and R 3 by x 3 )
x12 x 2 2 x 32 x x x1x 3 x1x 2
2 3

x12 x2 2 x32 1 1 1
 1 1 1  1(1  1)  1(1  1)  1(1  1)  4
x12 x2 2 x32 1 1 1
( x, y) (r , )
21. If x  r cos  , y  r sin  verify that X 1
(r , ) ( x, y)
Given x  r cos  , y  r sin 
x x y y
Then  cos  ,   r sin  ,  sin  ,  r cos 
r  r 
x x
 ( x, y ) r  cos  r sin 
Now    r  sin 2   cos 2   r (1)  r
 (r , ) y v sin  r cos 
r 
Now expressing r and  in terms of x and y
y  y
r 2  x 2  y 2  r  x 2  y 2 and tan      tan 1  
x x
r 1 x x r 1 y y
 (2 x)   ;  (2 y )  
x 2 x2  y 2 x2  y 2 r y 2 x2  y 2 x2  y 2 r

Page No. 38
Subject Code & Name: MA 8151 / ENGINEERING MATHEMATICS I Class Notes 2018 - 2019

 1  y  y y  1 1 x x
 2  2 
 2  ;  2    2  2
x y x  x  y2 x x y
1 2  1 2  
r2 y x 2
r
x x
r r x y
 (r , ) x y r  1 ( x2  y 2 )  1
  r
 ( x, y )   y x r3 r
x y r2 r 2

( x, y ) (r ,  ) 1
 X  r X 1
(r , ) ( x, y) r
( x, y ) (r ,  )
22. If x  er sec , y  er tan  verify that 1
(r , ) ( x, y )
Given x  er sec , y  er tan  x  r cos  , y  r sin 
x x y y
Then  er sec  ,  er sec  tan  ,  er tan  ,  er sec 2 
r  r 
x x
 ( x, y ) r  er sec  er sec  tan 
  r  e2 r sec   sec2   tan 2 
Now (r ,  ) y v e tan  e sec 
r 2

r 
 e2 r sec  (1)  e2 r sec 
Now expressing r and  in terms of x and y
x 2  y 2  e2 r (sec2   tan 2  )  e2 r
1
 r  log( x 2  y 2 )
2
y tan   y
Also   sin     sin 1  
x sec x
r 1 x r 1 y
 (2 x)  2 ;  (2 y)  2
x 2( x  y )
2 2
( x  y ) y
2
2( x  y )
2 2
(x  y2 )
 1  y  y  1 1 1
    ;    
x y 2  x  x x2  y 2
2
x y2  x  x2  y 2
1 2 1 2
x x
r r x y
 (r ,  ) x y (x  y2 )
2
(x  y2 )
2

 
 ( x, y )   y 1
x y x x2  y 2 x2  y 2
x y2
 2 
( x  y 2 )3/2 x( x 2  y 2 )3/2
x2  y 2 1 1 1
    2r
x( x  y ) x x 2  y 2 er sec  e2 r e sec 
2 2 3/2

 ( x, y )  ( r ,  ) 1
  e 2 r sec  2 r 1
 ( r ,  )  ( x, y ) e sec 
Page No. 39
Subject Code & Name: MA 8151 / ENGINEERING MATHEMATICS I Class Notes 2018 - 2019

(u, v)
23. If u  2 xy, v  x 2  y 2 , x  r cos  , y  r sin  , compute
 (r ,  )
Solution : Given u  2 xy, v  x 2  y 2 ,
u u
 (u, u ) x y 2 y 2 x
   4 y 2  4 x 2  4( x 2  y 2 )
 ( x, y ) v v 2 x 2 y
x y
x x
 ( x, y ) r  cos  r sin 
   r  sin 2   cos 2   r (1)  r
 (r , ) y v sin  r cos 
r 
(u, v) (u, v) ( x, y)
  X  4r ( x 2  y 2 )  4r 3 (sin ce x 2  y 2  r 2 )
(r , ) ( x, y) (r ,  )
x y xy
24. Prove that the function u  ,v  are functionally dependent
x y ( x  y)2
(u, u )
Solution: If u and v are functionally dependent, then their 0.
( x, y)
x y xy
Given u  ,v 
x y ( x  y)2
u ( x  y )  ( x  y ) 2 y u ( x  y )  ( x  y )(1) 2x
Then   ;  
x ( x  y) 2
( x  y ) y
2
( x  y) 2
( x  y )2
v ( x  y)2  2 xy( x  y)  y( x  y) v ( x  y)2  2 xy( x  y)(1) x( x  y)
  ;  
x ( x  y)4 ( x  y)3 x ( x  y)4 ( x  y )3
u u 2 y 2x
 (u, u ) x y ( x  y)2 ( x  y)2 2 xy ( x  y ) 2 xy ( x  y )
    0
 ( x, y ) v v  y( x  y) x( x  y ) ( x  y )5 ( x  y )5
x y ( x  y )3 ( x  y )3
Therefore u and v are functionally dependent.
25. If u  xy  yz  zx, v  x 2  y 2  z 2 , w  x  y  z , determine the functional relationship between
u, v, w.
Solution:
u u u
u  xy  yz  zx   y  z,  x  z,  x y
x y z
v v v
v  x2  y 2  z 2   2 x,  2 y,  2z
x y z
w w w
w x yz,   1,  1, 1
x y z

Page No. 40
Subject Code & Name: MA 8151 / ENGINEERING MATHEMATICS I Class Notes 2018 - 2019

yz xz x y
 (u, v, w )
 2x 2y 2z
Hence,  ( x, y, z )
1 1 1
 2( y  z )( y  z )  2( x  z )2( x  z )  2( y  x)( y  x)  0
Therefore u , v and w are functionally dependent.
The relation is w 2  (x  y  z) 2 = x 2 + y 2 + z 2 + 2(xy + yz + zx)  v  2u.
26. If u  sin 1 x  sin 1 y, v  x 1  y 2  y 1  x 2 , determine the functional relationship between u,
v.
Solution : Given u  sin 1 x  sin 1 y, v  x 1  y 2  y 1  x 2
u 1 u 1
 ;  ;
x 1 x 2 y 1 y2
v  xy v  xy
 1 y2  ;  1  x2 
x 1  x 2 x 1  x2
u u 1 1
 (u, u ) x y 1 x 2
1  y2
 
 ( x, y ) v v  xy  xy
1  y2  1  x2 
x y 1  x2 1  y2
   
  xy   xy 
 1   1   0


1  y2 

1  x2  
 
1  y2  1  x2    

Therefore u, v are functionally dependent.
Take x  sin  , y  sin     sin 1( x),   sin 1( y)
Now u  sin 1 x  sin 1 y     v  x 1  y 2  y 1  x 2  sin  1  sin 2   sin  1  sin 2 
 sin  cos   sin  cos   sin(   )  sin u

TAYLOR’S SERIES

TAYLOR’S SERIES FORMULA


f ( x, y )  f ( a , b )  ( x  a ) f x ( a , b )  ( y  b ) f y ( a , b )
1
 ( x  a)2 f xx (a, b)  2( x  a)( y  b) f xy (a, b)  ( y  b) 2 f yy (a, b) 
2!
1
 ( x  a)3 f xxx (a, b)  3( x  a) 2 ( y  b) f xxy (a, b)  3( x  a)( y  b) 2 f xyy (a, b)  ( y  b)3 f yyy (a, b)   ...
3!
When a=0 and b=0, the above series is called McLaurin’s series
1
f ( x, y )  f (0, 0)  xf x (0, 0)  yf y (0, 0)   x 2 f xx (0, 0)  2 xy f xy (0, 0)  y 2 f yy (0, 0)  
2!
1 3
  x f xxx (0, 0)  3x 2 y f xxy (0, 0)  3xy 2 f xyy (0, 0)  y 3 f yyy (0, 0) 
3!

Page No. 41
Subject Code & Name: MA 8151 / ENGINEERING MATHEMATICS I Class Notes 2018 - 2019

27. Expand e x sin y as McLaurin’s series


Solution:
Given f ( x, y)  e x sin y and here a= b=0. We use Mclaurin’s formula
1 2
f ( x, y )  f (0, 0)  xf x (0, 0)  yf y (0, 0)   x f xx (0, 0)  2 xy f xy (0, 0)  y 2 f yy (0, 0)  
2! 
1 3
  x f xxx (0, 0)  3x 2 y f xxy (0, 0)  3xy 2 f xyy (0, 0)  y 3 f yyy (0, 0) 
3!
f ( x, y)  e x sin y f (0,0)  e0 sin 0  0
f x ( x, y )  e x sin y f x (0, 0)  e0 sin 0  0
f xx ( x, y )  e x sin y f xx (0, 0)  e0 sin 0  0
f xxx ( x, y )  e x sin y f xxx (0, 0)  e0 sin 0  0
f y ( x, y)  e x cos y f y (0,0)  e0 cos 0  1
f yy ( x, y)  e x sin y f yy (0,0)  e0 sin 0  0

f yyy ( x, y)  e x cos y f yyy (0,0)  e0 cos 0  1


f xy ( x, y)  e x cos y f xy (0,0)  e0 cos 0  1
f xxy ( x, y)  e x cos y f xxy (0,0)  e0 cos 0  1
f xyy ( x, y)  e x sin y f xyy (0,0)  e0 sin 0  0
1 2 1
e x sin y  0  x(0)  y (1)   x (0)  2 xy (1)  y 2 (0)    x3 (0)  3 x 2 y (1)  3 xy 2 (0)  y 3 (1) 
2! 3!
x2 y y3
 y  xy  
2 6
28. Expand e xy in powers of x and y up to third degree
Solution:
Given f ( x, y)  e xy and here a= b=0. We use Mclaurin’s formula
1 2
f ( x, y )  f (0, 0)  xf x (0, 0)  yf y (0, 0)   x f xx (0, 0)  2 xy f xy (0, 0)  y 2 f yy (0, 0)  
2!
1 3
  x f xxx (0, 0)  3x 2 y f xxy (0, 0)  3xy 2 f xyy (0, 0)  y 3 f yyy (0, 0) 
3! 

f ( x, y)  e xy f (0,0)  e0  1

f x ( x, y )  ye xy f x (0,0)  0

f xx ( x, y )  y 2e xy f xx (0,0)  0

f xxx ( x, y )  y 3e xy f xxx (0,0)  0

f y ( x, y)  xe xy f y (0, 0)  0

Page No. 42
Subject Code & Name: MA 8151 / ENGINEERING MATHEMATICS I Class Notes 2018 - 2019

f yy ( x, y)  x2e xy f yy (0, 0)  0

f yyy ( x, y)  x3e xy f yyy (0, 0)  0

f xy ( x, y)  e xy  x2e xy f xy (0, 0)  1  0  1

f xxy ( x, y)  e xy y  2x e xy  x2 y e xy f xxy (0, 0)  0

f xyy ( x, y)  e xy 2 x  x2e xy y f xyy (0, 0)  0


1 2 1
e xy  1  x(0)  y (0)   x (0)  2 xy (1)  y 2 (0)    x 3 (0)  3 x 2 y (0)  3 xy 2 (0)  y 3 (0) 
2! 3!
1  xy  ....
29. Expand e x log(1  y) in powers of x and y up to third degree
Given f ( x, y)  e xy and here a= b=0. We use Mclaurin’s formula
1 2
f ( x, y )  f (0, 0)  xf x (0, 0)  yf y (0, 0)   x f xx (0, 0)  2 xy f xy (0, 0)  y 2 f yy (0, 0)  
2!
1 3
  x f xxx (0, 0)  3x 2 y f xxy (0, 0)  3xy 2 f xyy (0, 0)  y 3 f yyy (0, 0) 
3!
f x ( x, y )  e x log(1  y ) f x (0, 0)  e0 (log1)  0
f xx ( x, y )  e x log(1  y ) f xx (0, 0)  e0 (log1)  0
f xxx ( x, y )  e x log(1  y ) f xxx (0, 0)  e0 (log1)  0
ex e0
f y ( x, y)  f y (0,0)  1
1 y 1 0
ex e0
f yy ( x, y )   f yy (0, 0)    1
1  y  1  0 
2 2

2e x 2e0
f yyy ( x, y )   f yyy (0, 0)    2
1  y  1  0 
3 3

ex e0
f xy ( x, y )  f xy (0,0)  1
1  y  1 0
ex e0
f xxy ( x, y )  f xxy (0, 0)  1
1  y  1 0
ex e0
f xyy ( x, y )   f xyy (0,0)    1
1  y  1  0 
2 2

1 2
f ( x, y )  f (0, 0)  xf x (0, 0)  yf y (0, 0)   x f xx (0, 0)  2 xy f xy (0, 0)  y 2 f yy (0, 0)  
2! 
1 3
  x f xxx (0, 0)  3x 2 y f xxy (0, 0)  3xy 2 f xyy (0, 0)  y 3 f yyy (0, 0) 
3!

 y
1
2!

xy  y 2 
1 2
3!
 
x y  xy 2  y 3  ....... 
Page No. 43
Subject Code & Name: MA 8151 / ENGINEERING MATHEMATICS I Class Notes 2018 - 2019

  
30. Expand e x cos y in powers of  x  1 and  y   up to third degree
 4 

Given f ( x, y)  e x cos y and here a  1, b  .
4
f ( x, y )  f ( a , b )  ( x  a ) f x ( a , b )  ( y  b ) f y ( a , b )
1
 ( x  a ) 2 f xx (a, b)  2( x  a )( y  b) f xy (a, b)  ( y  b) 2 f yy (a, b) 
2!
1 ( x  a ) f xxx (a, b)  3( x  a) ( y  b) f xxy (a, b) 
3 2

    ...
3!  3( x  a)( y  b) 2 f xyy (a, b)  ( y  b)3 f yyy (a, b) 
 
    e
f x ( x, y )  e x cos y f x 1,   e1 cos   
 4 4 2

    e
f xx ( x, y )  e x cos y f xx 1,   e1 cos   
 4 4 2

    e
f xxx ( x, y )  e x cos y f xxx 1,   e1 cos   
 4 4 2

    e
f y ( x, y)  e x sin y f y 1,   e1 sin    
 4 4 2

    e
f yy ( x, y)  e x cos y f yy 1,   e1 cos    
 4 4 2

    e
f yyy ( x, y)  e x sin y f yyy 1,   e1 sin   
 4 4 2
    e
f xy ( x, y)   xe x sin y f xy 1,   1. e1 sin    
 4 4 2
    e
f xxy ( x, y)   x2e x sin y f xxy 1,   1. e1 sin    
 4 4 2

    e
f xyy ( x, y)   xe x cos y f xyy 1,   1. e1 cos    
 4 4 2
       
f ( x, y )  f 1,   ( x  1) f x 1,    y   f y 1, 
 4  4  4  4
1           
2

 ( x  1) f xx 1,   2( x  1)  y   f xy 1,    y   f yy 1,  
2

2!   4  4  4  4  4  
   2    
( x  1) f xxx 1, 4   3( x  1)  y  4  f xxy  1, 4 
3

1         ...

3!   
2
   
3
  
 3( x  1)  y   f xyy 1,    y   f yyy  1,  
  4  4  4  4 

Page No. 44
Subject Code & Name: MA 8151 / ENGINEERING MATHEMATICS I Class Notes 2018 - 2019

   1     
2

1  ( x  1)   y    ( x  1)  2( x  1)  y     y   
2

e   4  2!   4  4   
f ( x, y )     ...
2 1  2    
2
 
3


 3!  ( x  1) 3
 3( x  1)  y    3( x  1)  y    y    
   4  4  4   
31. Expand x 2 y  3 y  2 in powers of  x  1 and  y  2  using Taylor’s theorem
Given f ( x, y)  x 2 y  3 y  2 and here a  1, b  2 .
f ( x, y )  f ( a , b )  ( x  a ) f x ( a , b )  ( y  b ) f y ( a , b )
1
 ( x  a ) 2 f xx (a, b)  2( x  a )( y  b) f xy (a, b)  ( y  b) 2 f yy (a, b) 
2!
1 ( x  a ) f xxx (a, b)  3( x  a) ( y  b) f xxy (a, b) 
3 2

    ...
3!  3( x  a)( y  b) 2 f xyy (a, b)  ( y  b)3 f yyy (a, b) 
 
f ( x, y)  x 2 y  3 y  2 f (1, 2)  2  6  2  10
f x ( x, y)  2 xy f x (1, 2)  4
f xx ( x, y)  2 y f xx (1, 2)  4
f xxx ( x, y )  0 f xxx (1, 2)  0
f y ( x, y)  x2  3 f y (1, 2)  4
f yy ( x, y )  0 f yy (1, 2)  0
f yyy ( x, y )  0 f yyy (1, 2)  0
f xy ( x, y )  2 x f xy (1, 2)  2
f xxy ( x, y )  2 f xxy (1, 2)  2
f xyy ( x, y )  0 f xyy (0, 0)  0
f ( x, y )  f (1, 2)  ( x  1) f x (1, 2)  ( y  2) f y (1, 2)
1
 ( x  1) 2 f xx (1, 2)  2( x  1)( y  2) f xy (1, 2)  ( y  2) 2 f yy (1, 2) 
2!
1 ( x  1) f xxx (1, 2)  3( x  1) ( y  2) f xxy (1, 2) 
3 2

    ...
3!  3( x  1)( y  2) 2 f xyy (1, 2)  ( y  2)3 f yyy (1, 2) 
 
1 1
x 2 y  3 y  2  10  4( x  1)  4( y  2)  (4)( x  1) 2  4( x  1)( y  2)   6( x  1) 2 ( y  2)   ...
2! 3!
 10  4( x  1)  4( y  2)  (2)( x  1)2  2( x  1)( y  2)   ( x  1) 2 ( y  2)   ...

32. Expand x 2 y 2  2 x 2 y +3xy2 in powers of  x  2  and  y  1 using Taylor’s theorem


Given f ( x, y)  x 2 y 2  2 x2 y +3xy2 and here a  2, b  1 .

Page No. 45
Subject Code & Name: MA 8151 / ENGINEERING MATHEMATICS I Class Notes 2018 - 2019

f ( x, y )  f ( a , b )  ( x  a ) f x ( a , b )  ( y  b ) f y ( a , b )
1
 ( x  a ) 2 f xx (a, b)  2( x  a )( y  b) f xy (a, b)  ( y  b) 2 f yy (a, b) 
2!
1 ( x  a ) f xxx (a, b)  3( x  a) ( y  b) f xxy (a, b) 
3 2

    ...
3!  3( x  a)( y  b) 2 f xyy (a, b)  ( y  b)3 f yyy (a, b) 
 
f ( x, y)  x 2 y 2  2 x2 y +3xy2 f (2,1)  4  8  6  6

f x ( x, y )  2 xy 2  4 xy  3 y 2 f x (2,1)  4  8  3  9

f xx ( x, y )  2 y 2  4 y f xx (2,1)  6
f xxx ( x, y )  0 f xxx (2,1)  0
f y ( x, y)  2 x2 y  2 x2  6 xy f y (2,1)  4

f yy ( x, y)  2 x2  6 x f yy (2,1)  4

f yyy ( x, y )  0 f yyy (2,1)  0


f xy ( x, y )  4 xy  6 y  4 x f xy (2,1)  10

f xxy ( x, y )  4 y  4 f xxy (2,1)  8


f xyy ( x, y )  4 x  6 f xyy (2,1)  2

f ( x, y )  f (2, 1)  ( x  2) f x ( 2,1)  ( y  1) f y ( 2,1)


1
 ( x  2) 2 f xx (2,1)  2( x  2)( y  1) f xy ( 2,1)  ( y  1) 2 f yy ( 2,1) 
2!
1 ( x  2) f xxx (2,1)  3( x  2) ( y  1) f xxy ( 2,1) 
3 2

    ...
3!  3( x  2)( y  1) 2 f xyy (2,1)  ( y  1)3 f yyy ( 2,1) 
 
1
f ( x, y )  6  ( x  2)(9)  ( y  1)(4)  ( x  2) 2 (6)  2( x  2)( y  1)( 10)  ( y  1) 2 ( 4) 
2!
1
 ( x  2)3 (0)  3( x  2) 2 ( y  1)(8)  3( x  2)( y  1) 2 ( 2)  ( y  1) 3 (0)   ...
3!
 6  9( x  2)  4( y  1)  3( x  2) 2  10( x  2)( y  1)  2( y  1) 2 
 ( x  2)2 ( y  1)(4)  3( x  2)( y  1)2 
 y
33. Expand tan 1   at the point (1,1) up to second degree
x
 y
Given f ( x, y )  tan 1   and here a  1, b  1 .
x
f ( x, y )  f (a, b)  ( x  a ) f x (a, b)  ( y  b) f y (a, b) 
1
  ( x  a ) 2 f xx (a, b)  2( x  a )( y  b) f xy (a, b)  ( y  b) 2 f yy (a, b) 
2!

Page No. 46
Subject Code & Name: MA 8151 / ENGINEERING MATHEMATICS I Class Notes 2018 - 2019

 y 
f ( x, y )  tan 1   f (1,1)  tan 1 (1) 
x 4
y 1
f x ( x, y)  2 f x (1,1)  
x  y2 2
2 xy 1
f xx ( x, y )  f xx (1,1) 
( x  y 2 )2
2
2
x 1
f y ( x, y)  2 f y (1,1) 
x  y2 2
2 xy 1
f yy ( x, y )   2 f yy (1,1)  
( x  y 2 )2 2
y 2  x2
f xy ( x, y)  f xy (1,1)  0
( x 2  y 2 )2
f ( x, y )  f (1,1)  ( x  1) f x (1,1)  ( y  1) f y (1,1) 
1
 ( x  1) 2 f xx (1,1)  2( x  1)( y  1) f xy (1,1)  ( y  1) 2 f yy (1,1) 
2!

4 2
1 1

 ( y  1)  ( x  1)   ( x  1)2  ( y  1)2
 
2

 1
  ( y  x) 
4 2
1 2
2

x  y 2  2( x  y ) 
Maxima and Minima of a function of two variables
f f 2 f 2 f 2 f
Notation: p  ; q  ;r  2 ;s  ;t 2
x y x xy y
Working rule:
Let f(x, y) be the given function
f f
1. Find and .
x y
f f
2. Solve  0 and  0 simultaneously. Solution of the equations are stationary point
x y
3. Find the value of r, s, t and rt-s2 at all the stationary points.
r or t rt-s2 Conclusion
r<0 (rt-s2) > 0 f(x, y) attains its maximum at that stationary point
r>0 (rt-s2) > 0 f(x, y) attains its minimum at that stationary point
- (rt-s2) < 0 Neither maximum nor minimum. The stationary
point is saddle point
- (rt-s2) = 0 Further investigation needed
34. Find the maximum and minimum value for the function f ( x, y)  x 2  y 2  6 x  12 .
Solution:
Let f ( x, y)  x 2  y 2  6 x  12

Page No. 47
Subject Code & Name: MA 8151 / ENGINEERING MATHEMATICS I Class Notes 2018 - 2019

f f 2 f 2 f 2 f
p  2 x  6;q =  2 y; r  2 = 2; s = = 0 and t  2  2.
x y x xy y
p = 0 and q = 0 implies x = –3 and y = 0.
Therefore the stationary point is (–3, 0).
At (–3, 0), r = 2 > 0 and rt – s2 = 4 > 0.
Therefore f(x, y) obtains its minimum value at (–3, 0).
The minimum value is f(–3, 0) = 3.
35. Find the maximum and minimum of the function 3( x 2  y 2 )  x3  y3
Let f ( x, y)  3( x 2  y 2 )  x3  y 3
f f
p  6 x  3x 2 ;q =  6 y  3 y 2 ;
x y
2 f 2 f 2 f
r = 6 - 6x; s = = 0 and t   6  6 y.
x 2 xy y 2
p = 0 implies x = 0 and x = 2.
and q = 0 implies y = 0 and y = 2
Therefore the stationary points are (0, 0), (0, 2), (2, 0) and (2, 2).

At stationary points r = 6 – 6x rt – s2 Conclusion Extreme value


(0, 0) 6 –36 Saddle point –
(0, 2) 6 36 Minimum f(0, 2) = – 4
(2, 0) –6 36 Maximum f(2, 0) = 4
(2, 2) –6 –36 Saddle point –
Thus f(x, y) obtains its maximum at (2, 0) and the maximum value is 4. Similarly, f(x, y) obtains
its minimum at (0, 2) and the minimum value is 4.

36. Find the maximum and minimum of the function x3  y3  12 x  3 y  20


Solution: Let f ( x, y)  x3  y 3  12 x  3 y  20
f f
p  3x 2  12;q =  3 y 2  3;
x y
2 f 2 f 2 f
r = 6x; s = = 0 and t   6 y.
x 2 xy y 2
p = 0 implies x = –2 and x = 2.
and q = 0 implies y = –1 and y = 1
Therefore the stationary points are (–2, –1), (–2, 1), (2, –1) and (2, 1).

At stationary points r = 6x rt – s2 Conclusion Extreme value


(–2, –1) –12 72 Maximum f(–2,–1) = 38
(–2, 1) –12 –72 Saddle point –
(2, –1) 12 –72 Saddle point –
(2, 1) 12 72 Minimum f(2, 1) = 2
Thus f(x, y) obtains its maximum at (2, 0) and the maximum value is 4. Similarly, f(x,y) obtains its
minimum at (0, 2) and the minimum value is 4.
Page No. 48
Subject Code & Name: MA 8151 / ENGINEERING MATHEMATICS I Class Notes 2018 - 2019

37. Find the maximum and minimum values of f(x, y) = x3 + y3 – 3axy.


Solution:
Let f(x, y) = x3 + y3 – 3axy
p = fx = 3x2 – 3ay; q = fy = 3y2 - 3ax;
r = fxx = 6x; s = fxy = –3a; t = fyy = 6y.
p = 0 and q=0 implies 3x2 - 3ay = 0and 3y2 - 3ax = 0
i.e., x2 = ay and y2 = ax
i.e., x4 = a2y2
i.e., x4 = a3x
i.e., x (x3 - a3) = 0
i.e., x = 0 or x = a
When x = 0, we get, y = 0 and when x = a, we get, y = a
 The stationary points are (0,0) and (a, a)
At stationary points r rt – s2 Conclusion Extreme value
Neither maximum
2
(0, 0) 0 -9a < 0 nor minimum, Saddle -
point
If a > 0, then r > 0 and hence f(a, a) is a
minimum value.
(a, a) 6a 27a2
If a < 0, then r < 0 and hence f(a, a) is a
maximum value.
Thus the maximum or minimum value at (a, a) is f(a, a) = -a3.
38. Find the maxima or minima of f(x, y) = 2(x – y)2 – x4 – y4
Solution:
Let f(x,y) = 2(x – y)2 – x4 – y4
p = fx = 4(x – y) – 4x3; q = fy = – 4(x – y) – 4y3 ;
r = fxx = 4 – 12x2 ; s = fxy = – 4; t = fyy = 4 – 12y2
solving p = 0 and q = 0 implies x – y - x3 = 0 (1) and – (x – y) – y3 = 0 (2)
Adding (1) and (2) x3 + y3 = 0
i.e., (x + y)(x2 – xy + y2) = 0
 x = -y or x2 – xy + y2 = 0 (Check: x2 – xy + y2> 0, always)
Putting in (1) x = -y, we get,
-2y + y3 = 0
i.e., y(y2 - 2) = 0
i.e., y = 0, 2 , - 2
The corresponding x values are 0,  2 , 2
 The stationary points are (0, 0), ( 2 , - 2 ) and (- 2 , 2 )
At stationary points r=4–12x2 rt – s2 Conclusion Extreme value
Further -
(0, 0) 4 0 investigation
needed
( 2, - 2) –20 384 Maximum f ( 2, - 2) =8
(- 2, 2) –20 384 Maximum f (- 2, 2) =8

Page No. 49
Subject Code & Name: MA 8151 / ENGINEERING MATHEMATICS I Class Notes 2018 - 2019

Constrained maximum and minimum– Lagrange’s multipliers methods


Let f(x, y, z) = 0 be the function whose extreme values should be found subject to the condition (constrain)
 (x, y, z) = 0. We define F(x, y, z) = f(x, y, z) +  (x, y, z), where  is called Lagrange multiplier. For
F F F F
extreme values, solve  0;  0;  0; 0
x y z 
39.a Find the dimensions of the box that requires the least material for construction of the box
being opened at the top and having a volume 32cm.
Solution: Let x, y, z be the length, breadth and height of the box. Then surface area of the box =
xy + 2yz + 2zx, since the box is opened at the top.
Given, volume should be 32. Therefore, xyz = 32  xyz – 32 = 0
Thus F(x, y, z) = (xy +2yz + 2zx) +  (xyz – 32)  (1)
F
 y  2 z   ( yz )
x
F
 x  2 z   ( zx)
y
F
 2 y  2 x   ( xy )
z
F
 xyz  32

F F F  y  2 z  x  2 z 2 y  2 x
 0;  0; 0    
x y z yz xz xy
 y  2z  x  2z
  x y
yz xz
 x  2 z 2 y  2 x
   y  2z
xz xy
Thus x = y = 2z.
F
 0  xyz  32  0

x
 x  x   32
2
 x=4
 y = 4 and z = 2.
Thus the dimension of the box is (4, 4, 2)
39.b Find the dimension of the rectangular box without top of maximum capacity whose surface is
108 sq.cm.
Solution: Similar as 39.a. The dimension of the box is (6, 6, 3).
40.a Find the minimum distance from the point (3, 4, 15) to the cone x2 + y2 = 4z2.
Solution:
Let (x, y, z) be any point on the cone x2 + y2 = 4z2.
Then its distance from the point (3, 4, 15) is d  (x - 3) 2  ( y  4) 2  (z  15) 2 .
First we find the minimum value of d2 subject to the condition x2 + y2 = 4z2.
Page No. 50
Subject Code & Name: MA 8151 / ENGINEERING MATHEMATICS I Class Notes 2018 - 2019

Let F(x, y, z) = (x - 3) 2  ( y  4) 2  (z  15) 2 + (x2 + y2- 4z2)


The stationary points are given by,
Fx = 2(x - 3) + 2x = 0 (1)
Fy = 2(y - 4) + 2y = 0 (2)
Fz = 2(z -15) - 8z = 0 (3)
F = x2 + y2 - 4z2 = 0 (4)
3
From (1), x 
1 λ
4
From (2), y 
1 λ
15
From (3), z 
1  4λ
2 2 2
 3   4   15 
Substituting in (4),      4 
1 λ  1 λ   1  4λ 
i.e., 25(1 - 4)2 = 4.225 (1 +) 2
1  4λ
i.e.,  6
1 λ
1  4λ
From  6 we get λ  -12
1 λ
1  4λ
From  6 we get λ  -7 2
1 λ
When  = -1 2 , we get x = 6, y = 8, z = 5.
When  = -7 2 , we get x = -6/5, y = -8/5, z = 1.
Thus the stationary points are (6, 8, 5) and (-6/5, -8/5, 1).
Distance of (6, 8, 5) from (3, 4, 15) is d  (6  3) 2  (8  4) 2  (5  15) 2
 125  5 5
Distance of (-6/5, -8/5, 1) from (3, 4, 15) is d = (65  3) 2  (85  4) 2  (1  15) 2
441 784
   196
25 25
 49  196  245  7 5
 The minimum distance from the point (3, 4, 15) to the cone x2 + y2 = 4z2 is 5 5 .

40.b. Find the shortest and longest distance from (1, 2, –1) to the sphere x 2  y 2  z 2  24 using
Lagrange’s method of maxima and minima.
Solution: Similar as 40a. Shortest distance is 6 and longest distance is 3 6 .
Find the maximum and minimum distance of the point (3, 4, 12) from the sphere
40.c. x 2  y 2  z 2  1.
 3 4 12 
Solution: Similar as 40.a. The point is  x, y, z    , ,  . Minimum distance is 12 units
 12 13 13 
and maximum distance is 14 units.
Page No. 51
Subject Code & Name: MA 8151 / ENGINEERING MATHEMATICS I Class Notes 2018 - 2019

 25 
Find the length of the shortest line from the point  0, 0,  to the surface z = xy.
40.d  9 
 4 4 16 
Solution: Similar as 40a. The point is  x, y, z     ,  ,  . The minimum and the maximum
 3 3 9
41
distance is .
3
41. If u = a3 x2 + b3y2 + c3z2, where x-1 + y-1 + z-1 = 1, show that stationary value of u is given by
abc abc abc
x , y , z
a b c
Solution: Given u = a x + b y + c z and x-1 + y-1 + z-1 - 1 = 0.
3 2 3 2 3 2

1 1 1 
Let F(x, y, z) = a3x2 + b3 y2 + c3z2 + λ    1
x y z 
The stationary points are given by,
 1
Fx = 2a3 x + λ. 2  = 0 (1)
x 
 1
Fy = 2b3y + λ. 2  = 0 (2)
y 
 1
Fz = 2c3z + λ. 2  = 0 (3)
z 
1 1 1
F =   1 = 0 (4)
x y z
From (1) 2a3x3 -  = 0
1
λ 3 1
i.e., x 
2 a
Similarly from (2) and (3) we get,
1 1
λ 3 1 λ 3 1
y  , z 
2 b 2 c
substituting for x, y, z in (4) we get
1
 2 3
  (a  b  c)  1  0
λ
1
2
3
i.e., 1
  
 abc

1
3

  abc
i.e.,
2
  1 abc
1
3
 x   
2 a a
abc abc
Similarly y  and z  .
b c

Page No. 52
Subject Code & Name: MA 8151 / ENGINEERING MATHEMATICS I Class Notes 2018 - 2019

abc abc abc


Hence the stationary value of u is given by x  , y , z .
a b c

42. Find the minimum value of x2 + y2 + z2 where ax + by + cz = p.


Solution:

Method 1. Let f(x, y, z) = x2 + y2 + z2 (1)


To find the constrained minimum of f(x, y, z) subject to the condition.
ax + by + cz = p (2)
p  ax  by
i.e., z
c
 p  ax  by 
2

 2 2
f(x, y, z) = x + y +  
 c
2
fx = 2x + 2 (p – ax - by)(-a)
c
2
fy = 2y + 2 (p – ax - by)(-b)
c
2a 2
r = fxx = 2 + 2
c
2ab
s = fxy = 2
c
2b 2
t = fyy = 2 + 2
c
The stationary points are obtained by solving fx = 0 and fy = 0.
i.e., c2x – a (p – ax - by) = 0 (1)
and c2 y – b (p – ax - by) = 0 (2)
b×(1) – a×(2) gives bc2x – ac2 y = 0
i.e., c2 (bx - ay) = 0
i.e., bx = ay
a
i.e., x= y
b
ac 2 ay
Substituting in (1)y  ap  a 2  aby  0
b b
c2 a2y
yp  by  0
b b
y (a2 + b2 + c2) = bp
bp
y 2
a  b2  c2
a ap p  ax  by cp
 x y 2 and z   2
b a b c 2 2
c a  b2  c2
 ap bp cp 
The only stationary point is  2 , 2 , 2 
2 
 a  b 2
 c 2
a  b 2
 c 2
a  b 2
 c 

Page No. 53
Subject Code & Name: MA 8151 / ENGINEERING MATHEMATICS I Class Notes 2018 - 2019

 a 2  b 2  4a 2 b 2
rt - s2 =  2  2 2  2  2 2  
 c  c  c4
4b 2 4a 2
 4  2  0 at all points.
c2 c
Also r > 0 at all points.
 f(x, y, z) is minimum at the stationary point and the minimum value is
2 2 2
 ap   bp   cp 
 2 2 
 2 2 
 2 2 
a b c  a b c  a b c 
2 2 2

p2

a 2  b2  c2

Method 2.
We use Lagrange’s method. Let f(x, y, z) = x2 + y2 + z2.
φ(x, y, z) = ax + by + cz - p and F(x, y, z) = f(x, y, z) + φ(x, y, z) where  is the Lagrange
multiplier.
Then F(x, y, z) = x2 + y2 + z2 + (ax + by + cz - p)
The stationary points are obtained by solving

Fx = 2x + a = 0 (1)
Fy = 2y + b = 0 (2)
Fz = 2z + c = 0 (3)
and F = ax + by + cz - p (4)

From (1), x  
2

From (2), y  
2

From (3), z  
2
 aλ   bλ   cλ 
From (4), a.    b.    c.    p
 2  2   2
 2p
λ 2
a  b2  c2
ap bp cp
 x 2 , y 2 , z 2
a b c2 2
a b c
2 2
a  b2  c2
 ap bp cp 
The only stationary point is  2 , 2 , 2 
2 
 a  b  c a  b  c a b c 
2 2 2 2 2

2 2 2
 ap   bp   cp 
The minimum value of f ( x, y, z)   2 2 2    2 2 2    2 2 2 
a b c  a b c  a b c 

Page No. 54
Subject Code & Name: MA 8151 / ENGINEERING MATHEMATICS I Class Notes 2018 - 2019

p 2 (a 2  b 2  c 2 ) p2
 
a 2
 b2  c2 
2
a 2  b2  c2
43. Find the volume of the greatest parallelepiped which has its sides parallel to co-ordinate
x2 y 2 z 2
planes and inscribed in the ellipsoid 2  2  2  1 .
a b c
Solution: Let 2x, 2y, 2z be the dimension of the rectangular parallelepiped. We have to maximize
x2 y 2 z 2
8xyz subject to 2  2  2  1 .
a b c
 x2 y 2 z 2 
Therefore F  x, y, z   8 xyz    2  2  2  1
a b c 
F 2 x  F 2 y F 2 z
 8 yz  2 ;  8 xz  2 ;  8 xy  2 ;
x a y b z c
F F F a 2 yz b2 xz c 2 xy
 0;  0;  0    
x y z x y z
a 2 yz b2 xz x2 y 2
Choosing   2  2
x y a b
b2 xz c 2 xy y2 z2
Choosing   2  2
y z b c
x2 y 2 z 2
Thus  
a 2 b2 c 2
F  x 2 y 2 z 2 
     1
  a 2 b2 c 2 
F x2 y 2 z 2
 0  2  2  2 1
 a b c
x2 x2 x2
   1
a2 a2 a2
x2 a b c
3 2 1  x  Similarly, we can prove y  and z 
a 3 3 3
8abc
Thus the maximum volume is V = 8xyz = .
3 3
44. Find the maximum value of x m y n z p such that x + y + z = a
Solution: Given f ( x, y, z )  x m y n z p and  ( x, y, z )  x  y  z  a
F ( x, y, z)  xm y n z p    x  y  z  a 
F
 mx m 1 y n z p  
x
F
 nx m y n1 z p  
y
F
 px m y n z p 1  
z

Page No. 55
Subject Code & Name: MA 8151 / ENGINEERING MATHEMATICS I Class Notes 2018 - 2019

F F F
 0,  0,  0    mx m1 y n z p  nx m y n1 z p  px m y n z p 1
x y z
m n p
mx y z nx m y n z p px m y n z p
  
x y z
m n p mn p mn p
    (by property) 
x y z x yz a
am an ap
x ; y ; z
mn p mn p mn p
m n p
 am   an   ap 
Thus the maximum value of, F ( x, y, z )       
 mn p mn p mn p
a m n p  mm nn p p 

m  n  p
mn p

Total Derivative
u u
 If u = u (x, y), then du  dx  dy is called the total differential of u
x y
du u u dy
 If u = u (x, y) and y is a function of x,  
dx x y dx
du u dx u dy
 If u = u (x, y) and both x and y are functions of t, then  
dt x dt y dt
u u x u y
 If u = u (x, y) and both x and y are functions of v and w, then   and
v x v y v
u u x u y
 
w x w y w
45. du
Find if u = x2y and x2 + xy + y2 = 1
dx
du u u dy
We have   .
dx x y dx
dy
= 2xy + x2. (1)
dx
Let f(x, y) = x2 + xy + y2 - 1
f
 (2x  y)
  x 
dy
Then
dx f 2y  x
y
du  (2x  y)
 From (1)  2xy  x 2 .
dx 2y  x
4xy 2  2x 2 y  2x 3  x 2 y

x  2y
x(4y 2  xy  2x 2 )

x  2y

Page No. 56
Subject Code & Name: MA 8151 / ENGINEERING MATHEMATICS I Class Notes 2018 - 2019

UNIT III - INTEGRAL CALCULUS


Integration is the reverse process of differentiation. For that reason, it is also known as anti-derivative
which means that if given derivative of a function we can work backwards to find the function from which
it is derived.
d
For example: If F ( x)  f ( x) , then F(x) is the anti-derivative of f(x) and the process of finding F(x) is
dx
called integration. It is written as
 f ( x)dx  F ( x)  c , where the symbol  f ( x)dx is the integral of f(x),
f(x) is called the integrand and c is the constant of integration.
Fundamental Theorem of Calculus
Let f ( x) be a continuous function for a  x  b and F ( x) be an anti derivative of f ( x) . Then
b

 f ( x) dx   F ( x)
b

a
 F (b)  F (a)
a
Table of Derivatives and Integrals

Corresponding Derivative Formula Indefinite Integral


d
dx
[ x]  1  `1dx  x  C
d  x n 1  x n1
  x , n  1

n
 x n dx =  C , n  1
dx  n  1  n 1

 x dx  ln x  C
d 1 1
[ln x ] 
dx x

 e dx  e  C
d x
[e ]  e x
x x
dx
d kx e kx
dx
[e ]  kekx
 e kx dx 
k
C

d x ax
dx
[a ]  a x ln a
 a x dx 
ln a
C

d  2
tan x = sec2 x  sec x dx = tan x
dx
d  csc2 x dx = - cot x
cot x = - csc2 x 
dx
d 
sec x = tan x sec x  tan x sec x dx = sec x
dx
d 
csc x = - cot x csc x  cot x csc x dx = - csc x
dx
d x ax
a = ax ln(a) 
 a x
dx =
dx ln(a)
d 1
sin-1x =  1
dx = sin-1x
dx 1  x2 
 1 - x2
d 1
cos-1x = -  1
dx = - cos-1x
dx 1 x 2 
 1 - x2
d 1
tan-1x =  1 dx = tan-1x
dx 1 + x2 
 1 + x2
d -1
cot-1x =  1 dx = - cot-1x
dx 1 + x2 
 1 + x2
Page No. 57
Subject Code & Name: MA 8151 / ENGINEERING MATHEMATICS I Class Notes 2018 - 2019

d 1
sec-1x =  1
dx = sec-1x
dx x x2  1 
x x2 - 1
d 1
csc-1x = -  1
dx = - csc-1x
dx x x2  1  2
x x -1
d 
sinh x = cosh x  cosh x dx = sinh x
dx
d 
cosh x = sinh x  sinh x dx = cosh x
dx
d  2
tanh x = sech2 x  sech x dx = tanh x
dx
d  csch2 x dx = - coth x
coth x = - csch2 x 
dx
d 
sech x = - tanh x sech x  tanh x sech x dx = - sech x
dx
d 
csch x = - coth x csch x  coth x csch x dx = - csch x
dx
d 1
sinh-1x =  1
dx = sinh-1x
dx 1  x2 
 1 + x2
d 1
cosh-1x =  1
dx = cosh-1x
dx x 2  2
1
 x -1
d 1
tanh-1x =  1 dx = tanh-1x
dx 1 - x2 
 1 - x2
d 1
coth-1x =  1 dx = coth-1x
dx 1 - x2 
 1 - x2
d 1
sech-1x =  1
dx = - sech-1x
dx x 1  x2 
x 1 - x2
d 1
csch-1x =  1
dx = - csch-1x
dx x x2  1  2
x x +1
1. Determine the following integrals:


1
x  2x x
7 8
(a) dx (b) dx (c) 6
dx (d) 3x dx
Solutions:
x 7 1 x8
(a)  x dx 
7
c  c
7 1 8
2 x 81  2 x 7
 
(b) 2 x 8 dx  2 x 8 dx 
 8 1
c 
7
c

1 x 61  x 5
(c)  6 dx   x dx 
6
c  c
x  6 1 5
1 3
1 3
2 2
x x 2 3 2
 3x dx  3  x dx  3  x  3 c  3 c  x c
12
(d)
1 3 3
1
2 2

Page No. 58
Subject Code & Name: MA 8151 / ENGINEERING MATHEMATICS I Class Notes 2018 - 2019

2. Determine the following integrals:


 2x  1 
2
 2
(a)  (2 x  5 x  4 x )dx
3 2
(b)  (3x  5) dx 2
(c)   3 x   dx (d)   4 dx
 x  x 
Solutions:
(a)  (2 x  5 x 3  4 x 2 )dx

  
 2 xdx  5 x 3 dx  4 x 2 dx

 2 xdx  5 x dx  4 x dx
3 2

 x2
  x4   x3 
 2
  5   4   c
 2
  4   3 
5 4
 x2  x4  x3  c
4 3
(b)  (3x  5) dx   (9 x  30 x  25)dx
2 2

  9 x 2 dx   30 xdx   25dx
 9 x 2 dx  30 xdx  25 1dx
 x3   x2 
 9    30    25( x)  c
 3  2
 3x3  15 x 2  25 x  c

2
 2
(c)   3 x   dx
 x
 4 
   9 x 2  12  2 dx
 x 
4
  9 x 2 dx   12dx   dx
x2
 9 x 2 dx  12 1dx  4 x  2 dx

 x3   x 1 
 9   12 x  4
  
 
3   1 
4
 3x 3  12 x 
x
 2x  1 
(d)   4 dx
 x 
2x 1
  4 dx   4 dx
x x
 2 x 3 dx   x  4 dx

 x  2  x 3
 2  
 2  3
1 1
 2  3
x 3x
Page No. 59
Subject Code & Name: MA 8151 / ENGINEERING MATHEMATICS I Class Notes 2018 - 2019

3. Evaluate the following integrals:


(a)  e 2 x dx (b)  e 2 x 3 dx  e  e 3 x dx 
5x
(c)

 e   e 
2 2
 e 2 x dx  3 dx e  e 3 x dx
2x 2x x
(d) (e) (f)
Solutions:
e2x
(a)  e 2 x dx  c
2
e 2 x 3
(b)  e 2 x 3 dx 
2
c

e 5 x  e 3 x  e 5 x e 3 x
 e 
 e 3 x dx   e 5 x dx   e 3 x dx      c   c
5x
(c)
5  3  5 3

 e   
2
(d) 2x
 e2 x dx   e4 x  2  e 4 x dx   e 4 x dx   2dx   e 4 x dx
e4 x  e4 x  e4 x e4 x
  2x   c   2x  c
4  4  4 4

 e   e  6e  9dx
2
(e) 2x
 3 dx  4x 2x

  e dx   6e dx   9dx
4x 2x

e 4 x 6e 2 x
   9x  c
4 2
e4x
  3e 2 x  9 x  c
4
e4x
(f)  e x  e 3 x dx   e 4 x dx  c
4
1 7
5. Find: (a)  dx (b)  3 x dx (c)  5  x dx
x3
Solutions:
1
(a)  dx  ln x  3  c
x3
3x
(b)  3 dx 
x
c
ln 3
7 1 1
(c)  dx  7  dx  7 ln 5  x   c  7 ln 5  x  c
5 x 5 x 1

6. Solve the following integrals:


3  2x  1  e 4 x  e8x
1
2 2 5
(a)   3x  dx (b)   4 dx 
2

1
 x 1
 x 
(c)
0
2
e2x
dx (d) 1 x
dx

Solutions:
2
 2

2
(a)  3 x   dx
1  x

Page No. 60
Subject Code & Name: MA 8151 / ENGINEERING MATHEMATICS I Class Notes 2018 - 2019

2 4 
   9 x 2  12  2 dx
1
 x 
2 2 2 4
  9 x 2 dx   12dx   dx
1 1 1 x2
2 2 2
 9 x 2 dx  12 1dx  4 x  2 dx
1 1 1
2
 4
  3x 3  12( x)    (3(8)  12(2)  2)  (3  12  4)  11
 x1
3  2x  1 
(b)   4 dx
1
 x 
3 2x 3 1
  4 dx   4 dx
1 x 1 x

3 3
 2  x 3 dx   x  4 dx
1 1
3 3
 x 2  x 3
 2  
  2 1 3 1
3
 1 1   1 1  1  98
  2  3       1  
 x 3x  1  9 81   3  81
e 4 x  e8x
1

 e 
1

 dx   e 6 x dx
2 2 2x
(c)
0 e2x 0
1
 e2x e6x  2
   
 2 6  0
 1 2 1  1 6 1    1 2( 0) 1 6( 0) 
  e 2  e 2    e  e 
2 6  2 6 
 
1 1 1 1 1 1 2
 e  e3    e  e3 
2 6 2 6 2 6 3
dx  5  5 ln x 1  (5 ln 2)  (5 ln 1)  3.4657  0  3.4657
5 2 21

2
(d)
x
1 1 x
SUBSTITUTION RULE
“When the integration process is not immediately obvious, it may be possible to ‘reduce’ the
integral to a well-known form by means of substitution.”
2 
4
Example 1: Determine  2x x  1 dx by making the substitution u  x 2  1.
du
u  x2 1   2 x  du  2 xdx .
dx

 2x  x 
 1 dx   x 2  1 2 xdx  
4 4
2

.
1 1
  u 4 du  u 5  C  x 2  1  C  
5

5 5

  1 2
 
2 4 5
 2x x  1 dx  x  1  C.
5

Page No. 61
Subject Code & Name: MA 8151 / ENGINEERING MATHEMATICS I Class Notes 2018 - 2019

Example 2: Solve  sin 2x cos2xdx using a u substitution.


du
Let u = sin(2x),  2 cos2x   du  2 cos2x dx
dx
1
du  2 cos2x dx  du  cos2x dx .
2
1
 sin  2 x  cos  2 x  dx   u  2 du
1
2
 udu .

1  u2  1 1
   C   u 2  C  sin  2 x    C
2

2 2  4 4
1
 sin 2x  cos2x dx 
4
sin 2 x   C
2
.
Example 3: Evaluate  xe x dx .
2

du 1
Let u  x 2 . Then  2x  du  2 xdx or du  xdx .
dx 2
x2 u 1 1 u
Substituting,  xe dx   e  du   e du ,
2 2
1 u 1 u 1 x 2
and  e du  e  C  e  C .
2 2 2
x 2 1 x 2
Therefore,  xe dx  e  C .
2
3 x2
Example 4: Determine the value of  2 dx .
1 x 4x  7
2
Let u  x  4x  7 .
When x = 1, u = 12; when x = 3, u = 28
du 1
Then  2x  4  du  2 x  2 dx  du   x  2dx .
dx 2
Substituting and evaluating, with new limits of integration in terms of u:
x2
3 28 28
1 du 1
1 x 2  4 x  7dx  2 12 u  2 ln u 12
1 1 1
ln  28   ln 12   ln  28   ln 12  

2 2 2
1  28  1  7 
 ln    ln  
2  12  2  3 

Example 5: Evaluate 

x ln x 2  1 dx .
2
x 1

Let u  ln x 1 .
2

du 1 2x 1 x
Then  2  2x  du  2 dx or du  2 dx .
dx x 1 x 1 2 x 1

Page No. 62
Subject Code & Name: MA 8151 / ENGINEERING MATHEMATICS I Class Notes 2018 - 2019


x ln x 2  1 dx  1 1
Substituting,
x 2 1
  u  2 du  2  udu,
1 u2  1
1
   C .
2
and  udu  
  C
  ln x 2 1
2 2  2  4

x ln x 2  1 dx  1 lnx2 1 2  C .
Therefore,  x 2 1 4  
2
5x 2
Example 6: Determine the value of  3 dx .
0 ex
Letu  x 3.
du 5
Then  3x 2  du  3x 2 dx  du  5x 2 dx.
dx 3
Substituting and evaluating,
2 2
5x2 5 du 5 u 5  x3
e
0
x3
dx 
3 e u
 
3
e  C  
3
e
0

5 3  5 3 5 5
  e2    e0    e8  e0   e8  1
3  3  3 3
   
Example 7. Find  x 3 cos  x 4  2  dx
du
put u  x 4  2  du  4 x 3dx  x 3dx 
4
cos  x 4  2  dx   cos u du  sin u  c  sin  x 4  2   c
1 1 1
x
3

4 4 4
Example 8. Evaluate the following integrals.


ln x

(a) 4 x(2 x 2  3) 6 dx

(b) e x (e x  1) 3 dx (c)
x
dx

 
dx
(d) (e) 2 x( x  2) 5 dx
x ln x
Solutions:
du
(a) If u  2 x 2  3 , then  4 x  du  4 x dx
dx

 4 x( 2 x 
 3) 6 dx  u 6 du
2

u7 1

 c  (2 x 2  3)7  c
7 7
du
(b) If u  e x  1 , then  e x  du  e x dx
dx

 e (e  1) 3 dx  u 3 du 
x x

u4
 c
4
1
 (e x  1) 4  c
4

Page No. 63
Subject Code & Name: MA 8151 / ENGINEERING MATHEMATICS I Class Notes 2018 - 2019

du 1 1
(c) If u  ln x , then   du  dx
dx x x

 
ln x 1
dx  ln x  dx
x x
u2 1
  u du   c  (ln x)2  c
2 2
du 1 1
(d) If u  ln x , then   du  dx
dx x x

 
dx 1 1
  dx
x ln x ln x x


1
  du
u
 ln u  c
 ln(ln x )  c
du
(e) If u  x  2 , then  1  du  dx . Also x  u  2
dx

 2x( x  2) 
dx  2(u  2)u 5 du
5

  (2u  4)u du
5

  (2u  4u )du
6 5

2u 7 4u 6 2u 7 2u 6
  c   c
7 6 7 3
2u 6 2( x  2) 6
 (3u  7)  c  3( x  2)  7  c
21 21
tan x
Problem: Evaluate  dx
sec x  cos x
 sinx 
 
tanx  cosx  dx = sin x cos x sin x
 secx + cosx   1
dx =
  1  cos x cos x
2
dx  
1  cos 2 x
dx
 + cosx 
 cosx 

Put u=cos x, du=-sin x dx, -du=sin x dx

sin x du
 1  cos
x
dx   
1 u2
2
  tan 1 (u )  c   tan 1 (cos x)  c.
TECHNIQUES OF INTEGRATION

1. Integration by parts:
 udv  uv   vdu .
Example1: Find
 x cos xdx
we need to write this in the form u dv
If we let u = x and dv = cos x dx, then du = dx and v = sin x

 x cos xdx  x sin x   sin xdx  x sin x  cos x  C


Page No. 64
Subject Code & Name: MA 8151 / ENGINEERING MATHEMATICS I Class Notes 2018 - 2019

Example 2: Find xe x dx

If we let u = x and dv = ex dx, then du = dx and v = ex
 xe dx  xe   e dx  xe  e  C
x x x x x

Example 3: x ln xdx

If we let u =ln x and dv = x dx, then du = dx / x and v = x2/2
 x2  x2 1  x2  x
 x ln xdx  (ln x )   
 2

2 x
dx  (ln x )     dx
 2 2
 x2  x2  x2  x2
 (ln x)     C  (ln x)     C
 2  2(2)  2 4
Example 4: Evaluate  sin 1 x dx
Let u  sin 1 x dv  dx
1
du  dx vx
1  x2
Applying integration by parts,
1
 sin x dx  (sin 1 x)( x)   x
1
dx
1  x2
x
 x sin 1 x   dx  c
1  x2
2x
 x sin 1 x   dx  c
2 1  x2
1  x2
 x sin 1 x  c
2

Example 5: Find
 x cos x dx .
2

Let u  x 2 and dv  cos x dx  du  2 xdx and v 


 cos x dx  sin x .
x cos x dx   udv  uv   vdu  ( x 2 )(sin x)   (2 xdx)(sin x)
2

 x 2 sin x  2 x sin xdx

To find integral
 xsin x dx :
Applying integration by parts again
Let u  x and dv  sin x dx  du  dx and v 
 sin x dx   cos x .
 x sin x dx   udv  uv   vdu  x( cos x)   ( cos x) dx   x cos x  sin x
Finally, we get
 x cos x dx  x sin x  2   x cos x  sin x   C
2 2

 x 2 sin x  2 x cos x  2sin x  C

Page No. 65
Subject Code & Name: MA 8151 / ENGINEERING MATHEMATICS I Class Notes 2018 - 2019

Example 6: Find
 e sin x dx .
x

Let u  e x and dv  sin x dx  du  e x dx and v 


 sin xdx   cos x .
e sin x dx   udv  uv   vdu  (e x )( cos x)   ( cos x)(e x dx)
x

 e x cos x   e x cos xdx


To find  e x cos xdx :
Applying the integration by parts again.
Let u  e x and dv  cos xdx  du  e x dx and v 
 cos xdx  sin x .
e cos xdx   udv  uv   vdu  e x sin x   (sin x)(e x dx)
x

 e x sin x   e x sin xdx


Returning to the original problem,

 e sin x dx =  e cos x   e cos xdx  e cos x  e sin x   e sin x dx .


x x x x x x

Thus, 2 e x sin x dx  e x cos x  e x sin x


1 1
 e x sin x dx =  e x cos x  e x sin x  C
2 2
Example 7:  sin  
y dy .
Let t  y  t 2  y  2tdt  dy

 sin  
y dy   (sin t )(2tdt )  2 t sin t dt .
To find integral  t sin t dt :
Applying integration by parts
Let u  t and dv  sin t dt  du  dt and v   sin t dt   cos t .

 t sin t dt   udv  uv   vdu  t ( cos t )   ( cos t ) dt  t cos t  sin t


 t sin t dt = t cos t  sin t  C .
Thus, sin  x  dx = 2 t sin t dt = 2t cos t  2sin t  C  2 x cos  x   2sin  
 x C

Example 8: Evaluate  eax cos bx dx


I   eax cos bx dx u  cos bx dv  eax dx
eax
I  uv   vdu du  b sin bxdx v
a
 eax   eax 
I   cos bx        bsin bxdx 
 a   a 
 eax   b  ax
I   cos bx        e sin bxdx --------------- (1)
 a  a
Consider  eax sin bxdx
Applying integration by parts again for  eax sin bxdx with u  sin bx dv  eax dx
Page No. 66
Subject Code & Name: MA 8151 / ENGINEERING MATHEMATICS I Class Notes 2018 - 2019

eax
 e sin bxdx  uv   vdu du  b cos bxdx v
ax

a
 eax   eax 
 e ax
sin bxdx   sin bx        b cos bxdx 
 a   a 
 eax   b  ax
 e ax
sin bxdx   sin bx        e cos bxdx
 a  a
Using in (1),
 eax   b  ax  eax   b    eax   b  ax 
I   cos bx        e sin bxdx   cos bx        sin bx        e cos bxdx 
 a  a  a   a   a  a 
 e   b  e  b  e  b  e   b 
ax ax ax ax 2

I   cos bx        sin bx       I I   cos bx       sin bx       


 a   a   a  a   a  a  a   a 
2
b  cos bx b sin bx 
I    I  eax  
a  a a 2 
 a 2  b2  ax  a cos bx  bsin bx 
I e  

2
 a  a2
eax
I  2 2 a cos bx  bsin bx 
a b
Example 9: Evaluate  x tan 2 xdx.
I   x tan 2 xdx ux dv  tan 2 x
du  dx v   tan 2 xdx   (sec2 x  1)dx   (sec2 x)dx   dx  tan x  x
I  uv   vdu  x(tan 2 x  x)   (tan x  x)dx
x2
I  x tan 2 x  x 2   (tan x)dx   xdx  x tan 2 x  x 2  (  ln(cos x))  c
2
x2
 x tan 2 x  ln(cos x)  c
2
xe x
Example 10: Evaluate   x  1 2
dx

xe x ( x  1  1)e x  ( x  1)e x ex 
I  dx   dx     2
dx
 x  1  x  1   x  1  x  1 
2 2 2

ex ex
 dx   dx  I1  I 2        (1)
 x  1  x  1
2

ex dx
I2   dx u  ex dv 
 x  1  x  1
2 2

 x  1
1
1
v    x  1
2
du  e dx x
dx  
1  x  1
 1   1  x  e x   e x   e x 
I 2  uv   vdu  (e x )   
  e dx    
  dx     I1
 x 1   x 1   x 1   x 1   x 1 

Page No. 67
Subject Code & Name: MA 8151 / ENGINEERING MATHEMATICS I Class Notes 2018 - 2019

 e x   ex
From (1) , I  I  I  I   
 1I 
 x  1   x 1
1 2 1

TRIGONOMETRIC INTEGRALS

Type Expression Substitution Trig Identity

Keep one
n n
sin x or cos x, sin x or cos x
1 sin² x + cos² x = 1
n is odd or for dx
Convert remainder
n n Use half angle sin² x = ½(1 – cos 2x)
2 sin x or cos x, n is even formulas: cos² x = ½(1 + cos 2x)
From odd power,
keep one
m n
sin x • cos x, sin x or cos x,
3 sin² x + cos² x = 1
n or m is odd for dx
Use identities
to substitute
m n
sin x • cos x, Use half angle sin² x = ½(1 – cos 2x)
4
n & m are even identities cos² x = ½(1 + cos 2x)

From power pull out


2 2 2 2
n n tan x or cot x cot x = csc x - 1
5 tan x or cot x 2 2
and substitute or tan x = sec x – 1
using identities

Type I: sinn x or cosn x, n is odd


 Keep one sin x or cos x or for dx
 Convert remainder with sin² x + cos² x = 1

Problem 1: Find ∫ sin3 x dx


Solution: ∫ sin3 x dx = ∫ sin2 x sin x dx
= ∫ (1-cos 2 x) sin x dx
= ∫ (1-cos 2 x) d(-cos x) put u = cos x
= - ∫ (1-u ) d(u)
2

= - [u – (u3/3)] + c = -u +(u3 / 3) + c = -cos x +(cos3x / 3) +c

Problem 2: Find ∫ cos5 x dx


Solution:  cos5 xdx   cos 4 x cos xdx   (cos 2 x)2 d (sin x)   (1  sin 2 x) 2 d (sin x)
  (1  sin 2 x) 2 d (sin x) put u  sin x
  (1  u 2 ) 2 du   (1  2u 2  u 4 )du
2u 3 u 5 2 1
u   c  sin x  sin 3 x  sin 5 x  c
3 5 3 5
Type II: sinn x or cosn x, n is even
Use half angle formulas: sin² x = ½(1 - cos 2x) cos² x = ½(1 + cos 2x)
Page No. 68
Subject Code & Name: MA 8151 / ENGINEERING MATHEMATICS I Class Notes 2018 - 2019

 sin
4
Problem 1: Find xdx

1  cos 2 x 
2

 sin xdx   (sin x) dx    2  dx


4 2 2

1
  (1  2 cos 2 x  cos 2 2 x) dx
4
1 1  cos 4 x 1  cos 2 A
  (1  2 cos 2 x  )dx cos 2 A 
4 2 2
1 sin 2 x x  4 sin 4 x
1 x sin 4 x 
 x2   c   x  sin 2 x   c
4 2 2 4 2 8 
1  3x sin 4 x 
   sin 2 x  c
4 2 8 
Type III: sinm x • cosn x, n or m is odd
 From odd power, keep one sin x or cos x, for dx
 Use identities to substitute
Example1:Find  sin 5 x cos 2 xdx

 sin  
x cos 2 xdx   sin 4 x cos 2 x sin xdx   sin 2 x cos 2 x sin xdx
2
5

 
  1  cos 2 x cos 2 xd ( cos x)
2
put u  cos x


  1  u 2    
u 2 du    1  2u 2  u 4 u 2 du    u 2  2u 4  u 6 du 
2

 u 3 2u 5 u 7   cos3 x 2cos5 x cos7 x 


    c    c
 3 5 7   3 5 7 
Type IV: : sinm x • cosn x, n and m are even.
 Use half angle identities
Example 1:Find  sin 2 x cos 2 xdx
2
 sin 2 x  1
 sin x cos xdx   (sin x cos x) dx    2  dx  4  sin 2 xdx
2 2 2 2

1  1  cos 4 x  1 sin 4 x 
   dx   x  c
4  2  8 4 
Type V: tann x or cotn x
From power pull out tan2 x or cot2 x and substitute cot2 x = csc2 x - 1 or tan2 x = sec2 x – 1
Problem 1: Evaluate  tan3 x dx

 tan
3
x dx   tan x tan 2 x dx   tan x  sec 2 x  1 dx
   tan x sec 2 x  tan x  dx
put u  tan x  du  sec 2 x dx in the first int egral
  u du   tan x dx
u2 tan 2 x
 log  sec x   c 
  log  sec x   c
2 2
Type VI: tanm x• secn x or cotm x • cscn x , where n is even
Pull out sec2 x or csc2 x for dx
 tan x sec xdx
6 4
Problem 1: Find

Page No. 69
Subject Code & Name: MA 8151 / ENGINEERING MATHEMATICS I Class Notes 2018 - 2019

 tan
6

x sec 4 xdx 2   tan 6 x sec 2 x sec 2 xdx 

  tan x 1  tan 2 x d (tan x)
6
 put u  tan x
u7 u9 tan 7 x tan 9 x
 
  u 6 1  u 2 d (u )   u 6  u 8 du     c 
7 8 7

9
c

 tan x sec xdx


5 7
Problem 2: Find

 tan xdx   tan x sec x tan x sec xdx


5
x sec7 4 6

  (tan x) sec x tan x sec xdx   (sec


2 2 6 2
x  1) 2 sec6 xd (sec x)
  (sec x  2sec x  1) sec xd (sec x)
4 2 6
put u  sec x
u11 u9 u7
  (u 4  2u 2  1)u 6 du   (u10  2u 8  u 6 )du  2  c
11 9 7
sec11 x sec9 x sec7 x
 2  c
11 9 7

 tan
3
Problem 3: Find 2
x sec 4 xdx
Solution:
3 3 3
 tan 2
x sec4 xdx   tan 2
x sec2 x sec2 xdx   tan 2
x(tan 2 x  1)d (tan x)
put u  tan x
3
 u (u 2  1)du   (u
 32  2 3
2
 u 2 )du
 1 1  3 1
u 2 u 2
  (u
 12  23
 u )du  1  3 c
 2 1  2 1
1
2(u  1) 2(tan x  1)
1
u2 u 2 2
  1 c  2 u  c  c  c
1
2 2 u u tan x

Trigonometric Reduction Formulas

Expression Reduction Formula


1 n–1
∫ sinn x dx = - --- sinn-1 x cos x + ------- ∫ sinn-2 x dx
n n
1 n–1
∫ cosn x dx = --- cosn-1 x sin x + ------- ∫ cosn-2 x dx
n n
1
∫ tann x dx = ------- tann-1 x - ∫ tann-2 x dx
n-1
1 n-2
∫ secn x dx = ------- secn-2 x tan x + ------- ∫ secn-2 x dx
n-1 n-1

Remember the following integrals: (when n=1 in the above)

∫ tan x dx = ln |sec x| + C

∫ sec x dx = ln |sec x + tan x| + C

Page No. 70
Subject Code & Name: MA 8151 / ENGINEERING MATHEMATICS I Class Notes 2018 - 2019

1 n
 sin xdx   cos x sin n 1 x   sin n  2 xdx, n  2
n
Problem1: Prove the reduction formula
n n 1
Let us consider u  sin n1 x  du  (n  1) sin n2 x cos xdx and dv  sin xdx  v   cos x
Now
 sin xdx   sin x sin xdx   udv  uv   vdu
n n 1

  cos x sin n 1 x  (n  1)  cos 2 x sin n  2 xdx


  cos x sin n 1 x  (n  1)  (1  sin 2 x) sin n  2 xdx
  cos x sin n 1 x  (n  1)  (sin n  2 x  sin n x)dx
n  sin n xdx   cos x sin n 1 x  (n  1)  sin n 2 xdx
1 n 1
 sin xdx   cos x sin n 1 x   sin n  2 xdx
n

n n
1 n
Problem 2: Use the reduction formula  sin n xdx   cos x sin n 1 x 
n 1 
sin n  2 xdx, n  2 to prove that
n
 /2
1  3  5(2n  1) 
0 sin xdx  2  4  62n 2
2n

1 n 1
We have  sin n xdx   cos x sin n 1 x   sin n  2 xdx
n n
Put 2n for n: to get
1 2n  1
 sin xdx   2n cos x sin x  2n  sin xdx … …
2 n 1 2 n2
2n
(1)
 /2 1  /2 2n  1  / 2 2 n  2
0 2n  0
2 n 1
sin 2n
xdx   cos x sin x  sin xdx
2n 0

 /2 … …. (2)
2n  1
2n 0
2n2
 sin xdx

Now let us put 2n  2 , 2n  4 , 2n  6 , …3, 2 successively for 2n in (2): to get the following results:
 /2
 /2 2n  3
0 
2 n2
sin xdx  sin 2 n4 xdx
2n  2 0
 /2
 /2 2n  5
  sin
2 n4 2 n 6
sin xdx  xdx
0 2n  4 0
 /2
 /2 2n  3
 sin 2 n6 xdx   sin
2 n 8
xdx
0 2n  2 0


 /2  /2
3
 sin 4 xdx   sin
2
xdx
0 4 0
 /2
 /2 1  /2 1 
 sin 2 xdx 
1
 sin
0
xdx  x0  
0 2 0
2 2 2
 /2
(2n  1)( 2n  3)( 2n  5) 3  1 
 sin xdx  
2n
Finally from (2) we have the following result:
0
2n(2n  2)( 2n  4)  4  2 2

Page No. 71
Subject Code & Name: MA 8151 / ENGINEERING MATHEMATICS I Class Notes 2018 - 2019

Problem 3: Show that if I n   cos n  d , where n is a non-negative integer, then


sin  cos n 1  n  1
In   I n 2 , for n  2 . Hence evaluate I 5 and I 6 .
n n

Let us consider u  cosn1   du  (n  1) cosn2  sin  d and dv  cos d  v  sin 


Now
I n   cos n  d   cos n 1  cos  d   udv  uv   vdu
 sin  cos n 1   (n  1)  cos n  2  sin  sin  d
 sin  cos n 1   (n  1)  cos n  2  sin 2  d
 sin  cos n 1   (n  1)  cos n 2  (1  cos 2  )d

 sin  cos n 1   (n  1)   cos n  2  d   cos n  d 


 
 sin  cos n 1   (n  1)  cos n  2  d  (n  1)  cos n  d
I n  sin  cos n 1   (n  1) I n  2  (n  1) I n
I n  (n  1) I n  sin  cos n 1   (n  1) I n  2
nI n  sin  cos n 1   (n  1) I n  2
1 (n  1)
In  sin  cos n 1   I n2
n n
To find I5
I1   cos  d  sin 
1 (n  1)
put n  3 in I n  sin  cos n 1   I n2
n n
1 2 1 2
 I 3  sin  cos 2   I1  sin  cos 2    sin  
3 3 3 3
1 (n  1)
put n  5 in I n  sin  cos n 1   I n2
n n
1 4 1 4 1 2 
 I 5  sin  cos 4   I 3  sin  cos 4    sin  cos 2    sin   
5 5 5 5 3 3 
1 4 8
I 5  sin  cos 4   sin  cos 2   sin 
5 15 15
To find I6
1  cos 2
I 2   cos 2  d   d  12   sin22 
2
1 (n  1)
put n  4 in I n  sin  cos n 1   I n2
n n
 I 4  sin  cos3   I 2  sin  cos3    12   sin22  
1 3 1 3
4 4 4 4
1 3 3sin 2
I 4  sin  cos3    
4 8 16

Page No. 72
Subject Code & Name: MA 8151 / ENGINEERING MATHEMATICS I Class Notes 2018 - 2019

1 (n  1)
put n  6 in I n  sin  cos n 1   I n2
n n
1 5 1 5 1 3 3sin 2 
 I 6  sin  cos5   I 4  sin  cos5    sin  cos3    
6 6 6 6 4 8 16 
1 5 15 15sin 2
I 6  sin  cos5   sin  cos3    
6 24 48 96
   n 1 n  3 2
 ... when n is odd
2 2
 n n2 3
Problem 4: Show that  sin xdx   cos xdx  
n n

0 0  n  1 n  3 . . . 1 .  when n is even
 n n  2 2 2
1 n 1
We know that,  sin n xdx   cos x sin n 1 x   sin n  2 xdx … … (1)
n n
 /2  /2  /2  /2
1 n 1  n 1 n 1
0  
n2
sin n
xdx    n cos x sin x   sin xdx  sin n 2 xdx … …. (2)
0 n 0 n 0
Consider n is even
Now let us put n  2 , n  4 , n  6 , …4, 2 successively for n in (2)
we get the following results:
 /2  /2
n3
0 
n2
sin xdx  sin n4 xdx
n2 0
 /2  /2
n5
 sin  sin
n4 n 6
xdx  xdx
0
n4 0
 /2  /2
n 3
 sin  sin
n 6 n 8
xdx  xdx
0
n2 0
 /2  /2
3
 sin 4 xdx   sin
2
xdx
0 4 0
 /2
 /2 1  /2 1 
 sin xdx 
2 1
 sin
0
xdx  x0  
0 2 0
2 2 2
 /2
(n  1)(n  3)(n  5) 3 1 
 sin xdx  
n
Finally from (2) we have the following result:
0
n(n  2)(n  4) 4  2 2
Now let us put n  2 , n  4 , n  6 , …4, 2 successively for n in (2)
we get the following results:
 /2  /2
n3
0 sin xdx  n  2 0 sin xdx
n2 n4

 /2  /2
n5
 sin n4 xdx   sin
n 6
xdx
0
n4 0
 /2  /2
n 3
 sin  sin
n 6 n 8
xdx  xdx
0
n2 0

 /2  /2
3
 sin xdx   sin
4 2
xdx
0 4 0

Page No. 73
Subject Code & Name: MA 8151 / ENGINEERING MATHEMATICS I Class Notes 2018 - 2019

 /2
 /2 1  /2 1 
 sin 2 xdx 
1
 sin
0
xdx  x0  
0 2 0
2 2 2
 /2
(n  1)(n  3)(n  5) 3 1 
 sin xdx  
n
Finally from (2) we have the following result:
0
n(n  2)(n  4) 4  2 2
a a
By the properties of the definite integrals, 
0
f ( x)dx   f (a  x)dx
0
 /2  /2

 sin xdx   cos  sin( 2  x)  cos( x)


n n
we have xdx
0 0

2

Problem 5: Evaluate  sin x cos x dx


6 5

0

2

 sin
m
x cos n x dx
0

 m 1 m  3 m5 2  1 
 m  n m  n  2 m  n  4 . . . 3  n  n 1 when m is odd, n is even or odd
  
 m 1 m  3 m5 1  n 1 n  3 2
 ...  . . .  when m is even, n is odd
m  n m  n  2 m  n  4 2n n n2 3
 m 1 m  3 m5 1  n 1 n  3 1  
 ...  . . . .  when m is even, n is even
m  n m  n  2 m  n  4 2n n n2 2 2
here m  6, n  5
5 3 1 4 2 8
  .
11 9 7 5 3 693
TRIGONOMETRIC SUBSTITUTIONS

Expression Substitution Trig Identity

x = a sin θ
(a² - x²) 1 - sin² θ = cos² θ
-π/2 ≤ θ ≤ π/2

x = a tan θ
(a² + x²) 1 + tan² θ = sec² θ
-π/2 ≤ θ ≤ π/2

x = a sec θ
0 ≤ θ ≤ π/2
(x² - a²) sec²θ – 1 = tan² θ
or
π ≤ θ ≤ 3π/2
dx
Example 1: Evaluate x 2
x2  4
Solution:
Our goal first is to eliminate the radical. To do so, look up the table above and make the substitution
x  2 tan     tan 1  2x 

Page No. 74
Subject Code & Name: MA 8151 / ENGINEERING MATHEMATICS I Class Notes 2018 - 2019

so that dx  2sec2  d
dx 2sec 2  d
Our integral becomes x 2
x2  4

4 tan 2  4 tan 2   4
sec 2  d sec  d
 
2 tan 2  4sec 2  4 tan 2 
1
d
1 cos 2 
  cos  2  
1
d
sin  4 cos  sin 2 
4
cos 2 
1 cos  1 cos  1
  2 d   d
4 sin  4 sin  sin 
1
  cot  cos ec d
4
  cos ec  C   cos ec  tan 1  2x    C
1 1
4 4
Example 2: Evaluate .
Solution:
Again, we want to first to eliminate the radical. Consult the table above and substitute . Then
. Substituting back into the integral,

Using the integral identity from the section on Trigonometric Integrals,

and letting we obtain

Looking at the triangles above, the third triangle represents our case, with . So and
thus , which gives . Substituting,

Example 3: Evaluate .
Solution:
From the table above, let then Substituting into the integral,

But since

Page No. 75
Subject Code & Name: MA 8151 / ENGINEERING MATHEMATICS I Class Notes 2018 - 2019

Since

Looking at the triangles above, the first triangle represents our case, with So and thus
which gives Substituting,

INTEGRATION USING PARTIAL FRACTION:


Type I – Proper rational function:
2x
Example1: Evaluate  dx
( x  2)( x  3)
2x A B
  .
( x  2)( x  3) x  2 x  3
2 x  A( x  3)  B( x  2),
2 x  x( A  B)  (2 B  3 A) .
A  B  2; 2 B  3 A  0 .
Equating the coefficients, A  B  2; 2B  3 A  0 .
4 6
Solving for A and B , we obtain A  and B  so that
5 5
2x 4 6
  .
( x  2)( x  3) 5( x  2) ( x  3)
2x 4dx 6dx 4 6
 ( x  2)( x  3) dx   5( x  2)   5( x  3)  5 ln( x  2)  5 ln( x  3)  C

Example 2: Use the method of partial fractions to evaluate .


Solution:
According to the guide above (item #3), we must assign the sum of partial sums:

Multiply both sides by

Equating the coefficients of like terms from both sides,

Page No. 76
Subject Code & Name: MA 8151 / ENGINEERING MATHEMATICS I Class Notes 2018 - 2019

Thus A =1 and B = -1
Therefore the partial fraction decomposition is

Example 3:Evaluate .
We begin by factoring the denominator as
Then the partial fraction decomposition is

This equation is true for all values of


The most convenient values are the ones that make a factor equal to zero, namely, and
Substituting

Substituting ,

To find we can simply substitute any value of along with the values of and obtained.
Choose :

Now we have solved for and We use the partial fraction decomposition to integrate.

Type II -Improper rational function

Example 4: Evaluate the definite integral


This rational function is improper because its numerator has a degree that is higher than its denominator.
The first step is to divide the denominator into the numerator by long division and obtain

Now apply partial function decomposition only on the remainder,

As we did in the previous examples, multiply both sides by and then set and to
Page No. 77
Subject Code & Name: MA 8151 / ENGINEERING MATHEMATICS I Class Notes 2018 - 2019

obtain the basic equation

For

For

Thus our integral becomes

Integrating and substituting the limits,

Problem. Integrate the following by partial fractions


2x x+1 4
i) ii) iii)
(x – 1)(x + 3) 2
x + 5x + 6 2
2x – x – 1
3 x+1 2x + 1
iv) v) vi)
(x + 1)(x2 + 1) (x – 1)2(x – 2) x3 + 2x2 – x – 2
Solution

 2x 1  1 3  1
i)  dx =  dx +  dx
 (x – 1)(x + 3) 2  x–1 2  x+3

1 3 1
= ln (x – 1) + ln (x + 3) + C = ln ((x – 1)(x + 3)3) + C
2 2 2

 x+1  x+1   –1 2 
dx =  
3
ii)  dx =  + dx
2
 x + 5x + 6  (x + 2)(x + 3)   x + 2 x +

(x + 3)3
= – ln (x + 2) + 2 ln (x + 3) + C = ln   +C
 x+2 

 4  dx 4  1 1 
iii)  dx = 4 =   –
1
dx
 2x2 – x – 1  (2x + 1)(x – 1) 3 x–1
 x+ 
 2
4 1
=  ln (x – 1) – ln (x + ) + C
3 2
x – 1
= ln 
4
1
+C
3
x + 
 2

Page No. 78
Subject Code & Name: MA 8151 / ENGINEERING MATHEMATICS I Class Notes 2018 - 2019

4 x–1
= ln   +C
3 2x + 1

3 A Bx + C
iv) = +
(x + 1)(x2 + 1) x+1 x2 + 1

3 Bx + C
= + by the cover-up rule
2(x + 1) x2 + 1

3(x2 + 1) + 2(x + 1)(Bx + C)


=
2(x + 1)(x2 + 1)
So equating numerators we get, 3(x2 + 1) + 2(x + 1)(Bx + C) 3
3 3
The coefficient of x2 then gives B = – and putting x = 0 gives C =
2 2
3 3 3 (x – 1)

(x + 1)(x2 + 1) 2(x + 1) 2 x2 + 1

 3 3  dx 3 x–1
and therefore  dx =  –  dx
 (x + 1)(x2 + 1) 2  (x + 1) 2  x2 + 1

3 3  2x 3  dx
= ln (x + 1) –  dx + 
2 4  x2 + 1 2  x2 + 1

3 3 3
= ln (x + 1) – ln (x2 + 1) + tan1 x + C
2 4 4

3 (x+ 1)2 3
= ln + tan– 1 x + C
4  x2 + 1  2

v) In this case you can confirm that


x+1 3 3 2
= – –
(x – 1)2(x – 2) x – 2 x– 1 (x – 1)2

 x+1  dx  dx  dx
So  dx = 3  –3 –2
2
 (x – 1) (x – 2)  x – 2  x – 1  (x – 1)2

2 x – 2 2
= 3 ln(x – 2) – 3 ln(x – 1) + + C= 3 ln  + +C
x–1 x – 1 x – 1

 2x + 1  2x + 1 1  dx 1  dx  dx
vi)  3 dx =  dx =  +  – 
 x + 2x2 – x – 2  (x – 1)(x + 1)(x + 2) 2 x–1 2 x+1  x+2

1 1
= ln(x – 1) + ln(x + 1) – ln(x + 2) + C
2 2
Page No. 79
Subject Code & Name: MA 8151 / ENGINEERING MATHEMATICS I Class Notes 2018 - 2019

1 (x – 1)(x + 1)


2  (x + 2)2 
= ln +C

1  x2 – 1 
= ln  +C
2 (x + 2)2

Problem: Integrate the following by completing the square


1 3 2
i) ii) iii)
x2 + 2x + 5 x2 – 2x + 2 2x2 + 2x + 1
1 1
iv) v)
2
x + 6x + 10 2
2x + 12x + 27
Solution
i) By completing the square we have x2 + 2x + 5 = (x + 1)2 + 4

 1  dx 1 2
 2 dx = 
2
= 
2
dx
 x + 2x + 5  (x + 1) + 4  (x + 1)2 + 22

1 2
=  d(x + 1)
2  (x + 1)2 + 22

1 x + 1
= tan– 1   +C
2  2 

 3  3
ii)  dx =  dx = 3 tan– 1 (x – 1) + C
2
 x – 2x + 2 2
 (x – 1) + 1

 2  dx  d(2x + 1)
iii)  dx = 4 = 2 = 2 tan– 1 (2x + 1) + C
 2x2 + 2x + 1  (2x + 1)2 + 1  (2x + 1)2 + 12

 1  dx
iv)  dx =  = tan– 1 (x + 3) + C
2
 x + 6x + 10 2
 (x + 3) + 1

v) Note that 2x2 + 12x + 27 = 2(x2 + 6x) + 27 = 2((x + 3)2 – 9) + 27


= 2(x + 3)2 + 9

 dx  dx 1  3d( 2(x + 3))


 2x2 + 12x + 27 =  2(x + 3)2 + 9 = 
  3 2  ( 2(x + 3))2 + 32

1  2(x + 3)
= tan– 1   +C
3 2  3 

Page No. 80
Subject Code & Name: MA 8151 / ENGINEERING MATHEMATICS I Class Notes 2018 - 2019

x
Problem: Evaluate  3  2x  x2
dx

Solution:
3  2 x  x 2  3  ( x 2  2 x)  3  ( x 2  2 x  1  1)
 3  1  ( x 2  2 x  1)
 4  ( x  1) 2
x
I  dx
3  2x  x2
Take u  2sin  , du  2 cos  d
u u
 sin     sin 1  
2 2

I 
 2sin   1 2 cos  d   2sin   1 2 cos  d
4  4sin 2 
 2 1  sin 2 

 2sin   1 2 cos  d 

2 cos    2sin   1d
 2cos    c
u  x 1
  4  u 2  sin 1    c   3  2 x  x 2  sin 1  c
2  2 
x2 + 1 x3 3x4
Problem: Integrate i) ii) iii)
(x + 1)(x – 2) x2 + 2x + 2 (x – 1)(x + 1)
Solution
In this question we have to divide out first.
i) First note that by long division

x2 + 1 x2 – x – 2 + x + 3 x+3
= =1+
(x + 1)(x – 2) 2
x –x–2 2
x –x–2
x+3 –2 5
=1+ =1+ + by partial fractions
(x + 1)(x – 2) 3(x + 1) 3(x – 2)

 x2 + 1   –2 5 
 (x + 1)(x – 2) dx =  1 + 3(x + 1) + 3(x – 2) dx
   

5 2
=x+ ln(x – 2) – ln(x + 1) + C
3 3
1 (x – 2)5
= x + ln| |+ C
3 (x + 1)2
x3 2(x + 2)
ii) =x –2+ by long division
x2 + 2x + 2 x2 + 2x + 2

Page No. 81
Subject Code & Name: MA 8151 / ENGINEERING MATHEMATICS I Class Notes 2018 - 2019

 x3  2(x + 2)  x2  x+2
. dx =  x – 2 +  dx = – 2x + 2  dx
2
 x + 2x + 2  2
x + 2x + 2 2 2
 x + 2x + 2

x2  2x + 2 + 2
= – 2x +  dx
2  x2 + 2x + 2

x2  d(x2 + 2x + 2)  dx
= – 2x +  +2
2  x2 + 2x + 2  x2 + 2x + 2

x2
– 2x + ln(x2 + 2x + 2) + 2 tan– 1 (x + 1) + C
= by a substitution and a
2
completing the square

iii) This one is more of a test of your algebra, for which you really need your wits about you to avoid some
3x4 3(x2)2
nasty calculations! we have =
(x – 1)(x + 1) (x2 – 1)
3(x2)2 – 3x2 + 3x2 3x2
= = 3x2 +
x2 – 1 x2 – 1
3
= 3x2 + 3 +
2
x –1

 3x4   3   1 1 1 1 
So  (x – 1)(x + 1) dx =  3x2 + 3 + 2 3
 dx = x + 3x + 3  2(x – 1) – 2(x + 1) dx
   x – 1 

3 x–1
= x3 + 3x + ln| |+C
2 x+1
x2
Problem: Find  ( x  1)( x 2
 1)
dx

Solution:
x2 A Bx  C
  2 .
( x  1)( x  1) x  1 x  1
2

 x  2  A( x 2  1)  ( Bx  C )( x  1),
 x  2  x 2 ( A  B)  x( B  C )  ( A  C ).
Equating the coefficients, A  B  0; B  C  1; A  C  2 .
1 1 3
Solving A  , B   and C  , so that
2 2 2
x2 1  1 x  3  1  1  x  3 
 ( x  1)( x2  1) dx  2   x  1  x2  1 dx  2    x  1 dx   x 2  1 dx 
1  1  x 3 
    dx   2
2   x 1  x 1
dx   2 dx 
x 1 
Page No. 82
Subject Code & Name: MA 8151 / ENGINEERING MATHEMATICS I Class Notes 2018 - 2019

x2 1 1 
 ( x  1)( x 2
 1)
dx   ln( x  1)  ln( x 2  1)  3 tan 1 ( x) .
2 2 

dx 1 x 1 x
Using : x 2
a 2
 arctan  C  tan 1  C
a a a a

 2 x   x  1 x
dx dx dx
Examples: a. b. c.
2
9
2
4
2
 4x  5
dx d (2 x) 11 1  2 x   1 1  2 x 
a.   2x  2
9
 12  
 2 x   32  3
22  tan  3  

 c 
6
tan  3   c

dx d ( x  1) 1 1  x  1 
b.    tan  c
 x  1 4  x  1  22 2  2 
2 2

dx dx d ( x  2) 1  x  2
c.   2   tan 1   c  tan 1  x  2  c
x  4x  5 x  4x  4 1 ( x  2)  1 1  1 
2 2 2

dx x x
Using :  a x2
 arcsin  c  sin 1    c
a 2
a

  
dx dx dx
Examples: a. b. c.
9  x2 16  1  x   4x  x 2
2

dx x dx
a.    sin 1    c
9 x 2
3 x 3 2 2

dx d (1  x)  1 x 
b.    sin 1  c
16  1  x  4  1  x   4 
2 2 2

dx dx dx
c.   
4 x  x 2
4  4  4x  x 2
4  (4  4 x  x 2 )
d ( x  2)  x2
  sin 1  c
22  ( x  2)2  2 
Improper integrals.
b
The definite integral  a
f ( x)dx is said to be improper integral if
(i) the interval [a, b] is infinite OR (ii) f has an infinite discontinuity OR both (i) and (ii)
Type I (Infinite Integrals):
Case1: One infinite limit
 t t
a
f ( x)dx  lim  f ( x)dx
t  a
if 
a
f ( x)dx exists  t  a
OR
b b b

f ( x)dx  lim  f ( x)dx
t  t
if  t
f ( x)dx exists  t  b
Case2: Two infinite limit
 a  a 

f ( x)dx   f ( x)dx   f ( x)dx if both
 a 
f ( x)dx & a
f ( x)dx are convergent.
Note:
If the limit exists, then the integral is said to converge to the limit value.
Page No. 83
Subject Code & Name: MA 8151 / ENGINEERING MATHEMATICS I Class Notes 2018 - 2019

If the limit fails to exist, then the integral is said to diverge.


Type II (Discontinuous Integrands):
Case 1: If f x  is continuous on a, b and is discontinuous at b , then
  f xdx
b t
f x dx  lim
a t b a

f x  is continuous on a, b and is discontinuous at a , then f x dx  lim f x dx .


 
b b
Case 2: If
a t a  t

f x  is continuous on a, b except at c , where a  c  b , then f x dx  f x dx  f x dx


  
b c b
Case 3: If
a a c
, provided both integrals converge.
Comparison Theorem:
Suppose that f(x) and g(x) are continuous functions with f(x) ≥ g(x)≥0 for x ≥ a.
 
(i) If
 a  g xdx also converges.
f x dx converges, then
a
 

 
(ii) If g x dx diverges, then f x dx also diverges.
a a

Problems under Type I (case 1):



dx
1. Determine whether  is convergent or divergent.
1
x
 t
1 1
1 x dx  Lim dx  Lt  ln x 1  Lt  ln t  ln1  
t   x
t

t  t 
1
Limit does not exist, So the integral is divergent.

dx
2. Show that  2 is convergent.
1
x
 t
1
t
1
t
 x 1 
1 x 2 t   x 2 t  
2
dx  Lim dx  Lim x dx  Lim  1 
1 1
t 
 1
t
 1  1 
 Lim    Lim    1  0  1  1
t 
 x 1 t   t 
Limit exists, So the integral is convergent.

dx
3. Prove that  2 ; a  0 ; p  0 is convergent if p  1 and divergent if p  1 .
a
x
Case 1: p  1
  t
1 1 1
1 x p dx  1 x dx  Lim  dx  Lt  ln x 1  Lt  ln t  ln1  
t

t  x t  t 
1
Limit does not exist, So the integral is divergent.
Case 2: p  1  p  1 (OR) p  1
 t
1
t
1
t
 x  p 1 
a x p t   x p t  
p
dx  Lim dx  Lim x dx  Lim  
t   p  1
a a  a

1  t  p 1 a  p 1 
a x p dx  Lim 
t   p  1


 p  1

Page No. 84
Subject Code & Name: MA 8151 / ENGINEERING MATHEMATICS I Class Notes 2018 - 2019

 a  p 1
 0  if  p  1  0
 0 if m  0 
1   p 1
 Lim x m  
x 
 if m  0
a x p dx   a  p1
  if  p  1  0
  p 1
 a  p 1
 0   p  1 if  p  1  0

1   finite if 1  p  finite if p  1
a x p dx   a  p1 
inf inite if 1  p

inf inite if p  1
  if  p  1  0
  p 1
If p  1 , Limit exists, So the integral is convergent.
If p  1 , Limit does not exist, So the integral is divergent.
From case 1 and case 2,
If p  1 , Limit exists, So the integral is convergent.
If p  1 , Limit does not exist, So the integral is divergent.

1
4. Determine whether 
0
4
1 x
dx is convergent or divergent?
 t t
1 1
dx  Lt  1  x  4 dx
1


0
4
1 x
dx  Lt  4
t 
0 1 x t 
0
t
4 3  4 4
 Lt  1  x  4   Lt 1  t  4   
3


t  3
0 t  3 3
Limit does not exist, So the integral is divergent
0

 2 dr
r
5. Examine whether is convergent or divergent?
-
0
0
 2r 
0
1  2t  1 1
 2 dr  t 
Lt  2 dr  Lt    Lt   0   finite
r r
 
t 
t  ln 2
 t ln 2 t   ln 2  ln 2 ln 2
Limit exists, So the integral is convergent

 xe
 x2
6. Evaluate the improper integral dx and hence discuss their convergence.

 0 

  dx   xe x dx
 x2  x2
Let I  dx 
2
xe xe
  0
0 0

 xe dx  lim  xe  x dx
 x2 2

t 
 t
0
 lim     e x d   x 2 
2
1
2
t 
t

   12  lim  e x  
2 0

t    t 
   12  lim e0  et
t 
 2

     1 e    
1
2
 1
2  1  0   12
Limit exists, So the integral is convergent

Page No. 85
Subject Code & Name: MA 8151 / ENGINEERING MATHEMATICS I Class Notes 2018 - 2019

 t

 xe dx  lim  xe dx
x 2
x 2

t 
0 0
t
 lim     e x d   x 2 
2
1
2
t 
0

   12  lim  e x  
2 t

t    0 

 
   12  lim et  e0    12  e  1    12   0  1  12
t 
2

 
Limit exists, So the integral is convergent
 0 
I  xe x dx   xe x dx   xe x dx    12    12   0
2 2 2

  0
So the integral is convergent

1
7. Evaluate the improper integral  1 x

2
dx and hence discuss their convergence.
 0 
1 1 1
Let I   dx   dx   dx

1 x 2

1 x 2
0
1  x2
0 0
1 1
 1  x2 dx  tlim 
 1  x 2
t
dx


 lim  tan 1 x 
t  t
0


 lim  tan 1 0  tan 1 t   0  tan 1 ()  tan 1 ()  2
t 

Limit exists, So the integral is convergent
 t
1 1
0 1  x2 dx  lim 
t  1  x 2
0
dx


 lim  tan 1 x 
t  0
t


 lim  tan 1 t  tan 1 0   tan 1 ()  0  2
t 

Limit exists, So the integral is convergent
 0 
1 1 1
I dx   dx   dx  2  2  

1  x 2

1  x 2
0
1  x 2

So the integral is convergent

Infinite Discontinuities
5
dx
1. Find 
2 x2
1
Clearly, has infinite discontinuity at x = 2
x2

Page No. 86
Subject Code & Name: MA 8151 / ENGINEERING MATHEMATICS I Class Notes 2018 - 2019

5 5
dx dx

2
 lim 
x  2 t 2 t x  2
5
 lim  ( x  2) 2 dx
1
t 2
t
5
 ( x  2)  2 1 
1

 lim  
t 2
  2  1  t
1

5
 ( x  2) 2 
1

 lim  1 
t 2
 2  t
5
 lim  2 x  2   lim  2 5  2  2 t  2 
t 2 t t 2

 lim  2 3  2 t  2   2 3  2 2  2  2 3  0
t 2

The limit exists, so the integral is convergent to 23.


3
dx
2. Evaluate  , if possible.
0
x 1
1
Clearly, has infinite discontinuity at x = 1
x 1
3 1 3
1 1 1
Let I   dx   dx   dx
0
x  1 0
x  1 1
x  1

 
1 t
dx dx
0 x 1  lim  
 lim ln( x  1)0  lim  ln(t  1)  ln(0  1) 
t

t 1
0
x  1 t 1 t 1

1
dx
ln (0) and ln (-1) is not defined, hence the integral  x  1 is divergent.
0
3 1 3
1 1 1
Therefore I   dx   dx   dx is divergent. Hence the integral can not be evaluated.
0
x 1 0
x 1 1
x 1

2

3. Determine whether Let  sec xdx converges or diverges.


0

Clearly sec x has infinite discontinuity at x = /2.


 
2 t

 sec xdx  lim  sec xdx  lim  ln(sec x  tan x)0


t

t  2  t  

0 0 2

 lim  ln(sec t  tan t )  ln(sec 0  tan 0)   ln(sec  2   tan  2 )  ln(sec 0  tan 0)


t  2 

Since sec ( / 2 )= ∞ and tan 0 = ∞ , the limit does not exist.



2

Hence the integral  sec xdx is divergent


0
3
dx 4
4. What is wrong with the calculation x
1
2

3
?

1
The fundamental theorem of calculus is applied to only continuous function. Here, f ( x)  is not
x2
continuous on [-1,3].
Page No. 87
Subject Code & Name: MA 8151 / ENGINEERING MATHEMATICS I Class Notes 2018 - 2019

That is f(x) is discontinuous at x=0.


t
3
dx
0
dx
3 0
dx dx
t
dx
t
 x 21 
Let  2   2   2  2  lim  2  lim  x 2 dx  lim 
t
  lim   x 1   lim  t 1  1  
 2  1  1 t 0
x x 0 x 1 x t 0 x t 0 t 0 1 t 0
1 1 1 1

Limit does not exist so the integral is divergent.


3
dx
Hence,  2 is not a convergent integral  the integral does not exist.
1
x
Comparison theorem:

(1  e  x )dx
1. Show that the integral  is divergent by the Comparison Theorem.
1
x
1  e x 1
We know that  x  1
x x
 t
1 1
and  dx  Lim  dx  Lt  ln x 1  Lt  ln t  ln1  
t

x t  x t  t 
1 1

dx
Limit does not exist, So the integral  is divergent.
1
x

(1  e  x )dx
By the Comparison Theorem,  is divergent
1
x
Riemann sum
Definition of the Definite Integral:
Let f be a continuous function defined for a  x  b. Subdivide the interval [a,b] into n subintervals of
equal width x = (b – a)/n.
Let x0 = a, x1, x2, x3 . . . xn = b be the endpoints of these subintervals and we choose sample points x1*, x2*,
. . . xn* in these subintervals, so xi* lies in the Ith subinterval.
 n 
b
Then the definite integral of f from a to b is  f ( x)dx  lim  f ( xi* )x 
a
n 
 i 1 
n
The sum  f ( x )x
i 1
*
i is called a Riemann Sum.

Note:
1. If f(x) is positive, then the Riemann sum can be thought of as the area of approximating rectangles.
b
2. If f(x) is positive, the definite integral  f ( x)dx is equal to the exact area under the graph of f(x) between
a
a and b.
3. If f(x) is negative, then the value of the integral is the negative of the area between the curve and the x-
axis.

Problem1: Evaluate the Riemann sum for f(x) = x3 - 6x by taking the sample points as right
endpoints with a=0, b=3 and n=6.
Solution;
Given that f(x) = x3-6x ,a = 0 ,b = 3 and n = 6.
b  a 30 1
x   
n 6 2
Here the interval is subdivided into six equal parts as
[0,1/2] , [1/2,1] [1,3/2],[3/2,2],[2,5/2] ,[5/2,3]
The left end points are
Page No. 88
Subject Code & Name: MA 8151 / ENGINEERING MATHEMATICS I Class Notes 2018 - 2019

0, 1/2 , 1 ,3/2 , 2 ,5/2


The right end points are
1/2 , 1 , 3/2 , 2 , 5/2 , 3
The values of the function at these end points of the intervals are
i 0 1 2 3 4 5 6
xi = a +ix 0 1/2 1 3/2 2 5/2 3
f(x) 0 -23/8 -5 -45/8 -4 5/8 9

The Riemann sum at right end is


6
R4   f ( xi )x
i 1

 f ( x1 )x  f ( x2 )x  f ( x3 )x  f ( x4 )x  f ( x5 )x  f ( x6 )x


2
 23  1 1  45  1 1 5 1 1
      5        4       9 
 8  2 2  8  2 2 8 2 2
 23 45 5 1
    5   4   9
 8 8 8 2
63 1 63
      3.9375
8 2 16
Problem2: Find the approximate area L4 and R4 for f(x)=x 2 between x=0 and x=1.
Solution;
Given that f(x) = x2 ,a = 0 ,b = 1 and n = 4.
b  a 1 0 1
x   
n 4 4
Here the interval is subdivided into four equal parts as
[0,1/4] , [1/4,1/2] [1/2,3/4],[3/4,1]
The left end points are
0, 1/4 , 1/2 , 3/4
The right end points are
1/4 , 1/2 , 3/4 ,1
The values of the function at these end points of the intervals are
i 0 1 2 3 4
xi = a +ix 0 1/4 1/2 3/4 1
f(x) 0 1/16 1/4 9/16 1
The sum of the areas of the lower approximate rectangles is
4
L4   f ( xi 1 )x
i 1

 f ( x0 )x  f ( x1 )x  f ( x2 )x  f ( x3 )x


2 2 2
1 1 1 1 1 3 1
 (0)         
4 4 4 2 4 4 4
 1 1 9 1
 0   
 16 4 16  4
14 1 7
    0.21875
614 4 32

The sum of the areas of the upper approximate rectangles is

Page No. 89
Subject Code & Name: MA 8151 / ENGINEERING MATHEMATICS I Class Notes 2018 - 2019

4
R4   f ( xi )x
i 1

 f ( x1 )x  f ( x2 )x  f ( x3 )x  f ( x4 )x


2 2 2
1 1 1 1 3 1 1
          12
4 4 2 4 4 4 4
1 1 9 1
     1
 16 4 16  4
30 1 15
    0.46875
16 4 32

Properties of definite integrals


b b
1. f ( x)dx   f ( y)dy
a a
b a
2. f ( x)dx   f ( x)dx
a b
b b
3. f ( x)dx   f (a  b  x)dx
a a
a a
4. f ( x)dx   f (a  x)dx
0 0
b c b
5. f ( x)dx   f ( x)dx   f ( x)dx , a < c < b
a a c

 a

6.  f ( x)dx   0
2 f ( x)dxwhenf (2a  x)  f ( x)
2a

0  0 whenf (2a  x)   f ( x)

 a
7.  f ( x)dx   0
2 f ( x)dxwhenf ( x)  f ( x)
a

a  0whenf ( x)   f ( x)

a
dx
Problem1: Evaluate by substitution method ( i ) 
0 x  a2  x2
Solution:
a
dx
 x
0 a2  x2
put x  a sin  ; dx  a cos  d
when x  0,   0

x  a,  
2

2
acos  d
Let I   a  sin   cos  
0

Page No. 90
Subject Code & Name: MA 8151 / ENGINEERING MATHEMATICS I Class Notes 2018 - 2019


2
cos  d
I   sin   cos  
0
       (1)

By property of definite int egral,


  
co s     d
2 
2
I
    
0
sin      co s    
2  2 

2
sin  d
I   sin   cos  
0
       (2)
 

sin  d
2 2
cos  d
(1)  (2)  2I     sin   cos 
0
sin   cos  0


2
sin   cos  d 2
   
  0 d     2
   0 
0
sin   cos  0
2  2

 I
4
UNIT IV - MULTIPLE INTEGRALS
Area of a region as double integral
x2 y 2
Find the area bounded by the ellipse   1 using double integration.
1. a 2 b2
Solution: By the symmetry of the curve the area of the ellipse is
Area = 4 Area in the first quardrant
x2
b 1-
a a2
= 4  dydx
0 0

a x2
b 1-
= 4  [y]0 a2
dx
0
a
x2
= 4b  1 - dx
0
a2
a
4b
= 
a 0
a 2 - x 2 dx

a
4b  x 2 2 a 2  x 
=  a - x + sin -1   
a 2 2  a 0
4b  a 2 
=  sin -1 
a 2 
π
= 2ab  
2
= πab.

Page No. 91
Subject Code & Name: MA 8151 / ENGINEERING MATHEMATICS I Class Notes 2018 - 2019

2. Find the double integration the area by the curves y 2 = 4ax and x2 = 4ay .
Solution:
The area is closed by the parabola

y 2 = 4ax - - - - - - - -(1) and x 2 = 4ay - - - - - - - -(2)


To find the limits solve (1) and (2)
x2
(2)  y =
4a
sub in (1)
2
 x2 
  = 4ax
 4a 
x4 = 64a 3
( x4 - 64a 3 ) = 0
x = 0 or ( x3 - 64a 3 ) = 0
x = 0 or x3 = 64a 3 Þ x = 4a
4a 4ax 4a 4a
 x2 
 Area =   dydx =   y  x2 =   4ax -  dx
4ax
dx
0 
0 x2 0 4a
4a 
4a
4a
 x 2 1 x3 
3
 1 2
4a
=   2 a x - x  dx =  2 a
1
2
- 
0 
4a   3 4a 3 
 2 0
4 a 3 1
= (4a) 2 - (4a)3
3 12a
5
4 a 32 32 1 4 2 42 1
= (4) (a) - 64a 3 = a - 64a 3
3 12a 3 12a
5
(22 ) 2 2 16 2 32 2 16 2
= a - a = a - a
3 3 3 3
16
= a2
3
3. Find the area bounded by the parabolas y 2 = 4 - x and y 2 = x by double integration.
Solution:

The area is bounded by

y 2 = 4 - x        (1)
y 2 = x          (2)
y 2 = -(x - 4) is a parabola with vertex (4, 0)
and in the direction of negative x-axis both
the curves are symmetric about x-axis.
To find the limits solve (1) and (2)
Page No. 92
Subject Code & Name: MA 8151 / ENGINEERING MATHEMATICS I Class Notes 2018 - 2019

4-x = x
2x = 4  x  2
y2 = 2  y = ± 2
2
2 4-y
Area = 2   dxdy
0 y2

4-y 2
2
= 2   x dy
0 y2

2
= 2  (4 - y 2 - y 2 ) dy
0

2
= 2  (4 - 2y 2 ) dy
0
2
 2y 3 
= 2 4y -
 3  0

  
3
2 2
= 2 4 2 - - 0
 3 
 
 2(2)3 2 
= 2 4 2 -
 3 
 2(2)(2)1 2 
= 2 4 2 - 
 3 
 4 2
= 2 4 2 - 
 3 

  1
= 2 4 2 1 - 
 3

2
=8 2  
3
16
= 2
3
4.
 (x  y 2 )dy dx over the region R for which x, y  0, x  y  1.
2
Evaluate
R

Solution:

The region of integration is the triangle bounded by the lines

x  0, y  0, x  y  1

Page No. 93
Subject Code & Name: MA 8151 / ENGINEERING MATHEMATICS I Class Notes 2018 - 2019

Limits of y : 0 to 1–x ; Limits of x : 0 to 1


1 1 x

 ( x 2  y 2 )dy dx =   x  y 2 dydx
2

R 0 0

1 x
 1
y3

= x 2 y   dx
0
 3 
0
1
 2 (1  x)3 
 0 
 x (1  x ) 
3 
dx
1
 x3 x 4 (1  x)4 
   
3 4 12  0
1 1 1
  
3 4 12
1

6
Area of a region R in polar form is  rdrdθ
R
1. Find the area of the cardioid r = a(1 + cosθ) by using double integration.
Solution:
Given the curve in polar co ordinates r = a(1 + cosθ)
 Area of the cardioid = 2(Area above the initial line)
θ varies from 0 to π
r varies from 0 to r = a(1 + cosθ)

π a(1+cosθ)
Area = 2  r drdθ
0 0
a(1+cosθ)
 r2 
π
= 2   dθ
0 
2 0
π
=  a 2 (1 + cosθ)2 dθ
0
π
= a 2  (1 + 2cosθ + cos 2θ)dθ
0

 1  cos 
π
 
 1 + 2cosθ +    dθ
2
=a
0
2 

π
3 1 
= a 2   + 2cosθ + cos2θ  dθ
0  
2 2
π
3 1 sin2θ 
= a  θ + 2sinθ +
2
sinnπ = 0, n
2 2 2  0

Page No. 94
Subject Code & Name: MA 8151 / ENGINEERING MATHEMATICS I Class Notes 2018 - 2019

3 
= a2  π 
2 
3πa2
=
2
2. Find the area inside the circle r = asinθ but lying outside the cardioid r = a 1- cosθ .
Solution:
Given r = asinθ        (1)
and r = a 1- cosθ     (2)
Eliminating r from (1) and (2)
asinθ = a(1 - cosθ)
sinθ + cosθ = 1- - - - - -(3)
(3)2  sin 2θ + cos 2θ + 2sinθcosθ = 1
1 + 2sin2θ = 1
sin2θ = 0
2θ = 0, π
π
θ = 0,
2

π
2 asinθ
Area =  
0 a(1-cosθ)
r drdθ

π asinθ
2
 r2 
Area = 
0
2
  a(1-cosθ)
π
2
1
 a sin θ - a (1 - cosθ)  dθ
2 2 2 2
=
2 0
π
a2
 sin θ - 1- 2cosθ + cos θ  dθ
2
2 2
=
2 0
π
a2 2

 sin θ -1  2cosθ  cos θ dθ


2 2
=
2 0
π  
a2 2 2 2

 -1  2cosθ dθ  cos θ d   sin θ d


2 2
=
2 0 0 0
2
a
-θ + 2sinθ0 2
π
=
2
a 2  π π 
=   - + 2sin  - 0 
2  2 2 
a2  π  a2
= - + 2 = 4 - π
2 2  4
Page No. 95
Subject Code & Name: MA 8151 / ENGINEERING MATHEMATICS I Class Notes 2018 - 2019

3. Find the area bounded between r = 2cosθ and r = 4cosθ .


Solution:
Area =  r dr dθ
R

Where the region R is the area between


the circles r = 2cosθ and r = 4cosθ
 r varies from r = 2cosθ to r = 4cosθ
π π
θ varies from - to
2 2
π
2 4cosθ
Area =  
-π 2cosθ
r dr dθ
2
π 4cosθ
 r2 
2
=   dθ
-π  2  2cosθ
2
π
1 2
=  16cos 2θ - 4cos 2θ  dθ
2 -π
2
π
1 2
= 
2 -π
12cos 2θ dθ
2
π
2
= 6  cos 2θ dθ

2
π
2
= 6(2)  cos 2θdθ
0
π
1π 2

 cos θdθ = 2 2
2
= 6(2)
22 0

= 3π
CHANGE THE ORDER OF INTEGRATION
For changing the order of integration in a given double integral
Step 1: Draw the region of integration by using the given limits
Step 2: After changing the order, consider
 dxdy as horizontal strip
 dydx as vertical strip
Step 3: Find the new limits
Step 4: Evaluate the double integral.

Page No. 96
Subject Code & Name: MA 8151 / ENGINEERING MATHEMATICS I Class Notes 2018 - 2019

a a
xdydx
1. Change the order of integration in  x
0 y
2
+ y2
and hence evaluate it.

Solution:
Given limits x : y  a ; y :0  a
After changing the order

xdydx
 x2
+ y2
dy dx  vertical strip
Now, limit x :0  a ; y :0  x
a x
x
a x
 1 
 0 0 x2 + y 2 dydx  0 0 x  x2 + y 2  dy dx
x
1  y 
a
  x  tan 1    dx
0 
x  x  0
a

  x  tan 1 1  tan 1  0   dx tan 1 1  , tan 1  0   0
0
4
 
a
    dx
0
4
  a
    x 0
4
a

4
e y
a a

2. Evaluate 0 y y dydx by changing the order of integration.


Solution:
e y
a a
Given 0 y y dydx (correct form)

Given lim its :


y  x, y  
x  0, x  
After changing the order,
dxdy  horizontal strip
a a
e y  a e y  a 
0 y y dydx   
0 y
dy  

 dx 

y 
e y
a
  x 0 dy
y

0
y
e y
a
 ( y  0)dy
0
y

Page No. 97
Subject Code & Name: MA 8151 / ENGINEERING MATHEMATICS I Class Notes 2018 - 2019

a
  e  y dy
0

 e y 
 
 1  0
 e   e 0  
  
  1  1  
 e  e0
1 e  0; e0  e0  1
a a2  x2

3. Evaluate  
0 0
xy dy dx by changing the order of integration.

Solution:
Given y  0, y  a 2  x 2
y 2  a2  x2
x2  y 2  a2
x  0, x  a
After changing the order,
dxdy  horizontal strip
a2  x2 a2  y2
a
 x2 
a

 
0 0
xy dy dx   y  
0  2 a
dy

a
 
y 2
2
a  y 2 dy
0
a a
a2 1 3

2  ydy 
0
2 0
y dy

a a
a2  y2  1  y4 
     
2  2 0 2  4 0
a4 a4
 
4 8
a4 a4
 
4 8
a4

8

1 2 x

4. Changing the order of integration and hence evaluate   xy dy dx .


0 x2

Solution:
Given limits:

Page No. 98
Subject Code & Name: MA 8151 / ENGINEERING MATHEMATICS I Class Notes 2018 - 2019

y  x2
y  2 x  x y  2
x  0, y  1
After changing the order,
dxdy  horizontal strip
1 2 x 1 y 2 2 y


0 x2
xy dy dx  
0 0
 xy dxdy   
1 0
xy dxdy

 I1  I 2 ( say )      (1)
To find I1 :

1 y

I1    xy dxdy
0 0

y
 x2 
1
  y   dy
0  2 0
y 1

  y   0 dy
0  2 
1
y2
 dy
0
2
1
 y3 
 
 6 0
1 
   0
6 
1
I1 
6
To find I 2 :
2 2 y
I2    xy dxdy
1 0
2 y
 x2 
2
  y   dy
1  2 0
  2  y 2 
2
  y dy
 2 
1  
2

y
2
 4  4 y  y 2 dy
1
2

1
21  4 y  4 y 2  y 3 dy

Page No. 99
Subject Code & Name: MA 8151 / ENGINEERING MATHEMATICS I Class Notes 2018 - 2019

2
1  4 y 2 4 y3 y 4 
    
2 2 3 4 1

1  4(2) 2 4  2  24  4(1) 2 4 1 14  


3 3

       
2 2 3 4  2 3 4  
 
1  32 4 1 
 8   4  2   
2 3 3 4
1 28 1 
 10   
2 3 4
1  10(12)  28(4)  1(3) 
  
2 12 
1 5 
  
2  12 
5
I2 
24
1 5
(1)  I  
6 24
9

24
3
I
8
 y y2

5. Change the order of integration   ye
0 0
x
dx dy and hence evaluate it.

Solution

Given limits:
x  0, x  y
y  0, y  
After changing the order,
dy dx  vertical strip
 y y2  y 2
 1 
  ye
0 0
x
dx dy    2 ye x dy dx
20x
 y 2
1 
   2 ye x dy dx
20x

1  
  y 2

    2 ye x dy  dx
2 0  x 
1   yx 
  2

    e d ( y 2 )  dx
2 0  x 

Page No. 100


Subject Code & Name: MA 8151 / ENGINEERING MATHEMATICS I Class Notes 2018 - 2019


 y2 
1
   e x 
2 0
   dx
x  1 
  x  
x

1  y 
  2

     xe x  dx
2 0  x  x
1   
 x2

  0    xe x  dx
2 0   


1

20 xe x dx


1  xe x  e x  
   (1)   
2  1   1 1   0

1 
   xe  x  e  x 
2 0

1
 (0  0)  (0  1) e   0, e0  1
2
1

2
4 a 2 ax

6. Change the order of integration and hence evaluate   dydx .


0 x2
4a

Solution:
Given limits:
x2
y  x 2  4ay        (1)
4a
y  2 ax  y 2  4ax      (2)
x0 , x  4a
Sub (1) in (2),
2
 y2 
   4ay
 4a 
y4
 4ay
16a 2
y 4  64a3 y
 y  64a y   0
4 3

y  y  64a   0
3 3

y  0 and y 3  64a3  0
y0 and y 3  64a3

Page No. 101


Subject Code & Name: MA 8151 / ENGINEERING MATHEMATICS I Class Notes 2018 - 2019

y  0 and y  4a
when y  0  x  0
16a 2
when y  4a  x   4a
4a
After changing the order,
dy dx  vertical strip
4 a 2 ax 4 a 2 ay

 
0 x 2
dydx   
0 y 2
dxdy
4a 4a

4a 2 ay

   x
0 y2
dy
4a
4a
 2

   2 ay  y dy
4 a
0  
4a
 2

  2 y
1

4a 
2
a ( y) dy
0
4a
 ( y )
3
2 3 
 2 a y 
 3 12a 
 2 0
4 3 (4a)3 
 a (4a) 2  
3 12a 
 32a 2 (4a)3  3
   (4) 2
4 4 8
 3 12a 
 32a 2 64a 3 
  
 3 12a 
 32a 2 16a 2  16a 2
   
 3 3  3
Evaluation of double integrals by changing Cartesian coordinates to polar coordinates:
Changing from ( x, y ) to (r ,  ) , the variables are related by x  r cos  , y  r sin 
and dx dy  J dr d  r dr d
   f ( x, y ) dx dy    f (r cos  , r sin  ) dr d
 

e e
( x 2
 y2 )  x2
1. Evaluate dx dy by changing to polar coordinates. And hence find dx
0 0 0

Solution:
x  r cos  , y  r sin  and dx dy  r dr d

r varies from 0 to  ,  varies from 0 to
2

I    e( x  y2 )
2
dx dy
0 0

Page No. 102


Subject Code & Name: MA 8151 / ENGINEERING MATHEMATICS I Class Notes 2018 - 2019


  r2
2

 0  0 e r dr  d


1   t
2


2 0  0 e dt  d let r 2  t  2r dr  dt and r: 0 to   t: 0 to 


2
1 
 e  d
t

2 0
0


2
1
 e  e0  d

 e   0, e0  1
2 0

1 2
1  1   

2  d  2  
0
0
2
   0 
22  4
  

e dx dy   e  x dx  e  y dy 
( x 2
 y2 ) 2 2
Since
0 0 0 0
4
 

e dx  e x dx 
 x2 2

0 0
4
2
 2   

   e x dx     e x dx 
2

0  4 0
2
2 2 x  x2
x
2. Evaluate  
0 0 x2  y 2
dy dx by changing to polar coordinates.

Solution:
x  r cos  , y  r sin  and dx dy  r dr d
The limits of x are x=0 to x=2,
The limits of y are y=0 to y= 2x x2
y  0  r cos   0

 r  0 and cos   0   
2
y  2x  x2  y 2  2x  x2 x  0  r sin   0
x2  y 2  2x  0 sin   0
  0
 r 2  2r cos   0
 r  2 cos 

r var ies from 0 to 2 cos  ,  var ies from 0 to
2
2 2 x x 2
x
I   dy dx
0 0 x2  y 2

Page No. 103


Subject Code & Name: MA 8151 / ENGINEERING MATHEMATICS I Class Notes 2018 - 2019


2 2cos
r cos 
  
0 0
r2
r dr d


2 2cos
  
0 0
r cos  dr d

 2cos
2
 r2 
  cos    d
0  2 0
 
2  4
2 2
1
  4 cos  d  2  cos3  d  2  .1 
3

2 0 0 3  3
a a
x2
3. Evaluate 
0 y x2  y 2
dx dy by changing to polar coordinates.

Solution:
x  r cos  , y  r sin  and dx dy  r dr d
The limits of x are x=y to x=a, The limits of y are y=0 to y=a

x  y  r cos   r sin     ,
4
a
x  a  r cos   a  r 
cos 
y  0  r sin   0
 r  0 and sin   0    0
a 
r varies from 0 to ,  varies from 0 to
cos  4
a a
x2
I   dx dy
0 y x2  y 2
 a
cos
4
r 2 cos2 
  
0 0
r
r dr d

 a
4 cos

  
0 0
r 2 cos 2  dr d

 a
4
 r 3  cos
  cos    d
2

0  3 0

1 4
 a3 
 0   cos3   0d
2
cos
3  

a3 4

3  sec d
0

a3     a3
a3
log(sec  tan  )0 4  log  sec  tan   log  sec 0  tan 0   log  

 2 1 
3 3   4 4  3 

Page No. 104


Subject Code & Name: MA 8151 / ENGINEERING MATHEMATICS I Class Notes 2018 - 2019

2 a 2 x  x2

  x  y 2  dy dx by changing to polar coordinates


2
4. Evaluate
0 0

Solution:
x  r cos  , y  r sin  and dx dy  r dr d

The limits of x are x=0 to x=2a, The limits of y are y=0 to y = 2ax  x
2

y  0  r sin   0
 r  0 and sin   0    0
y  2ax  x 2  y 2  2ax  x 2 x  0  r cos   0

x 2  y 2  2ax  0 cos   0   
2
 r  2ar cos   0
2

 r  2 a cos 

r var ies from 0 to 2 a cos  ,  var ies from 0 to
2
2 a 2 ax  x 2

I   x  y 2  dy dx
2

0 0

2 2 a cos
   r  r dr d
2

0 0

2 2 a cos
   r  dr d
3

0 0

2 a cos
 r4 
2
   d
0
4 0

  2a cos  4
2 
   0 d

0
4 

4 3 1  3a 4
4 2

0  
16a
 cos 4
 d   4 a . . . 
4 4 2 2 4
a a2  y2
5.
  x  y 2  dy dx by changing to polar coordinates.
2
Evaluate
0 0

Solution:

x  r cos  , y  r sin  and dx dy  r dr d

The limits of x are x=0 , x  a 2  y 2 , and limits of y are y=0, y=a

Page No. 105


Subject Code & Name: MA 8151 / ENGINEERING MATHEMATICS I Class Notes 2018 - 2019

x  0  r cos   0 x  a2  y 2  x2  y 2  a2 y0

r  0 and cos   0    r 2  a2 r sin   0
2
ra sin   0    0
π
r varies from 0 to a, θ varies from 0 to
2
a a2  y2

I   x  y 2  dy dx
2

0 0

2a
    r  r dr d
2

0 0

2 a
    r 3  dr d
0 0

a
 r4 
2
    d
0
4 0

 a4 
2
a4  a 4  a 4
    d   0 
2

0
4  4 4 2 8
Change of variables

   x  y
2
1. Evaluate dxdy , where R is the parallelogram in the xy-plane with vertices
R

1, 0 ,  3,1 ,  2, 2 ,  0,1 using the transformation u  x  y and v  x  2 y


Solution:
The vertices A 1,0 , B  3,1 , C  2, 2 , D  0,1 of the parallelogram ABCD in the xy-plane become
A' 1,1 , B'  4,1 , C '  4, 2 , D' 1, 2  in the uv-plane under the given transformation u  x  y and
v  x  2y .
The region R in the xy-plane becomes the region R' in the uv-plane which is the rectangle bounded
by the lines u  1, u  4, and v  2, v  1
1 1
Solving the given equations , we get x   2u  v  , y   u  v 
3 3
dx dy  J du dv
x x
  x, y  u v 1
J  
  u, v  y y 3
u v

    x  y dxdy    u 2 J dudv
2

R R'

Page No. 106


Subject Code & Name: MA 8151 / ENGINEERING MATHEMATICS I Class Notes 2018 - 2019

1 4
1
  u
2
dudv
2 1
3
4
1  u3 
1
    dv
2 
3 3 1
1
  7 dv
2

 7  v 2
1

 21
Triple Integral
z2 y2 x2

I   
z  z1 y  y1 x  x1
f (x, y, z) dx dy dz

Also
x2 y2 z2

I   
x  x1 y  y1 z  z1
f (x, y, z) dz dy dx

1 2 3

1. Evaluate    xyz dz dy dx
0 0 0

Solution:
3
1 2 3 1 2
 z2 
I    xyz dz dy dx   
x=0 y=0 z=0 0 0
  dy dx
 2 0
2
 y2 
3
9 
1 2 1
9
  2 0  2 0
  0  dy dx  x   dx
0 0  2 0
1
9
1
4  9
1 1
 x2  1  9
  x   0  dx   2x dx  9 x dx  9    9   0  
20 2  20 0  2 0 2  2
1 1 1

  e
x+y+z
2. Evaluate dxdydz
0 0 0

Solution:

1 1 1 1 1

  e
x  yz
  e  e y z  dy dz
1 y  z
I= dx dy dz =
z=0 y=0 x=0 0 0

 e 
z2
=  2ez 1  ez dz
0

 e3  3e2  3e  1
  e  1
3

4 x x y
3. Evaluate    z dx dy dz
0 0 0
Page No. 107
Subject Code & Name: MA 8151 / ENGINEERING MATHEMATICS I Class Notes 2018 - 2019

4 x xy

I=   
x=0 y=0 z=0
z dz dy dx

x y
4xz2 
=    dydx
0 0  2  0
4x
= 1   x  y dydx
2 00
x 4
1 4 y 2  1  2 x 
4 2
34 2 3  x 3 
=   xy  dx =   x   dx =  x dx   16
2 0  2  2 0 2  40 4 3 
0  0
a a2  x2 a2  x2  y2
dz dy dx
4. Evaluate  
0 0

0 a  x2  y2  z2
2

Solution:
a a2  x2 a2  x2  y 2
dz dy dx
Let I  
x 0

y 0

z 0 a2  x2  y 2  z 2
a2  x2  y 2
a a2  x2   z 
  sin 1   dy dx
0 0   a  x  y   0
2 2 2

a a2  x2

  sin 1 1  sin 1  0  dy dx


0 0

a a2  x2
   a

    
a2  x2
2  0  dy dx  y dx
 
0
0 0 20


a
 x   x
a
a2
2 0
 a  x dx  
2
a  x  sin 1   
2 2 2

2 2 2  a  0

  a2     2 a2
  0     0  0  
2  2 2  8
dz dy dx
5. Evaluate   1  x2  y2  z2
for all positive values of x,y,z for which the integral is real.

Solution:
1 1 x 2 1 x 2  y 2
dz dy dx
Let I
x 0
  y 0

z 0 1  x2  y 2  z 2
1 x 2  y 2
1 1 x 2   z 
  sin 1   dy dx
  1  x2  y 2 
0 0  0

Page No. 108


Subject Code & Name: MA 8151 / ENGINEERING MATHEMATICS I Class Notes 2018 - 2019

1 1 x 2

  sin 1 1  sin 1  0   dy dx


0 0

1 1 x 2
   1

 2  0  dy dx  2   y 0
1 x 2
  dx
0 0   0

 1
2
2 0
 1  x 2 dx 
8
dz dy dx
6. Evaluate   x  y  z  1
V
3
over the region of integration bounded by the planes x  0, y  0 ,

z  0, x  y  z  1
Solution:
Here z varies from z  0 to z  1  x  y
y varies from y  0 to y  1  x
x varies from x  0 to x  1

1 1 x 1 x  y
dz dy dx 1
       x  y  z  1 dz dy dx
 x  y  z  1
3 3
V x 0 y 0 z 0

1 1 x 1 x  y

    x  y  z  1
3
dz dy dx
0 0 0

1 x  y
1 1 x   x  y  z  1 2 
    dy dx
0 0  2  0

1 1 x
1  2 2   x  y  1 2  dy dx
2 0 
  
0

1 1 x
1 1 
2 0   4   x  y  1
2
  dy dx
0  

1 x
1 1
1

   y
 x  y  1 
1

 dx
2 0  4 1  0

1 x
1 1 1 
1

    y   x  y  1  dx
2 0 4 0

 1 1 
 1 

1
1

2 0  4 1  x   2   0   x  1  dx
 

1 1 x 1 
1
1
     
2 0  4 4 2 1 x  dx

Page No. 109


Subject Code & Name: MA 8151 / ENGINEERING MATHEMATICS I Class Notes 2018 - 2019

1 3 x 
1
1
      dx
2 0  4 4 1 x 

1
1 3 x2 
   x   log 1  x  
2 4 8 0

1  3 1  
     log 2   0  0  0 
2  4 8  
1 5
 log 2 
2 16
Volume Integral
2
x y2 z2
1. Find the volume of the ellipsoid 2  2  2  1
a b c
Solution:
x2 y 2 z 2
Since the ellipsoid    1 is symmetric about the coordinate planes.
a 2 b2 c 2
Volume of ellipsoid = 8  volume in the first octant.
In the first octant,
x2 y 2
Z varies from 0 to c 1  
a 2 b2
x2
Y varies from 0 to b 1  2
a
x varies from 0 to a
x2 x2 y 2
b 1 2
c 1 2  2
a a a b
volume  8    dz dy dx
x 0 y 0 z 0

x2
b 1
a a2 x2 y 2
c 1 
 8
0
  z
0
0
a 2 b2
dy dx

x2
b 1
a a2
x2 y 2
 8  c 1  dy dx
0 0
a 2 b2
x2
b 1
a2
c  2  x2  2 
a
 8   b 1  2   y  dy dx
b  a  
0 0  
x2
b 1
  x   2
 a2

a 
b 2 1  2   
8c  y 2  x  2
2
 a  1  y 
  b 1  2   y  sin dy dx
b 0 2  a  2  x2 
  b 1 2 
  a 0
Page No. 110
Subject Code & Name: MA 8151 / ENGINEERING MATHEMATICS I Class Notes 2018 - 2019

4c 2  x 2 
a
  
b 1  2  sin 1 1 sin 1 0 dx
b 0  a 

4c 2  x 2  
a

b 0  a 2  2
 b 1   dx

a
 x2 
 2 bc  1  2  dx
0
a 
a
 1 x3 
 2 bc  x  2 
 a 3 0
 a3   a  2a  4
 2 bc  a  2  0  2 bc  a    2 bc     abc
 3a   3  3  3
x y z
2. Find the volume of the tetrahedron bounded by the plane    1 and the coordinate’s planes.
a b c
Solution:
x y z
The region of integration is the region bounded by    1, x  0, y  0, z  0
a b c
 x y
Z varies from 0 to c 1   
 a b
 x
y varies from 0 to b  1  
 a
x varies from 0 to a
 x  x y
b 1  c  1  
a  a  a b
volume   
x 0 y 0

z 0
dz dy dx

 x
b1 
 x y

  z
a  a c1  
  a b
dy dx
0
0 0
 x
b1 
a  a
 x y
  c 1    dy dx
0 0  a b
 x
b1 
 x 
a
y2   a
 c  1   y   dx
0 
a 2b  0
 x   x  1 2  x  2 
a
 c  1   b 1    b 1    dx
0 a   a  2b  a  
a
  x 2 b  x 2 
 c  b 1    1    dx
0  a  2  a  
a
 b  x 2 
 c   1    dx
0 2  a  

Page No. 111


Subject Code & Name: MA 8151 / ENGINEERING MATHEMATICS I Class Notes 2018 - 2019

bc  x  
a 2

  1    dx
2 0  a  
a
  x 3 
 1 
bc   a   abc abc


  0  1 
2  3  6 6
 a 
 0
Find the volume of sphere x 2  y 2  z 2  a 2 using triple integrals.
3.
Solution:
Since the sphere x 2  y 2  z 2  a 2 is symmetric about the coordinate plane
Volume of sphere = 8  volume in the first octant.
Int the first octant,
Z varies from 0 to a2  x2  y 2

Y varies from 0 to a2  x2
x varies from 0 to a
a a2  x2 a2  x2  y 2

Volume of sphere =  8    dz dy dx
y 0 x 0 z 0

a a2  x2

  z 0
a2  x2  y 2
 8 dy dx
0 0

a a2  x2
 8  a 2  x 2  y 2 dy dx
0 0

a2  x2
a y a2  x2  y 2 a2  x2  y 
 8   sin 1   dx
0   a x   0
 2 2 2 2

a
 a 2  x 2 1   a 2  x 2 1 
 8  0  sin (1)   0  sin (0)  dx
0   
2 2
 a2  x2 
a
 8   sin (1)  sin (0)  dx
1 1

0
2 
 
a
 4   a 2  x 2    0  dx
0 2 
a
 x3   a3   2a 3  4 a 3
 2  a 2 x    2  a 3    2  
 3 0  3  3  3

Page No. 112


Subject Code & Name: MA 8151 / ENGINEERING MATHEMATICS I Class Notes 2018 - 2019

UNIT V - DIFFERENTIAL EQUATIONS


To Find C.F
S.I Roots of A.E C.F
1 If the roots are real and
different Ae m1 x  B e m2 x
m1, m2 (m1 ≠m2)
2 If two roots are equal and real
m1 = m2 = m(say)
 Ax  B  e mx
3 If three roots are equal and real
m1 = m2 = m3 = m(say)
 Ax 2
 Bx  C  e mx
4 If the roots are complex e x  A cos  x  B sin  x 
α±iβ
5 If the complex roots are
repeated α ± i β (twice) e x   Ax  B  cos  x   Cx  D  sin  x 

To Find P.I
S.No f(x) P.I
1 1
P .I  e x  e x , provided f    0
f  D f  
If f    0, then
1
1 e x P .I  x e x , provided f '    0
f  
'

If f    0, then
'

1
P .I  x 2 e x , provided f "   0
f  "

1 1
P .I  x n   f  D   x n
f  D
n
2 x
1
Expand  f  D   and then operate

1
P .I  sin  x (or ) cos  x
f  D
Replace D 2    2
3 sin αx (or) cos αx
After replacing D2    2 , if the denominator = 0, then
multiply x with the numerator and differentiate the
denominator with respect to D.

1 1
P .I  e ax g ( x )  e ax g( x)
4 e ax
g( x) f  D f  D  a

Page No. 113


Subject Code & Name: MA 8151 / ENGINEERING MATHEMATICS I Class Notes 2018 - 2019

1
P .I  x V( x )
f  D
f ' D 
5 x V(x)
1
x V( x )  V( x )
f  D  f  D  
2

1
P .I  x n sin  x (or ) x n cos  x
f  D
6 xn sinax (or) xn cosax
1
 R.P of ( x n e i x ) (or ) I.P of ( x n e i x )
f  D

Binomial Expansions:
(i) ( 1+ x )-1 = 1 – x + x2 – x3 + ………+
(ii) ( 1- x )-1 = 1 + x + x2 + x3 + ………+
(iii) ( 1+ x )-2 = 1 – 2x + 3x2 – 4 x3 + ………+
(iv) ( 1- x )-2 = 1 + 2x + 3 x2 + 4x3 + ………+
Solve  D2  2D  8 y  4cosh x sinh 3x   e2 x  e x   1 .
2
1.

Solution:
The A.E. is  m2  2m  8   0
  m  4 m  2  0
 m  2, 4
C.F.: Ae2 x  Be4 x
R.H.S  4cosh x sin 3x   e2 x  e x   1
2

 e x  e x  e3 x  e3 x 
  e  e  1
x 2
 4  
2x

 2  2 
   e  e  e  e   e 4 x  2e3 x  e 2 x  1
4x 2 x 2x 4 x

 e2 x  e4 x  2e3 x  1e0 x


P.I. 
 D  4
1
 D  2 
 
e2 x 
 D  4
1
 D  2 
e4 x  2e3 x  e0 x 
1 e4 x 2e3 x 1
 e2 x   
 2  4  D  2   8 2   1 5  4  2 
 xe2 x e4 x 2e3 x 1
   
6 16 5 8
2 x
2 x xe e4 x 2e3 x 1
G.S is y  Ae  Be  4x
   .
6 16 5 8
2
d x
2. Solve  10 x  cos8 y.
dy 2
Solution:
Here y is independent and x is dependent variable
d
Let D  .
dy
The A.E is m 2  10  0
 m2  10
Page No. 114
Subject Code & Name: MA 8151 / ENGINEERING MATHEMATICS I Class Notes 2018 - 2019

 m   10i
C.F.: A cos 10 y  B sin 10 y
1 cos8 y  cos8 y
P.I  2 cos8 y  
 D  10  64  10 54
cos8 y
G.S. is x  A cos 10 y  B sin 10 y 
54
3. d2y dy
Solve 2
 6  9 y  sin x cos 2 x .
dx dx
Solution:
The A.E is m2  6m  9  0
 m  3  0
2

m  3,  3.
C.F.:  A  Bx  e3x
2sin x cos 2 x 1
R.H.S   sin 3x  sin   x  
2 2
1
 sin 3 x  sin x 
2
 2sin A cos B  sin  A  B   sin  A  B   A  x, B  2 x

1 1 1 1
P.I.  sin 3 x  sin x
2  D  3 2
2  D  32
1 1 1 1
 sin 3x  sin x
2 D  6D  9
2
2 D  6D  9
2

1 1 1 1
P.I.   sin 3 x   sin x
2 9  6 D  9 2 1  6 D  9
1 1 1 1
  sin 3x   sin x
12 D 2 8  6D
 cos3x  4  3D 
  sin x
12  3 4  4  3D  4  3D 
1 1  4  3D 
 cos 3 x  sin x
36 4 16  9 D 2
1 1 4sin x  3cos x
 cos 3x  
36 4 16  9
 cos 3x sin x 3cos x
  
36 25 100
 cos 3x sin x 3cos x
y=  A  Bx  e3x +   
36 25 100
4. Solve  D  4  y  x  cos x
2 4 2

Solution:
The A.E. is m 2  4  0
m  2i
C.F.: A cos 2x  B sin 2x

Page No. 115


Subject Code & Name: MA 8151 / ENGINEERING MATHEMATICS I Class Notes 2018 - 2019

1 1  1  cos 2 x 
P.I  x4  2  
D 4
2
D 4 2 
1 1 1 1 1 1
 x4  e0 x  cos 2 x
4 D  2
2  D  4
2
2 D2  4
1  
 4 
 x sin 2 x 
1
1  D2  4 1
 1   x  
4 4  2  4  2  2  2 
1  D 2 D 4  4 1 x sin 2 x
 1   x  
4 4 16  8 8
x 4 12 x 2 4.3.2.1 1 x sin 2 x
    
4 16 64 8 8
4 3x 2 x 4 x sin 2 x
G.S. is y  A cos 2 x  B sin 2 x    
8 4 4 8
Solve  D 2  2 D  1 y   x  e x   cos 2 x cosh x .
2
5.

Solution:
The A.E is m 2  2m  1  0
2  4  4
m  1  2
2
 1 2  x  1 2  x
C.F.: Ae  Be
e x
 e x 
P.I. 
1
x 2
 2 xe  e
x 2x
 1
cos 2 x
 D2  2D  1 D 2
 2D  1 2
1 1
P.I1 = x2   x2
 D 2
2 D  1 1   2 D  D  
 2

  1   2 D  D 2    2 D  D 2   .... x 2
2

 
  1  2 D  D  4 D  x
2 2 2

  1 x 2   2 D  x 2   5D 2  x 2  

   x 2   2  2 x   5  2  
  x 2  4 x  10
1
x 2   x 2  4 x  10
D  2D  1
2

2  2e x 
P.I 2  2 xe x   x
D  2D  1   D  12  2  D  1  1 
 
1
 2e x 2 x
D  2D  1  2D  2  1
2e x
 2  x
D  4D  2

Page No. 116


Subject Code & Name: MA 8151 / ENGINEERING MATHEMATICS I Class Notes 2018 - 2019

2e x 1
 x
2   D2 
1   2 D  
  2 
1
  D2 
 e 1   2 D 
x
 x
  2 
  D2  
 e x 1   2 D    ... x
  2  
 e x 1  2D x
2
P.I 2  xe x  e x  x  2   x  2  e x
D 2
 2 D  1
1 1 e2 x
P.I 3  e 2x
 e 2x

D2  2D  1  4  4  1 7
1 e x cos 2 x
P.I 4 
D2  2D  1 2
x
e 1
 cos 2 x
2  D  1  2  D  1  1
2

ex 1
 cos 2 x
2 D  2D  1  2D  2  1
2

ex 1
 cos 2 x
2 4  4 D  2
ex 1
 cos 2 x 
2 4D  2
ex 1
 cos 2 x
2 2(2 D  1)
ex  2D  1
 cos 2 x
2 2(2 D  1)  2 D  1
e x  2 D  1
 cos 2 x
2 2(4 D 2  1)
e x  2 D  1
 cos 2 x
4 (4  4   1)
ex 1
  2.2sin 2 x  cos 2 x 
4  16  1
e x  4sin 2 x  cos 2 x 

4  16  1
e x  cos 2 x  4sin 2 x 

17
1 e x cos 2 x
P.I 5  2
D  2D  1 2

Page No. 117


Subject Code & Name: MA 8151 / ENGINEERING MATHEMATICS I Class Notes 2018 - 2019

e x 1
 cos 2 x
2  D  1  2  D  1  1
2

e x 1
 cos 2 x
2 D2  2
e x 1
 cos 2 x
2 4  2
e x
 cos 2 x
12
The General Solution is
 1 2  x 1 2  x e2 x
y  Ae  Be  10  4 x  x 2    x  2 ex
7
x x
e e
  cos 2 x  4sin 2 x   cos 2 x
17 12
6. Solve  D 2  4  y  x 2 cos 2 x.
Solution:
The A.E is m 2  4  0
 m2  4
 m  2 i
C.F.: A cos 2x  B sin 2x
P.I  2
1
D 4
 x 2 cos 2 x 

1 e2ix
 R.P of 2 x 2ei 2 x  R.P. of x2
D 4  D  2i  4
2

1
P.I.  R.P of e 2ix x2
D  4iD  4  4
2

1 1
 R.P of e2ix 2 x 2  R.P of e2ix x2
D  4iD D  D  4i 
1 1
 R.P of e 2ix x2
D  D
4i 1  
 4i 
1
e 2ix 1  D  2
 R.P of 1   x
4i D  4i 
e2ix 1  D D 2  2
 R.P of 1   x
4i D  4i 16 
e2ix  2 2 x 2 
4i  
 R.P of x   dx
4i 16 
 ie2ix  x3 x 2 x 
 R.P of     
 4  3 4i 8 
 ie2ix  x3 ix 2 x 
 R.P of     
 4  3 4 8

Page No. 118


Subject Code & Name: MA 8151 / ENGINEERING MATHEMATICS I Class Notes 2018 - 2019

 e2ix   x3i x 2 ix 
 R.P of     
 4  3 4 8
 cos 2 x  i sin 2 x    x3i x 2 ix 
 R.P of    
4  3 4 8
1  x 2 cos 2 x x3 sin 2 x x sin 2 x 
   
4 4 3 8 
1  x 2 cos 2 x x3 sin 2 x x sin 2 x 
P.I.   
4  4 3 8 
x 2 cos 2 x x3 sin 2 x x sin 2 x
G.S.: y  A cos 2 x  B sin 2 x    .
16 12 32
7. Solve  D 2  4 D  3 y  e x sin x  xe3 x .
Solution:
The A.E is m2  4m  3  0
 m  1 m  3  0
m  1, 3
C.F.: Ae x  Be3 x
1 1
P.I  e x sin x  xe3 x
 D  3  D  1  D  1 D  3 
e x e3 x
  sin x    x
 D  1  3 D  1  1  D  3  1 D  3  3
1 1
 e x sin x  e3 x x
 D  2 D  D  4  D  6 
1 1
 e x
 D  2  sin x dx  e3 x
 D  4  D  6 
x

D2 1
 e x cos x  e3 x 2 x
 D  2 D  2 D  10D  24
D2 e3 x 1
 e  x cos x  x
D2  4 24 10 D D 2
1 
24 24
1
x D2 e3 x  5D D 2 
 e cos x  1  12  24  x
1  4 24  
D  cos x   2 cos x
1
x  5D D 2 
e3 x
 e 
1  12  24  x
1  4  24 
e   sin x  2cos x  e  5D 
x 3x
  1 x
 1  4 24  12 
e x
 sin x  2cos x    x  
e3 x  5

5 24  12 
x
e e3 x  5
G.S. is y  Ae x  Be3 x   sin x  2cos x    x  .
5 24  12 

Page No. 119


Subject Code & Name: MA 8151 / ENGINEERING MATHEMATICS I Class Notes 2018 - 2019

Cauchy’s homogeneous linear differential equation.


The general form of Cauchy’s homogeneous linear differential equation is given by
n 1
dny n 1 d y d n2 y
a0 x n
 a1x  a2 x  ...  a n y  f  x 
n2
dx n dx n 1 dxn  2
where a0, a1, a2 ,..., a n areconstants
Procedure:
Substitute x  e z or z  log x.
d d
xD  D ' , x 2 D 2  D ' ( D '  1).D 
, D' 
dx dz
After substitution convert the given equation to ordinary into linear differential equation with
constant coefficients
9.
 
Solve x 2 D2  3 xD  5 y  x cos  log x 
Solution:
Substitute x  e z or z  log x.
d d
xD  D ' , x 2 D 2  D ' ( D '  1) .D 
, D' 
dx dz
 D ( D  1)  3D  5 y  e cos z
' ' ' z

D '2
 2 D '  5  y  e z cos z
Auxillary equation is m 2  2m  5  0
 2  4  20
m
2
 2  4i
m  1  2i
2
Complementary Function is e  z ( A cos 2 z  B sin 2 z )
e z cos z
Particular Integral= '2
D  2 D'  5
cos z
= ez
( D  1)  2( D'  1)  5
' 2

cos z
= e z '2 [substitute D'2  1]
D  4D  8 '

cos z
= ez
7  4D'
z (7  4 D' ) cos z
=e
(7  4 D' )(7  4 D' )
(7 cos z  4 sin z )
= ez
49  16
z
e
= (7 cos z  4 sin z )
65
ez
y= e  z ( A cos 2 z  B sin 2 z ) + (7 cos z  4 sin z )
65
1 x
y = ( A cos(2 log x)  B sin(2 log x))  (7 cos(log x)  4sin(log x))
x 65

Page No. 120


Subject Code & Name: MA 8151 / ENGINEERING MATHEMATICS I Class Notes 2018 - 2019

10. 2
 log x 
Transform the differential equation ( x D  xD  1) y  
2 2
 into linear differential
 x 
equation with constant coefficients and solve.
Solution:
2
 x 2 D 2  xD  1 y   
 log x 
 x 


x  e z , z  log x, xD  D' , x 2 D2  D' D'  1 

 D'2  2D'  1 y  z 2e2 z
A.E is m 2  2m  1  0
m  1,1
CF  ( Az  B ')e z
z 2 e 2 z
PI 
D'  2D'  1
2

 z2 
 e 2 z  '2 
 D  2D  1 
'

replaceD'byD'  2
 z2 
 PI  e2 z  ' 
 ( D  2)  2( D  2)  1 
2 '

 z2 
 e 2 z  '2 
 D  4D  4  2D  4  1 
' '

 z2  2 z  z2


 e 2 z  '2   e
 D  6D  9 
'   D '  32 
 
 
 
 e  2  D  
2 z ' 2
2 z  z2
e   z 1  
' 2  9   3  

 9 1 D    
   
  3  

e 2 z  
2
 D'   D' 
 1  2    3    ..  z 2
9   3   3  
 
2 z
e  2 4 2
 z  z 
9  3 3
Y= CF+PI
e2 z  2 4 2
= ( Az  B ')e + z  z 
z

9  3 3
1  4 2
= ( A log x  B) x + 2 
(log x) 2  (log x)  
9x  3 3

Page No. 121


Subject Code & Name: MA 8151 / ENGINEERING MATHEMATICS I Class Notes 2018 - 2019

11.  1  12  log x 
Solve the given equation  D 2  D  y 
 x  x2
Solution:
 1  12  log x 
Given  D 2  D  y 
 x  x2
Multiply throught by x2
d2y dy
We get x 2 2  x  12  logx  →(1)
dx dx
x  ez  or  z  log x
xD  D   2
d
x2 D2  D  D 1   3 Where D denotes
dz
Sub (2) & (3) in (1) we get,
 D  D 1  D y  12z
 i.e   D2  y  12 z
The A.E is m2  0
C.F.: Az  B
To find the P.I
12 z 1  z2 
 12 '  
D 2 D  2
 z3 
 12  
6
P.I  2z3
 y  ( Az  B)  2 z 3
 ( A log x  B)  2  log x 
3

12. d2 y dy
Solve x 2
2
 4x + 2y = sin  logx 
dx dx
Solution:
2
2 d y dy
Given equation is x 2
 4 x  2 y  sin  log x  .
dx dx
 x D  4 xD  2  y  sin  log x   1
2 2

Put x  ez  or  z  log x
xD  D   2
d
x2 D2  D  D  1   3 Where D denotes
dz
Sub (2) & (3) in (1) we get
 D  D 1  4D  2 y  sin z
 i.e   D2  D  4 D  2  y  sin z
 D2  3D  2  y  sin z   4 
The A.E is m2  3m  2  0
Page No. 122
Subject Code & Name: MA 8151 / ENGINEERING MATHEMATICS I Class Notes 2018 - 2019

 m  1 m  2  0
m  1, 2
C.F.: Ae z  Be2 z
1
P.I.= 2 sin z
D  3 D  2
1
 sin z
1  3D  2
1
 sin z
3 D  1
3D  1
 sin z
9 D 2  1

 3D  1 sin z  Replace D2 by  1
9  1  1  

3D  sin z   sin z



10
3cos z  sin z

10
 The solution of (4) is
3cos z  sin z
y  Ae  z  Be 2 z 
10
Sub z  log x or x  e , we get
z

3cos  log x   sin  log x 


y  Ae log x  Be2log x 
10
y  Ax 1  Bx 2 
3cos  log x   sin  log x 
10
A B 3cos  log x   sin  log x 
y  2 
x x 10
Legendre’s linear differential equation.
The general form of Legendre’s linear differential equation is given by
n 1
dny n 1 d y n2 d n2 y
A0 (a  bx) n
 A1(a  bx)  A2 (a  bx)  ...  A n y  f ( x)
dx n dx n 1 dx n 2
ez  a
Put bx  a  e , z  log  bx  a  , x 
z
b ,
 ax  b  D  aD '
 ax  b 2 D2  a2  D'2  D' 
13. Solve the Legendre’s linear equation  3x  2 D2  3  3x  2  D  36 y  3x 2  4 x  1.
2
 
Solution:
Let  3x  2  D2  3  3x  2  D  36 y  3x 2  4 x  1
2
 
ez  2
Let 3x  2  e z or z  log  3x  2   x 
3
dz 3
 
dx 3x  2
Page No. 123
Subject Code & Name: MA 8151 / ENGINEERING MATHEMATICS I Class Notes 2018 - 2019

3x  e z  2
1 2
x  ez 
3 3
Let  3x  2 D  3D
 3x  2  D 2  9 D  D  1
2

1 2 1 2
9 D  D  1  3  3D   36  y  3  e z    4  e z    1
3 3 3 3
1 4 4  4 8
9 D2  9 D  9 D  36  y  3  e 2 z   e z   e z   1
9 9 9  3 3
1 4 4 4 8
9 D2  36  y  e 2 z   e z  e z   1
3 3 3 3 3
1 2z 1
 e 
3 3
A.E is 9m  36  0
2

9m 2  36
m2  4
m 2
C.F  Ae 2 z  Be 2 z
 A  3x  2   B  3x  2 
2 2

1 e2 z
P.I1 
9 D2  36 3
1 1
 . e2 z
3 36  36
1 1 2z
 z e
3 18D
1 e2 z
 z
54 2
1
 ze 2 z
108
1
  log 3x  2  3x  2 
2

108
1 e0 z
P.I 2 
9 D12  36 3
1 1 0z 1
 . e 
3 36 108
y  C.F  P.I1  PI 2
1 1
 A  3x  2   B  3x  2    3x  2  log  3x  2  
2 2 2

108 108
1 
 A  3x  2   B  3x  2    3x  2  log  3x  2   1 .
2 2 2

108 
14. Solve  x  1 D 2   x  1 D  1 y  4cos log( x  1).
2
 
Solution:
Put x  1  e z , z  log  x  1 , x  e z  1
Page No. 124
Subject Code & Name: MA 8151 / ENGINEERING MATHEMATICS I Class Notes 2018 - 2019

 x  12 D2  D'  D' 1 


 x  1 D  D'
 
 D' D'  1  D'  1 y  4 cos z
 
 D' 2  1 y  4cos z
 
To find C.F :
The Auxillary equation is m 2  1  0
m 2  1
m  i
 y  A cos z  B sin z
To find P.I :
4cos z
P.I  ' 2
D 1
put D'  1
4 cos z
P.I 
1  1
4 cos z
=
0
1
4z '
cos z
= 2 D
 2 z  cos z dz

= 2z sin z
 y  A cos z  B sin z  2 z sin z
 A cos log  x  1  B sin log  x  1  2log  x  1 sin log  x  1
15. Transform the differential equation  2 x  5  D 2  6  2 x  5  D  8  y  6 x into linear
2
 
differential equation with constant coefficients and solve the equation.
Solution:
Put 2 x  5  e z , z  log  2 x  5 

 2 x  52 D2  22  D'  D' 


2

 
 2 x  5 D  2 D '
Hence the given equation becomes
  '2 ' 

 4  D  D   12 D  8 y  3 e  5
' z

  D'  4 D'  2 y  e z  5
3
 
2

  4
To find C.F :
Auxillary equation is

Page No. 125


Subject Code & Name: MA 8151 / ENGINEERING MATHEMATICS I Class Notes 2018 - 2019

m 2  4m  2  0
(4  2 2)
m
2
 2 2  z  2 2  z
 CF  Ae  Be

4
 e  5
3 z
3 ez  3 5e0 z 
PI       
 D'  4D'  2 4  D'  4D'  2  4  D'  4D'  2 
2 2 2

 
3 5
  e z  
4 2
 y  CF  PI
 2 2  z 3 5
 2 2  z
 Ae  Be
  e z   wherez  log(2 x  5)
4 2
16. Solve Dx  y  sin 2t , and  x  Dy  cos 2t
Solution:
Dx+y=sin2t…..(1)
-x+Dy=cos2t…..(2)
(1) => Dx+y=sin2t
Operate D on 2 =>  Dx  D2 y  2sin 2t.....(3)
(1)+(3)=> y  D 2 y   sin 2t
( D 2  1) y   sin 2t
Auxillary equation is m 2  1  0
m 2  1
m  i
 y  A cos t  B sin t
 sin 2t
Particular Integral= 2
D 1
(sub D 2  4)
 sin 2t sin 2t
= 
3 3
sin 2t
 y  A cos t  B sin t 
3
-x+Dy=cos2t…..(2)
 sin 2t 
-x+D  A cos t  B sin t   =cos2t
 3 
1
 x  A sin t  B cos t  (2 cos 2t )  cos 2t
3
1
 x  A sin t  B cos t  cos 2t
3
cos 2t
 x   A sin t  B cos t 
3
cos 2t
x   A sin t  B cos t  ;
3
sin 2t
y  A cos t  B sin t 
3
Page No. 126
Subject Code & Name: MA 8151 / ENGINEERING MATHEMATICS I Class Notes 2018 - 2019

17. Solve the simultaneous ordinary differential equation  D  4  x  3 y  t , 2 x   D  5 y  e2t .


Solution:
Given  D  4 x  3 y  t  1
2 x   D  5 y  e 2t   2 
2  1   D  4   2
6 y   D  4 D  5 y  2t   D  4 e2t
6  D 2  9 D  20 y  2t  2e 2t  4e zt

 D2  9D  14  y  6e2t  2t
The A.E. is m2  9m  14  0
 m  7 m  2  0
m  2,  7
C.F.: Ae 2t  Be 7 t
6 2
P.I.  2 e 2t  2 t
 D  9D  14   D  9D  14 
6e 2t 2 1
  t 
4  18  14 14 9D D2
1 
14 14
1
6e 2 t 1  9 D D 2 
  1    t 
36 7  14 14 
e2t 1  9 D  e 2t 1  9
  1    t    t  
6 7  14  6 7  14 
2t
e t 9
G.S. is y  Ae2t  Be7t   
6 7 98
To Calculate x
2e2t 1
Dy  2 Ae2t  7 Be7t  
6 7
2t
5e 5t 45
5 y  5 Ae2t  5Be7t   
6 7 98
7e2t 5t 1 45
 D  5 y  3 Ae2t  2Be7t    
6 7 7 98
 2   2 x    D  5 y  e 2t

7e2t 5t 31 2t
 3 Ae2t  2 Be7t    e
6 7 98
3 A 2t 7 5t 31
x e  Be7 t  e 2t  
2 72 14 196
The General solution is
3 A 2t 7 t e2t 5t 31
x e  Be   
2 12 14 196
e 2t t 9
y  Ae2t  Be7t    .
6 7 98

Page No. 127


Subject Code & Name: MA 8151 / ENGINEERING MATHEMATICS I Class Notes 2018 - 2019

Method of variation of Parameters:


d2 y dy
2
 a  by  R, R is a function of x.
dx dx
The complementary function is C.F = c1 f1 + c2f2
Where c1, c2 are constants and f1 , f2 are functions of x.
Particular Integral is P.I = P f1 + Q f2
f2 R f1 R
Where P    dx and Q   dx

f f f f  f f  ff

1 2 1 2 1 2 1 2

Where wronskian Value =w(f1,f2) = f 2 f1  f1 f 2


Hence the complete solution is y = C.F + P.I

d2y
18. Solve by the method of variation of parameters  4 y  sec 2 x .
dx 2
Solution:
The A.E is m 2  4  0
m  2i
C.F = c1 cos 2x  c2 sin 2x
P.I = Pf1 + Qf2
f1 = cos 2x; f2 = sin 2x
 
f1  2sin 2 x ; f 2  2 cos 2 x
f f  ff 2
2 1 1 2

f2 R
Now, P    dx
f1 f 2  f1 f 2
sin 2 x
=  sec 2 xdx
2
=  1  tan 2 xdx  1 log  cos 2 x 
2 4
f1 R
Q dx
f1 f 2  f1 f 2
1 1
=
2  cos 2 x sec 2 xdx  x
2

 y = C.F + Pf1 + Qf2


1
= c1 cos 2x  c2 sin 2x  1 log  cos 2 x  cos 2 x  x sin 2 x .
4 2
19. d y 2
Solve:  y  tan x by method of variation of parameters.
dx 2
Solution:
A.E is m 2  1  0
m  i
C.F = c1 cos x  c2 sin x
P.I  PI1  PI 2
f1  cos x ; f 2  sin x
 
f1   sin x ; f 2  cos x
Page No. 128
Subject Code & Name: MA 8151 / ENGINEERING MATHEMATICS I Class Notes 2018 - 2019

f 2 f1 f1f 2  1
f2 R
Now, P    dx
f1 f 2  f1 f 2
=   sin x tan xdx
sin 2 x (1  cos 2 x)
=  dx   dx
cos x cos x
=   sec xdx   cos xdx
=  log  sec x  tan x   sin x
f1 R
Q dx
f1 f 2  f1 f 2

=  cos x tan xdx
=  cos x
 y = C.F + Pf1 + Qf2
= c1 cos x  c2 sin x + [ log  sec x  tan x   sin x]cos x  cos x sin x
= c1 cos x  c2 sin x  log(sec x  tan x) cos cos x .
20. d2 y
Solve  y  cot x by using Method of Variation of Parameters.
dx 2
Solution:
The Auxiliary equation (A.E) is m2  1  0  m  i  0  i    i

C.F .  e x ( A cos  x  B sin  x)


 A cos x  B sin x
=A.f1+B.f2
f1  cos x, f 2  sin x
f1   sin x, f 2  cos x

 
f1 f 2'  f 2 f1'  cos x  cos x   sin x   sin x   cos2 x  sin 2 x  1
P.I .  P f1  Q f2
 f2 R
P dx
f1 f 2'  f1' f 2
 sin x cot x
 dx
1
 sin x cos x
 dx
sin x

   cos x dx
P   sin x
f1 R
Q dx
f1 f 2  f1' f 2
'

 cos x cot x
 dx
1

Page No. 129


Subject Code & Name: MA 8151 / ENGINEERING MATHEMATICS I Class Notes 2018 - 2019

 cos x cos x
 dx
sin x

 cos 2 x
 dx
sin x
1  sin 2 x
 dx
sin x
  (cos ecx  sin x) dx
Q = -log(cosecx-cotx)+cosx
P.I .   sin x cos x  log  cosecx  cotx   cosx  sin x
  sin xlog  cosecx  cotx 
Complete solution is
y=C.F +P.I
y=Acosx+Bsinx  sin xlog  cosecx  cotx 
21. Solve y’’ – 2y’ + y = ex logx using method of variation of parameter.
The given differential equation can be written as
( D2 – 2D + 1)y = ex logx
A.E is m2 – 2m + 1=0
m= 1,1
C.F = (c1x + c2)ex
Here f1 = xex, f2 = ex
’ x
f1 = xe + e x
f2’ = ex

Now f1 f2’ - f1’ f2 = xe2x- (xex+ ex ) ex => - e2x


Particular Integral P.I = P f1 + Q f2
f2 R
P   dx

f f  ff
1 2 1 2
x x
e e log x
=  dx   log x dx  x log x  x
e 2 x
f1 R
Q dx
f1 f 2'  f1' f 2
xe x e x log x  x2 
= dx    x log x dx    log x d  
e2 x  2
x2 x2
 log x   d (log x)
2 2
2
x 1 x2
  log x 
2 2 2
 x2 x2 
P.I= ( x log x  x) xe x    log x   e x
 2 4
x 2e x log x x 2e x
 x 2e x log x  x 2e x  
2 4
1 2 x 3 2 x 1 2 x
 x e log x  x e  x e (2 log x  3)
2 4 4
Complete solution is
Page No. 130
Subject Code & Name: MA 8151 / ENGINEERING MATHEMATICS I Class Notes 2018 - 2019

y = C.F + P.I
1
y=(c1 x + c2)ex+ x 2 e x (2 log x  3)
4
Method of undetermined co-efficients
S.No Special form of X Trial solution yp for P.I
x n (or) an x n (or)
1 A0  A1 x  A2 x 2   An x n
a0  a1 x  a2 x 2   an x n
2 e ax (or) P e ax A e ax

an x n e ax (or) A  A1 x  A2 x 2   An x n  e ax
 a0  a1 x  a2 x 2   an x n  e ax
0
3

p sinax (or) q cosax (or) A sinax + B cosax


4
p sinax + q cosax
5 e bx sinax (or) e bx cosax ebx  Asinax  B cosax 
A  A x A x 
0 1 2
2
 An x n  sin  x 
6 xn sinax (or) xn cosax A  A x A x 
0 1 2
2
 An x n  cos  x

22. Solve (D2  2D) y  5e x cos x by using Method of undetermined co-efficients .


(JAN’18)
Given (D  2 D) y  5e cos x ......(1)
2 x

Auxillary equation is
(m2  2m)  0
m(m  2)  0
m=0 ; m=2
 y  C1e0 x  C2 e 2 x  C1  C2 e 2 x ......(2)
Since RHS is of the form ebx cos ax
Let the trial solution be
 y p  ex  A1 cos x  A2 sin x  .......(3)
since y p must satisfy equation (1)
D2 y p  2D y p  5e x cos x ....  4
when D y p  D e x  A1 cos x  A2 sin x  
 ex   A1 sin x  A2 cos x   e x  A1 cos x  A2 sin x 
 e x  A2  A1  sin x   A1  A2  cos x  .....(5)
D 2 y p  e x  A2  A1  cos x   A1  A2  sin x   e x  A2  A1  sin x   A1  A2  cos x 
D2 y p  e x  2 A1 sin x  2 A2 cos x.....(6)
Using (5) & (6) in (4) ,we get
e x  2 A1 sin x  2 A2 cos x   2 e x  A2  A1  sin x   A1  A2  cos x   5e x cos x
e x  2 A1  2 A1  2 A2  sin x   2 A2  2 A1  2 A2  cos x   5e x cos x
ex  2 A2 sin x  2 A1 cos x  5e x cos x
Page No. 131
Subject Code & Name: MA 8151 / ENGINEERING MATHEMATICS I Class Notes 2018 - 2019

2 A2 e x sin x  2 A1e x cos x  5e x cos x


Equating the corresponding coefficients on both sides
2 A2  0 ; 2 A1  5
5
A2  0 ; A1  
2
Substitute A1 and A2 in (3), we get
 5  5
 y p  e x   cos x  0    e x cos x
 2  2
 The general solution of (1) is y=C.F +yp
5
 y  C1  C2 e 2 x  e x cos x
2
23. Solve (D2  2 D  5) y  x 2 by using Method of undetermined coefficients.
Auxillary equation is m2  2m  5  0
2  4  4(1)(5) 2  16
m=   1  2i
2 2
 y  e  x (C1 cos 2 x  C2 sin 2 x)......(2)
Since RHS is of the form x n
Let the trial solution be
 y p  A0  A1 x  A2 x2 .......(3)
since y p must satisfy equation (1)
D2 y p  2D y p  5 y p  x2 ....  4
when D y p  D  A0  A1 x  A2 x 2   A1  2 A2 x
D y p  A1  2 A2 x .....(5)
D y p  2 A2 .....(6)
2

Using (5) & (6) in (4) ,we get


2 A2  2  A1  2 A2 x   5  A0  A1 x  A2 x 2   x 2
2 A2  2 A1  4 A2 x  5 A0  5 A1 x  5 A2 x 2  x 2
Equating the corresponding coefficients on both sides
A0  0 ; 2 A2  2 A1  5 A0  0
 2 A2  2 A1  0.........(7)
1
5 A2  1  A2 
5
1
Substitute A2 in Eqn.(7) , we get A1 
5
x x2
Substitute A0 , A1 and A2 in (3), we get  y p   
5 5

Hence the P.I is  y  5  x  x 


1 2

 The general solution of (1) is y= C.F +yp  y  e  x (C1 cos 2 x  C2 sin 2 x) 


5
 x  x
1 2

Page No. 132

You might also like